You are on page 1of 162

Situation 1 - Jimmy developed this goal for hospitalization.

"To get a handle on my


nervousness." The nurse is going to collaborate with him to reach his goal. Jimmy was
admitted to the hospital because he called his therapist that he planned to asphyxiate
himself with exhaust from his car but frightened instead. He realized he needed help.

1. The nurse recognized that Jimmy had conceptualized his problem and the
next priority goal in the care plan is: 

a. help the client find meaning in his experience


b. help the client to plan alternatives
c. help the client cope with present problem
d. help the client to communicate

2. The nurse is guided that Jimmy is aware of his concerns of the "here and now" when
he crossed out which item from this "list of what to know"

a. anxiety laden unconscious conflicts


b. subjective idea of the range of mild to severe anxiety
c. early signs of anxiety
d. physiological indices of anxiety

3. While Jimmy was discussing the signs and symptoms of anxiety with his
nurse, he recognized that complete disruption of the ability to perceive occurs
in:

a. panic state of anxiety


b. severe anxiety
c. moderate anxiety
d. mild anxiety

4. Jimmy initiates independence and takes an active part in his self care with
the following EXCEPT:

a. agreeing to contact the staff when he is anxious


b. becoming aware of the conscious feeling
c. assessing need for medication and medicating himself
d. writing out a list of behaviors that he identifies as anxious

5. The nurse notes effectiveness of Interventions in using subjective and


objective data in the: 

a. initial plans or order


b. database
c. problem list
d. progress notes

Situation 2 - A research study was under taken in order to identify and analyze a
disabled boy's coping reaction pattern during stress.

6. This study which is a depth study of one boy is a:

a. case study
b. longitudinal study
c. cross-sectional study
d. evaluative study

7. The process recording was the principal tool for data collection. Which of
the following is NOT a part of a process recording?

a. Non verbal narrative account


b. Audio and interpretation
c. Audio-visual recording
d. Verbal narrative account

8. Which of these does NOT happen in a descriptive study?

a. Exploration of relationship between two or more phenomena


b. Exploration of relationships between two or more phenomena
c. Manipulation of phenomenon in real life context
d. Manipulation of a variable

9. The investigator also provided the nursing care of the subject. The
investigator is referred to as a/an.

a. Participant-observer
b. Observer researcher
c. Caregiver
d. Advocate

10. To ensure reliability of the study, the investigator analysis and


interpretations were:

a. subjected to statistical treatment


b. correlated with a list coping behaviors
c. subjected to an inter-observe agreement
d scored and compared standard criteria

Situation 3 - During the morning endorsement, the' outgoing nurse informed the
nursing staff that Regina, 5 years old, was given Flurazepam (Dalmane) 15 mg at
10:00pm because she had trouble going to sleep. Before approaching Regina, the nurse
read the observation of the night nurse.

11. Which of the following approaches of the nurse validates the data
gathered?

a. "I learned that you were up till ten last night, tell me what happened before you
were finally able to sleep and how was your sleep?"
b. "Hmm...You look like you had a very sound sleep. That pill you were given last night
is effective isn't it?"
c. "Regina, did you sleep we!!?" 
d. "Regina, how are you?"

12. Regina is a high school teacher. Which of these information LE^ST


communicate attention and care for her needs for information about her
medicine?

a. Guided by a medication teaching plan go over with her the purpose, indications and
special instructions, about the medication and provide her a checklist 
b. Provide a drug literature 
c. Have an informal conversation about the medication and its effects
d. Ask her what time she would like to watch the informative video about the
medication

13. The nurse engages Regina in the process of mutual inquiry to provide an
opportunity for Regina to

a. face emerging problems realistically


b. conceptualize her problem
c. cope with her present problem
d. perceive her participation in an experience

14. Which of these responses indicate that Regina needs further discussion
regarding special instructions?

a. "I have to take this medicine judiciously."


b. "I know 1 will stop taking the medicine when there is an advice form the doctor for
me to discontinue."
c. "I will inform you and the doctor any untoward reactions I have."
d. "I like taking this sleeping pill. It solves my problem of insomnia. I wish I can take it
for life."

15. Regina commits to herself that she understood and will observe all the
medicine precautions by;

a. affixing her signature to the teaching plan that she has understood the nurse
b. committing what she learned to her memory
c. verbally agreeing with the nurse
d. relying on her husband to remember the precautions

Situation 4 - The nurse-patient relationship is a modality through which the nurse


meets the client's needs.

16. The nurse's most unique tool in working with the emotionally ill client is
his/her:

a. theoretical knowledge
b. personality make up
c. emotional reactions
d. communication skills

17. The psychiatric nurse who is alert to both the physical and emotional
needs of clients is working from the philosophical framework that states: 

a. All behavior is meaningful, communicating a message or a need


b. Human beings are systems of interdependent and interrelated parts 
c. Each individual has the potential for growth and change in the direction of positive
mental health
d. There is a basic similarity among all human beings

18. One way to increase objectivity in dealing with one’s fears and anxieties is
through the process of:

a. observation
b. intervention
c. validation
d. collaboration

19. All of the following response are non therapeutic. Which is the MOST direct
violation of the concept, congruence of behavior?

a. Responding in a punitive manner to the client


b. Rejecting the client as a unique human being
c. Tolerating all behavior in the client 
d. Communicating ambivalent messages to the client

20. The rnentally ill person responds positively to the nurse who is warm and
caring. This demonstration of the nurse’s role as:

a. counselor
b. mother surrogate
c. therapist
d. socializing agent

Situation 5 - The nurse engages the client in a. nurse-patient interaction.

21. The best time to inform the client about terminating the nurse-patient
relationship is

a. when the client asks, how long one relationship would be


b. during the working phase
c. towards the end of the relationship
d. at the start of the relationship

22. The client says, "I want to tell you something but can you promise that
you will keep this, a secret?" A therapeutic response of the nurse is:

a. "Yes, our interaction is confidential provided the information you tell me is not
detrimental to your safety."
b. "Of course yes, this is just between you and me. Promise!"
c. "Yes, it is my principle to uphold my client's rights." 
d. "Yes, you have the right to invoke confidentiality of our interaction."

23. When the nurse respects the client's self-disclosure, this is a gauge for the
nurse's:

a. trustworthiness
b. loyalty
c. integrity
d. professionalism

24. Rapport has been established in the nurse-client interaction time. I am


committed to have this time available for us while you are at the hospital and
ends after your discharge."

a. "The best time to talk is during the nurse-client interaction time. I am committed to
have this time available for us while you are at the hospital and ends after your
discharge." 
b. "Yes, if you keep it confidential, this is part of privileged communication." 
c. "I am committed for your care."
d. "I am sorry, though I would want to, it is against hospital policy."
25. The client has not been visited by relatives for months. He gives a,
telephone number and requests the nurse to call. An appropriate action of the
nurse would be:

a. Inform the attending psychiatric about the request of the client


b. Assist the client to bring his concern to the attention of the social worker
c. "Here (gives her mobile phone). You may call this number now."
d. Ask the client what is the purpose of contacting his relatives

Situation 6 - Camila, 25 years old, was reported to be gradually withdrawing and


isolating herself from friends and family members. She became neglectful of her
personal hygiene. She was observed to be talking irrelevantly and incoherently. She
was diagnosed as schizophrenia

26. The past history of Camila would most probably reveal that her premorbid
personality is:

a. schizoid
b. extrovert
c. ambivert
d. cycloid

27. Camila refuses to relate with others because she: 

a. is irritable
b. feels superior of others
c. anticipates rejection
d. is depressed

28. Which of the following disturbances in interpersonal relationships MOST


often predispose, to the development of schizophrenia?

a. Lack of participation in peer groups


b. Faulty family atmosphere and interaction
c. Extreme rebellion towards authority figures
d. Solo parenting

29. Camila's indifference toward the environment is a compensatory behavior


to overcome:

a. Guilt feelings
b. Ambivalence
c. Narcissistic behavior
d. Insecurity feelings

30. Schizophrenia is a/an: 

a. anxiety disorder
b. neurosis
c. psychosis
d. personality/disorder

Situation 7 - Salome, 80 year old widow, has been observed to be irritable, demanding
and speaking louder than usual. She would prefer to be alone and take her meals by
herself, minimized receiving visitors al home and no longer bothers to answer telephone
calls because of deterioration of her hearing. 'She was brought by her daughter to, the
Geriatic clinic for assessment and treatment.

31. The nurse counsels Salome's daughter that Salome's becoming very loud
and tendency to become aggressive is a/an:

a. beginning indifference to the world around her 


b. attempt to maintain authoritative role
c. overcompensation for hearing loss
d. behavior indicative of unresolved repressed conflict of the part 

32. A nursing diagnosis for Salome is:

a. sensory deprivation
b. social isolation
c. cognitive impairment
d. ego despair

33. The nurse will assist Salome and her daughter to plan a goal which is:

a. adjust to the loss of sensory and .perceptual function


b. participate in conversation and other social situations
c. accept the steady loss of hearing that occurs with aging
d. increase her self-esteem to maintain her authoritative role

34. The daughter understood, the following ways to assist Salome meet her
needs and avoiding which of the following:

a. Using short simple sentences


b. Speaking distinctly and slowly
c. Speaking at eye level and having the client's attention
d. Allowing her to take her meals alone
35. Salome was fitted a hearing aid. She understood the proper .use and wear
of this device when she ways that the battery should be functional, the device
is turned on and adjusted to a:

a. therapeutic level
b. comfortable level
c. prescribed level
d. audible level

Situation 8 - For more than a month now, Cecilia is persistently feeling restless, worried
and feeling as if something dreadful is going to happen. She fears being alone in places
and situations where she thinks that no one might come to rescue her just in case
something happens to her.

36. Cecilia is demonstrating:

a. acrophobia
b. claustrophobia
c. agoraphobia
d. xenophobia

37. Cecilia's problem is that she always sees and thinks negative hence she is
always fearful Phobia is a symptom described as:

a. organic
b. psychosomatic
c. psychotic
d. neurotic

38. Cecilia has a lot of irrational thoughts: The goal of therapy is to modify
her:

a. communication
b. cognition
c. observation
d. perception

39. Cognitive therapy is indicated for Cecilia when she is already able to
handle anxiety reactions. Which of the following should the nurse implement?

a. assist her in recognizing irrational beliefs, and thoughts


b. help find meaning in her behavior
c. provide positive reinforcement for acceptable behavior
d. administer anxiolytic drug

40. After discharge, which of these behaviors indicate a positive result of


being able to overcome her phobia?

a. she read a book in the public library


b. she drives alone along the long expressway 
c. she watches television with the family in the recreation room
d. she joint an art therapy group

Situation 9 - it is the first day of clinical experience of nursing students at the


Psychiatry Ward- During the orientation, the nurse emphasizes that the team members
including nursing students are legally responsible to safeguard patient's records from
loss or destruction or from people not authorized to bead it.

41. It is unethical to tell one's friends and family member’s data bout patients
because doing so is violation of patients’ rights to: 

a. Informed consent
b. Confidentiality
c. Least restrictive environment
d. Civil liberty

42. The nurse must see to it that the written consent of mentally ill patients
must be taken from:

a. Doctor
b. Social worker
c. Parents or legal guardian
d. Law enforcement authorities

43. In an extreme situation and when no other resident or intern is available,


should a nurse receive, telephone orders, the order has to be correctly written
and signed by the physician within. 

a. 24 hours
b. 36 hours
c. 48 hours
d. 12 hours

44. The following are SOAP (Subjective - Objective - Analysis - Plan)


statements on a problem: Anxiety about diagnosis. What is the objective data?

a. Relate patient's feelings to physician initiate and encourage her to verbalize her fears
give emotional support by spending more time with patient, continue to make
necessary explanations regarding diagnostic test.
b. Has periods of crying, frequently verbalizes fear of what diagnostic tests will reveal
c. Anxiety due to the unknown
d. "I’m so worried about what else they'll find wrong with me"

45. Nursing care plans provide very meaningful data for the patient profile and
initial plan because the focus is on the:

a. Summary of chronological notations made by individuals health team members 


b. Identification of patient's responses to medical diagnosis and treatment 
c. Patient's responses to health: and illness as a total person in interaction with the
environment
d. Step procedures for the management of common problems

Situation 10 - Marie is 5 ½ years old and described by the mother as bedwetting at


night. 

46. Which of the following is the MOST common physiological cause of night
bedwetting?

a. deep sleep factors


b. abnormal bladder development or structure problems
c. infections familial and genetic factors

47. All of the following, EXCEPT one comprise the concepts of behavior therapy
program:

a. reward and punishment


b. extinction
c. learning
d. placebo as a form treatment

48. The help Marie who bed wets at night practice acceptable and appropriate
behavior, it is important for the parents to be consistent with the following
approaches EXCEPT:

a discipline with a king attitude


b matter of fact in handling the behavior
c. sympathize for the child
d. be lowing yet firm

49. A therapeutic verbal approach that communicates strong disapproval is: 


a. You are supposed to get up and go in the toilet when you feel you have to go and did
not. The next time you bed wet, I’ll tell your friends and hand your sheets out the
window for them to see."
b. "You are supposed to get up and go in the toilet when you feel you have to go and
you did not. I expect you to from now on without fail." 
c. "If you bed wet, you will change your bed linen and wash the sheets." 
d. "If you don't make an effort to control your bedwetting, I'd be upset and
disappointed."

50. During your conference, the parent inquires how to motivate Marie to be
dry in the morning. Your response which is an immediate intervention would
be:

a. Give a star each time she wakes up dry and every set of five stars, give a prize 
b. Tokens make her materialistic at an early age. Give praise and hugs occasionally 
c. What does you child want that you can give every time he/she wakes up dray in the
morning
d. Promise him/her a long awaited vacation after school is over.

Situation 11 - The nurse is often met with t-he following situations when clients become
angry and hostile.

51. To maintain a therapeutic eye contact and body posture while interacting
with angry and aggressive individual, the nurse should:

a. keep an eye contact while staring at the client


b. keep his/her hands behind his/her back or in one's pocket
c. fold his/her arms across his/her chest 
c. keep an "open" posture, e.g. Hands by sides but palms turned outwards

52. During the pre-interaction phase of the N-P relationship/the nurse


recognizes this normal INITIAL reaction to an assaultive or potentially
assaultive person.

a. To remain and cope with the incident


b. Display empathy towards the patient 
c. To call for help from the other members of the team
d. To stay and fight or run away

53. Which of the following is an accurate way of reporting and recording an


incident?

a. "When asked about his relationship with his father, client became anxious." 
b. "When asked about his relationship with his father, client clenched his jaw/teeth
made a fist and turned away from the nurse."
c. "When asked about his relationship with his father, client was resistant to respond." 
d. "When asked about his relationship with his father, his anger was suppressed." 

54. To encourage thought. Which of the following approaches is NOT


therapeutic?

a. "Why do you feel angry?"


b. "When do you usually feel angry?"
c. "How do you usually express anger?" 
d. "What situations provoke you to be angry?"

55. A patient grabs a chair and about to throw it. The nurse best responds
saying.

a. "Stop! Put that chair down."


b. "Don't be silly."
c. "Stop, the security will be here in a minute."
d. "Calm down."

Situation 12 - Nursing care for the elderly.

56. In planning care for a patient with Parkinson's disease, which of these
nursing diagnoses should have priority?

a. potential for injury


b. altered nutritional state
c. ineffective coping
d. altered mood state

57. A healthy adaptation to aging is primarily related to an individual.

a. Number of accomplishments 
b. Ability to avoid interpersonal conflict
c. Physical health throughout life
d. Personality development in his life span

58. The frequent use of the older client's name by the nurse is MOST effective
in alleviating which of the following responses to old age?

a. Loneliness 
b. Suspicion 
c. Grief
d. Confusion
59. An elderly confused client gets out of bed at night to go to the bathroom
and tries to go to another bed when she returns. The MOST appropriate action
the nurse would take is to:

a. Assign client to a single room


b. Leave a light on all night
c. Remind client to call the nurse when she wants to get up
d. put side rails on the bed

60. An elderly who has lots of regrets, unhappy and miserable1 is


experiencing:

a. Crisis
b. Despair
c. Loss
d. Ambivalence

Situation 13 - Graciela 1 ½ year old is admitted the hospital from the emergency room
with a fracture of the left femur due to a Tall down a flight of stairs. Graciela is placed
oh Bryant's traction. 

61. While on Bryant's traction, which of these observations of Graciela and her
traction apparatus would indicate a decrease in the effectiveness of her
traction?

a. Graciela's buttocks are resting on the bed 


b. The traction weights are hanging 10 inches above the floor 
c. Graciela's legs are suspended at a 90 degree angle to her trunk 
d. The traction ropes move freely through the pulley

62. The nurse notes that the fall might also cause a possible head injury. She
will be observed for signs of increased intracranial pressure which include:

a. Narrowing of the pulse pressure


b. Vomiting
c. Periorbital edema 
d. A positive Kernig's sign

63. Graciela is assessed to have no head injury. The Bryant's traction is


removed. A plaster of Paris his spica is applied. Which of these finding as a
concern of immediate attention that must be reported to the physician
immediately?
a. Graciela is scratching the cast over her abdomen
b. The toes of Graciela's left foot blanch when the nurse applies pressure on them
c. Graciela's cast is still damp
d. The nurse is unable to insert a finger under the edge of Graciela's cast on her left
foot

64. Part of discharge plan is for the nurse to give instructions about the care
of Graciela's cast to the mother. Which of these statements indicate that the
mother understood an important aspect of case care?

a. I will use white shoe polish to keep the cast neat


b. I will place plastic sheeting around the perineal area of the cast
c. I will use cool water to wash the cast
d. I will reinforce cracked areas on the cast with adhesive tape

65. The nurse counsels Graciela's mother ways to safeguard safety white
providing opportunities of Graciela to develop a sense of:

a. Trust
b. Initiative
c. Industry
d. Autonomy

Situation 14 - Jolina is an 18 year old beginning college student. Her mother observed
that she is having problems relating with her friends. She is undecided about her
future. She has lost insight, lost interest in anything and complained and complained of
constant tiredness.

66. Jolina is out on antidepressant drugs. These drugs act on the brain
chemistry, therefore they would be useful in which type of depression?

a. exogenous depression
b. neurotic depression
c. endogenous depression
d. psychotic depression

67. This is a tricyclic antidepressant drug:

a. Venlafaxine (Effexor)
c. Setraline (Zoloft)
b. Flouxetine (Prozac)
d. Imipramine (Tofranil)

68. After one week of antidepressant medication, Jolina still manifests


depression. The nurse evaluates this as;

a. Unusual because action of antidepressant drug is immediate


b. Unexpected because therapeutic effectiveness takes within a few days
c. Expected because therapeutic effectiveness takes 2-4 weeks
d. Ineffective result because perhaps the drug's dosage is inadequate

69. Jolina continues to verbalize feeling sad and hopeless. She is not mixing
well with other clients. One of the nurse's important consideration for Jolina
Initially is to:

a. Formulate a structured schedule so she is able to channel her energies externally


b. Let her alone until she feels like mingling with others
c. Encourage her to join socialization hour so she will start to relate with others
d. Encourage her to join group therapy with other patients

70. During the predischarge conference, the nurse suggests vocational


guidance because it should help Jolina to:

a. Find a good job


b. Make some decision about her future
c. Realistically assess her assets and limitations
d Solve her own problems

Situation 15 - Group Approach" in Nursing.

71. Membership dropout generally occurs in group therapy after a member:

a. Accomplishes his goal in joining the group


b. Discovers that his feelings are shared by the group members
c. Experiences feelings of frustration in the group
d. Discusses personal concerns with group members

72. Which of the following questions illustrates the group role of encourager?

a. What were you saying?


b. Who wants to respond next?
c. Where do you go from here?
d. Why haven't we heard from you?

73. The goal of remotivation therapy is to facilitate:

a. Insight
b. Productivity
c. Socialization
d. Intimacy

74. The treatment of the family as a unit is based on the belief that the family:

a. is a social system and all the members are interrelated components of that system
b. as a unit of society needs the opportunity to change its own destiny
c. who has therapy together will tend to remain together
d. is "contaminated" by the presence of deviant member and all members need
treatment

75. The working phase in therapy group is usually characterized by which of


the following?

a. Caution
b. Cohesiveness
c. Confusion
d. Competition

Situation 16 - The mental health - psychiatric nurse functions in a variety of setting


with different types of clients.

76. Poverty as reflected in prevalence of communicable diseases, malnutrition


and social ills such as street children, homeless and prostitution is a
predisposing factor to mental illness. A community approach to cope with this
problem is for the nurse to support:

a. aggressive family planning methods 


b. provision of social welfare benefits for the poor
c. social action
d. free clinics and more hospitals

77. The MOST cost effective way to meet the mental health needs of the public
is through programs with a priority goal of: 

a. treatment
b. prevention
c. rehabilitation
d. research

78. Lorelle upon discharge was referred to a volunteer group where she has
learned to read patterns, cut out fabric and use a sewing machine to make
simple outfits that will help her earn in the future. What type of activity
therapy is this?
a. Recreational therapy
b. Art therapy
c. Vocational therapy
d. Educational therapy

79. In a residential treatment home for adolescent girl's the clients were
becoming increasingly tense and upset because of shortening of their
recreation time. To die escalate possible anger and aggression among the
clients it is BEST to play:

a. religious music 
b. relaxation music
c. dance music
d. rock music

80. The parents of special children who are behaviorally disturbed need
mental health education. Which of these topics would the school nurse
consider as priority for their parents’ class?

a. Drug education
b. Child abuse
c. Effective parenting
d. Sex education

Situation 17 - Nurse's in all practice areas are likely to come in contact with clients
suffering from acute or chronic drug abuse.

81. The psychodynamic therapy of substance abuse is based upon the premise
that drug abuse is:

a. a common problem brought about by socioeconomic deprivation


b. caused by multiplicity of factors
c. predisposed by an inability to develop appropriate psychological resources to manage
developmental stresses
d. due to biochemical factors

82. Being in contact with reality and the environment is a function of the:

a. conscience
b. ego
c. id
d. super ego
83. Substance abuse is different from substance dependence is than,
substance dependence:

a. includes characteristics of adverse consequences and repeated use


b. requires long term treatment in a hospital based program
c. produces less severe symptoms than that of abuse
d. includes characteristics of tolerance and withdrawal

84. During the detoxification stage, it is a priority for the nurse to:

a. teach skills to recognize and respond to health threatening situations 


b. increase the client's awareness of unsatisfactory protective behaviors 
c. implement behavior modification
d. promote homeostasis and minimize the client’s withdrawal symptoms

85. Commonly known as "shabu" is: 

a. Cannabis Sativa
b. Lysergic add diethylamide
c. Methylenedioxy, methamphetamine
d. Methamphetamine hydrochloride

Situation 18 - It is common that client ask the nurse personal questions.

86. Anticipation of personal questions is given adequate attention during


which phase of the nurse patient relationship?

a. Orientation phase
b. Working phase
c. Pre-interaction phase
d. Termination phase

87. The client asks for the nurse's telephone number, which of these
responses is NOT appropriate?

a. "it is confidential I just don't give it to anyone."


b. "What would you do with my number if I give it to you?"
c. "If I say. No to your request, what are your thoughts about it?”
d. "Are you asking for an official number of the hospital/clinic for your reference?" 

88. When the client asks about the family of the nurse the MOST appropriate
response is:

a. Avoid the situation and redirect the client's attention


b. Give a brief and simple response and focus on the client
c. "Why don't we talk about your family instead?"
d. Introduce another topic like the client's interests

89. When the nurse is asked a personal question, which of these reactions
indicate a need her to introspect?

a. The client is simply curious


b. His/her right to privacy is being intruded
c. The client knows no other way to begin a conversation
d. Some patients are like children in seeking recognition from the nurse

90. It is 10 o'clock of your watch. The client asks, "What time is it?" The
nurse's appropriate response is:

a. "Are you bored?"


b. "It is 10 o’clock."
c. "Why do you ask?"
d. "Guess, what time is it?"

Situation 19 - Ricky is a 12 year old-boy with Down’s syndrome. He stands 5' ½" and
weight 100 lbs. He is slim and walks sluggishly with a limp. He wears a neck brace as
support for his neck. X - ray of cervical spine showed "subluxation of CI in relation to
C2 with cord compression." He attends a school for special education.

91. The classroom teacher consults the school for guidance on how to take
care of Ricky while inside the, classroom. The nurse considers as priority,
Ricky's:

a. Physiological needs
b. Need for self-esteem 
c. Needs for safety and Security
d. Needs for belonging

92. Ricky's mother visited the school nurse. She asked, " What should I do
when Ricky fond his genitalia?" Appropriate response of the nurse is for the
mother to:

a. Divert Ricky's attention and engage him in satisfying activities


b. Tell Ricky that it is wrong to keep fondling his genitalia 
c. Ignore Ricky's behavior because he will outgrow it later
d. Engage him in computer TV games that engage his hands

93. The nurse has one on one health education sessions with Ricky's mother.
The mother understood that for her son to learn to cope and be independent,
she should constantly provide activities for Ricky to be able to:

a. socialize with people


b. eventually go to school alone
c. select and prepare his own food
d. do activities of daily living

94. All of the following activities are appropriate for Ricky EXCEPT:

a. Working with clay


b. Competitive sports
c. Preparing and cooking simple menu
d. Card and table games

95. Ricky's IQ falls within the range of 50-55. He can be expected to:

a. Profit from vocational training with moderate supervision


b. Live successfully in the community
c. Perform simple tasks in closely supervised settings
d. Acquire academic skills of 6th grade; level

Situation 20 - The abuse of dangerous drug is a serious public health concern that
nurses need to address,

96. The nurse should recognize that the unit primarily responsible for
education and awareness of the members of the family on the ill effects of
dangerous drugs is the:

a. law enforcement agencies


b. school
c. church
d. family

97. A drug dependent utilizes this defense mechanism and enables him to
forget shame and pain.

a. repression
b. rationalization
c. projection
d. sublimation

98. This drug produces mirthfulness, fantasies, flight of ideas, loss of train of
thought, distortion of size, distance and time, and "bloodshot eyes", due to
dilated pupils.

a. Opiates
b. LSD
c. Marijuana
d. Heroin

99. The nurse evaluates that-.her health teaching to a group of high school
boys is effective if these students recognize which of the following dangers of
inhalant abuse. 

a. Sudden death from cardiac or respiratory depression 


b. Danger of acquiring hepatitis or AIDS
c. Experience of "blackout"
d. Psychological dependence after prolonged use

100. The mother of a drug dependent would never consider referring her son
to a drug rehabilitation agency because she fears her son might just becomes
worse while relating with other drugs users. The mother's behavior can be
described as:

a. Unhelpful
b. Codependent 
c. Caretaking
d. Supportive

ANSWER KEY:
1. C
2. C
3. B
4. A
5. D
6. A
7. C
8. D
9. D
10. A
11. A
12. D
13. D
14. D
15. A
16. D
17. C
18. B
19. 
20. 
21. 
22. 
23. 
24. 
25. A
26. A
27. A
28. B
29. C
30. C
31. A
32. A
33. A
34. D
35. D
36. C
37. D
38. B
39. A
40. A
41. B
42. C
43. A
44. B
45. C
46. 
47. 
48. 
49. 
50. 
51. D
52. B
53. B
54. A
55. A
56. A
57. C
58. D
59. A
60. B
61. A
62. B
63. D
64. D
65. D
66. B
67. D
68. C
69. C
70. C
71. C
72. B
73. B
74. A
75. B
76. B
77. B
78. C
79. B
80. C
81. B
82. B
83. D
84. D
85. D
86. B
87. A
88. B
89. D
90. B
91. C
92. 
93. D
94. B
95. C
96. D
97. A
98. B
99. A
100. A
1. Following surgery, Mario complains of mild incisional pain while performing deep-
breathing and coughing exercises. The nurse’s best response would be:
A. “Pain will become less each day.”
B. “This is a normal reaction after surgery.”
C. “With a pillow, apply pressure against the incision.”
D. “I will give you the pain medication the physician ordered.”

Answer: (C) “With a pillow, apply pressure against the incision.”


Applying pressure against the incision with a pillow will help lessen the intra-
abdominal pressure created by coughing which causes tension on the incision
that leads to pain.

2. The nurse needs to carefully assess the complaint of pain of the elderly
because older people
A. are expected to experience chronic pain
B. have a decreased pain threshold
C. experience reduced sensory perception
D. have altered mental function

Answer: (C) experience reduced sensory perception


Degenerative changes occur in the elderly. The response to pain in the elderly
maybe lessened because of reduced acuity of touch, alterations in neural
pathways and diminished processing of sensory data.

3. Mary received AtropineSO4 as a pre-medication 30 minutes ago and is now


complaining of dry mouth and her PR is higher, than before the medication was
administered. The nurse’s best
A. The patient is having an allergic reaction to the drug.
B. The patient needs a higher dose of this drug
C. This is normal side-effect of AtSO4
D. The patient is anxious about upcoming surgery

Answer: (C) This is normal side-effect of AtSO4


Atropine sulfate is a vagolytic drug that decreases oropharyngeal secretions and
increases the heart rate.

4. Ana’s postoperative vital signs are a blood pressure of 80/50 mm Hg, a pulse
of 140, and respirations of 32. Suspecting shock, which of the following orders
would the nurse question?
A. Put the client in modified Trendelenberg's position.
B. Administer oxygen at 100%.
C. Monitor urine output every hour.
D. Administer Demerol 50mg IM q4h

Answer: (D) Administer Demerol 50mg IM q4h


Administering Demerol, which is a narcotic analgesic, can depress respiratory
and cardiac function and thus not given to a patient in shock. What is needed is
promotion for adequate oxygenation and perfusion. All the other interventions
can be expected to be done by the nurse.

5. Mr. Pablo, diagnosed with Bladder Cancer, is scheduled for a cystectomy with
the creation of an ileal conduit in the morning. He is wringing his hands and
pacing the floor when the nurse enters his room. What is the best approach?
A. "Good evening, Mr. Pablo. Wasn't it a pleasant day, today?"
B. "Mr, Pablo, you must be so worried, I'll leave you alone with your thoughts.
C. “Mr. Pablo, you'll wear out the hospital floors and yourself at this rate."
D. "Mr. Pablo, you appear anxious to me. How are you feeling about tomorrow's
surgery?"

Answer: (D) "Mr. Pablo, you appear anxious to me. How are you feeling about
tomorrow's surgery?"
The client is showing signs of anxiety reaction to a stressful event. Recognizing
the client’s anxiety conveys acceptance of his behavior and will allow for
verbalization of feelings and concerns.

6. After surgery, Gina returns from the Post-anesthesia Care Unit (Recovery
Room) with a nasogastric tube in place following a gall bladder surgery. She
continues to complain of nausea. Which action would the nurse take?
A. Call the physician immediately.
B. Administer the prescribed antiemetic.
C. Check the patency of the nasogastric tube for any obstruction.
D. Change the patient’s position.

Answer: (C) Check the patency of the nasogastric tube for any obstruction.
Nausea is one of the common complaints of a patient after receiving general
anesthesia. But this complaint could be aggravated by gastric distention
especially in a patient who has undergone abdominal surgery. Insertion of the
NGT helps relieve the problem. Checking on the patency of the NGT for any
obstruction will help the nurse determine the cause of the problem and institute
the necessary intervention.

7. Mr. Perez is in continuous pain from cancer that has metastasized to the bone.
Pain medication provides little relief and he refuses to move. The nurse should
plan to:
A. Reassure him that the nurses will not hurt him
B. Let him perform his own activities of daily living
C. Handle him gently when assisting with required care
D. Complete A.M. care quickly as possible when necessary

Answer: (C) Handle him gently when assisting with required care
Patients with cancer and bone metastasis experience severe pain especially when
moving. Bone tumors weaken the bone to appoint at which normal activities and
even position changes can lead to fracture. During nursing care, the patient
needs to be supported and handled gently.

8. A client returns from the recovery room at 9AM alert and oriented, with an IV
infusing. His pulse is 82, blood pressure is 120/80, respirations are 20, and all
are within normal range. At 10 am and at 11 am, his vital signs are stable. At
noon, however, his pulse rate is 94, blood pressure is 116/74, and respirations
are 24. What nursing action is most appropriate?

A. Notify his physician.


B. Take his vital signs again in 15 minutes.
C. Take his vital signs again in an hour.
D. Place the patient in shock position.

Answer: (B) Take his vital signs again in 15 minutes.


Monitoring the client’s vital signs following surgery gives the nurse a sound
information about the client’s condition. Complications can occur during this
period as a result of the surgery or the anesthesia or both. Keeping close track of
changes in the VS and validating them will help the nurse initiate interventions
to prevent complications from occurring.

9. A 56 year old construction worker is brought to the hospital unconscious after


falling from a 2-story building. When assessing the client, the nurse would be
most concerned if the assessment revealed:

A. Reactive pupils
B. A depressed fontanel
C. Bleeding from ears
D. An elevated temperature

Answer: (C) Bleeding from ears


The nurse needs to perform a thorough assessment that could indicate
alterations in cerebral function, increased intracranial pressures, fractures and
bleeding. Bleeding from the ears occurs only with basal skull fractures that can
easily contribute to increased intracranial pressure and brain herniation

10. Which of the ff. statements by the client to the nurse indicates a risk factor
for CAD?
A. “I exercise every other day.”
B. “My father died of Myasthenia Gravis.”
C. “My cholesterol is 180.”
D. “I smoke 1 1/2 packs of cigarettes per day.”

Answer: (D) “I smoke 1 1/2 packs of cigarettes per day.”


Smoking has been considered as one of the major modifiable risk factors for
coronary artery disease. Exercise and maintaining normal serum cholesterol
levels help in its prevention.

11. Mr. Braga was ordered Digoxin 0.25 mg. OD. Which is poor knowledge
regarding this drug?
A. It has positive inotropic and negative chronotropic effects
B. The positive inotropic effect will decrease urine output
C. Toxixity can occur more easily in the presence of hypokalemia, liver and renal
problems
D. Do not give the drug if the apical rate is less than 60 beats per minute.

Answer: (B) The positive inotropic effect will decrease urine output
Inotropic effect of drugs on the heart causes increase force of its contraction.
This increases cardiac output that improves renal perfusion resulting in an
improved urine output.

12. Valsalva maneuver can result in bradycardia. Which of the following activities
will not stimulate Valsalva's maneuver?
A. Use of stool softeners.
B. Enema administration
C. Gagging while toothbrushing.
D. Lifting heavy objects

Answer: (A) Use of stool softeners.


Straining or bearing down activities can cause vagal stimulation that leads to
bradycardia. Use of stool softeners promote easy bowel evacuation that prevents
straining or the valsalva maneuver.
13. The nurse is teaching the patient regarding his permanent artificial
pacemaker. Which information
given by the nurse shows her knowledge deficit about the artificial cardiac
pacemaker?

A. take the pulse rate once a day, in the morning upon awakening
B. may be allowed to use electrical appliances
C. have regular follow up care
D. may engage in contact sports

Answer: (D) may engage in contact sports


The client should be advised by the nurse to avoid contact sports. This will
prevent trauma to the area of the pacemaker generator.

14. A patient with angina pectoris is being discharged home with nitroglycerine
tablets. Which of the
following instructions does the nurse include in the teaching?

A. “When your chest pain begins, lie down, and place one tablet under your
tongue. If the pain continues, take another tablet in 5 minutes.”
B. “Place one tablet under your tongue. If the pain is not relieved in 15 minutes,
go to the hospital.”
C. “Continue your activity, and if the pain does not go away in 10 minutes, begin
taking the nitro tablets one every 5 minutes for 15 minutes, then go lie down.”
D. “Place one Nitroglycerine tablet under the tongue every five minutes for three
doses. Go to the hospital if the pain is unrelieved.

Answer: (D) “Place one Nitroglycerine tablet under the tongue every five minutes
for three doses. Go to the hospital if the pain is unrelieved.
Angina pectoris is caused by myocardial ischemia related to decreased coronary
blood supply. Giving nitroglycerine will produce coronary vasodilation that
improves the coronary blood flow in 3 – 5 mins. If the chest pain is unrelieved,
after three tablets, there is a possibility of acute coronary occlusion that requires
immediate medical attention.

15. A client with chronic heart failure has been placed on a diet restricted to
2000mg. of sodium per day. The client demonstrates adequate knowledge if
behaviors are evident such as not salting food and avoidance of which food?

A. Whole milk
B. Canned sardines
C. Plain nuts
D. Eggs

Answer: (B) Canned sardines


Canned foods are generally rich in sodium content as salt is used as the main
preservative.

16. A student nurse is assigned to a client who has a diagnosis of


thrombophlebitis. Which action by this team member is most appropriate?
A. Apply a heating pad to the involved site.
B. Elevate the client's legs 90 degrees.
C. Instruct the client about the need for bed rest.
D. Provide active range-of-motion exercises to both legs at least twice every
shift.

Answer: (C) Instruct the client about the need for bed rest.
In a client with thrombophlebitis, bedrest will prevent the dislodgment of the clot
in the extremity which can lead to pulmonary embolism.

17. A client receiving heparin sodium asks the nurse how the drug works. Which
of the following points would the nurse include in the explanation to the client?
A. It dissolves existing thrombi.
B. It prevents conversion of factors that are needed in the formation of clots.
C. It inactivates thrombin that forms and dissolves existing thrombi.
D. It interferes with vitamin K absorption.

Answer: (B) It prevents conversion of factors that are needed in the formation of
clots.
Heparin is an anticoagulant. It prevents the conversion of prothrombin to
thrombin. It does not dissolve a clot.

18. The nurse is conducting an education session for a group of smokers in a


“stop smoking” class.
Which finding would the nurse state as a common symptom of lung cancer? :

A. Dyspnea on exertion
B. Foamy, blood-tinged sputum
C. Wheezing sound on inspiration
D. Cough or change in a chronic cough

Answer: (D) Cough or change in a chronic cough


Cigarette smoke is a carcinogen that irritates and damages the respiratory
epithelium. The irritation causes the cough which initially maybe dry, persistent
and unproductive. As the tumor enlarges, obstruction of the airways occurs and
the cough may become productive due to infection.

19. Which is the most relevant knowledge about oxygen administration to a


client with COPD?
A. Oxygen at 1-2L/min is given to maintain the hypoxic stimulus for breathing.
B. Hypoxia stimulates the central chemoreceptors in the medulla that makes the
client breath.
C. Oxygen is administered best using a non-rebreathing mask
D. Blood gases are monitored using a pulse oximeter.

Answer: (A) Oxygen at 1-2L/min is given to maintain the hypoxic stimulus for
breathing.
COPD causes a chronic CO2 retention that renders the medulla insensitive to the
CO2 stimulation for breathing. The hypoxic state of the client then becomes the
stimulus for breathing. Giving the clientoxygen in low concentrations will
maintain the client’s hypoxic drive.

20. When suctioning mucus from a client's lungs, which nursing action would be
least appropriate?
A. Lubricate the catheter tip with sterile saline before insertion.
B. Use sterile technique with a two-gloved approach
C. Suction until the client indicates to stop or no longer than 20 second
D. Hyperoxygenate the client before and after suctioning

Answer: (C) Suction until the client indicates to stop or no longer than 20 second
One hazard encountered when suctioning a client is the development of hypoxia.
Suctioning sucks not only the secretions but also the gases found in the airways.
This can be prevented by suctioning the client for an average time of 5-10
seconds and not more than 15 seconds and hyperoxygenating the client before
and after suctioning.

21. Dr. Santos prescribes oral rifampin (Rimactane) and isoniazid (INH) for a
client with a positive Tuberculin skin test. When informing the client of this
decision, the nurse knows that the purpose of this choice of treatment is to

A. Cause less irritation to the gastrointestinal tract


B. Destroy resistant organisms and promote proper blood levels of the drugs
C. Gain a more rapid systemic effect
D. Delay resistance and increase the tuberculostatic effect

Answer: (D) Delay resistance and increase the tuberculostatic effect


Pulmonary TB is treated primarily with chemotherapeutic agents for 6-12 mons.
A prolonged treatment duration is necessary to ensure eradication of the
organisms and to prevent relapse. The increasing prevalence of drug resistance
points to the need to begin the treatment with drugs in combination. Using drugs
in combination can delay the drug resistance.

22. Mario undergoes a left thoracotomy and a partial pneumonectomy. Chest


tubes are inserted, and one-bottle water-seal drainage is instituted in the
operating room. In the
postanesthesia care unit Mario is placed in Fowler's position on either his right
side or on his back to

A. Reduce incisional pain.


B. Facilitate ventilation of the left lung.
C. Equalize pressure in the pleural space.
D. Increase venous return

Answer: (B) Facilitate ventilation of the left lung.


Since only a partial pneumonectomy is done, there is a need to promote
expansion of this remaining Left lung by positioning the client on the opposite
unoperated side.

23. A client with COPD is being prepared for discharge. The following are
relevant instructions to the client regarding the use of an oral inhaler EXCEPT
A. Breath in and out as fully as possible before placing the mouthpiece inside the
mouth.
B. Inhale slowly through the mouth as the canister is pressed down
C. Hold his breath for about 10 seconds before exhaling
D. Slowly breath out through the mouth with pursed lips after inhaling the drug.

Answer: (D) Slowly breath out through the mouth with pursed lips after inhaling
the drug.
If the client breathes out through the mouth with pursed lips, this can easily
force the just inhaled drug out of the respiratory tract that will lessen its
effectiveness.

24. A client is scheduled for a bronchoscopy. When teaching the client what to
expect afterward, the nurse's highest priority of information would be
A. Food and fluids will be withheld for at least 2 hours.
B. Warm saline gargles will be done q 2h.
C. Coughing and deep-breathing exercises will be done q2h.
D. Only ice chips and cold liquids will be allowed initially.

Answer: (A) Food and fluids will be withheld for at least 2 hours.
Prior to bronchoscopy, the doctors sprays the back of the throat with anesthetic
to minimize the gag reflex and thus facilitate the insertion of the bronchoscope.
Giving the client food and drink after the procedure without checking on the
return of the gag reflex can cause the client to aspirate. The gag reflex usually
returns after two hours.

25. The nurse enters the room of a client with chronic obstructive pulmonary
disease. The client's nasal cannula oxygen is running at a rate of 6 L per minute,
the skin color is pink, and the respirations are 9 per minute and shallow. What is
the nurse’s best initial action?

A. Take heart rate and blood pressure.


B. Call the physician.
C. Lower the oxygen rate.
D. Position the client in a Fowler's position.

Answer: (C) Lower the oxygen rate.


The client with COPD is suffering from chronic CO2 retention. The hypoxic drive
is his chief stimulus for breathing. Giving O2 inhalation at a rate that is more
than 2-3L/min can make the client lose his hypoxic drive which can be assessed
as decreasing RR.

26. The nurse is preparing her plan of care for her patient diagnosed with
pneumonia. Which is the most appropriate nursing diagnosis for this patient?
A. Fluid volume deficit
B. Decreased tissue perfusion.
C. Impaired gas exchange.
D. Risk for infection

Answer: (C) Impaired gas exchange.


Pneumonia, which is an infection, causes lobar consolidation thus impairing gas
exchange between the alveoli and the blood. Because the patient would require
adequate hydration, this makes him prone to fluid volume excess.

27. A nurse at the weight loss clinic assesses a client who has a large abdomen
and a rounded face. Which additional assessment finding would lead the nurse to
suspect that the client has Cushing’s syndrome rather than obesity?
A. large thighs and upper arms
B. pendulous abdomen and large hips
C. abdominal striae and ankle enlargement
D. posterior neck fat pad and thin extremities

Answer: (D) posterior neck fat pad and thin extremities


“Buffalo hump” is the accumulation of fat pads over the upper back and neck.
Fat may also accumulate on the face. There is truncal obesity but the extremities
are thin. All these are noted in a client with Cushing’s syndrome.

28. Which statement by the client indicates understanding of the possible side
effects of Prednisone therapy?
A. “I should limit my potassium intake because hyperkalemia is a side-effect of
this drug.”
B. “I must take this medicine exactly as my doctor ordered it. I shouldn’t skip
doses.”
C. “This medicine will protect me from getting any colds or infection.”
D. “My incision will heal much faster because of this drug.”

Answer: (B) “I must take this medicine exactly as my doctor ordered it. I
shouldn’t skip doses.”
The possible side effects of steroid administration are hypokalemia, increase
tendency to infection and poor wound healing. Clients on the drug must follow
strictly the doctor’s order since skipping the drug can lower the drug level in the
blood that can trigger acute adrenal insufficiency or Addisonian Crisis

29. A client, who is suspected of having Pheochromocytoma, complains of


sweating, palpitation and headache. Which assessment is essential for the nurse
to make first?
A. Pupil reaction
B. Hand grips
C. Blood pressure
D. Blood glucose

Answer: (C) Blood pressure


Pheochromocytoma is a tumor of the adrenal medulla that causes an increase
secretion of catecholamines that can elevate the blood pressure.

30. The nurse is attending a bridal shower for a friend when another guest, who
happens to be a diabetic, starts to tremble and complains of dizziness. The next
best action for the nurse to take is to:
A. Encourage the guest to eat some baked macaroni
B. Call the guest’s personal physician
C. Offer the guest a cup of coffee
D. Give the guest a glass of orange juice

Answer: (D) Give the guest a glass of orange juice


In diabetic patients, the nurse should watch out for signs of hypoglycemia
manifested by dizziness, tremors, weakness, pallor diaphoresis and tachycardia.
When this occurs in a conscious client, he should be given immediately
carbohydrates in the form of fruit juice, hard candy, honey or, if unconscious,
glucagons or dextrose per IV.

31. An adult, who is newly diagnosed with Graves disease, asks the nurse, “Why
do I need to take
Propanolol (Inderal)?” Based on the nurse’s understanding of the medication and
Grave’s
disease, the best response would be:

A. “The medication will limit thyroid hormone secretion.”


B. “The medication limit synthesis of the thyroid hormones.”
C. “The medication will block the cardiovascular symptoms of Grave’s disease.”
D. “The medication will increase the synthesis of thyroid hormones.”

Answer: (C) “The medication will block the cardiovascular symptoms of Grave’s
disease.”
Propranolol (Inderal) is a beta-adrenergic blocker that controls the
cardiovascular manifestations brought about by increased secretion of the
thyroid hormone in Grave’s disease.

32. During the first 24 hours after thyroid surgery, the nurse should include in
her care:
A. Checking the back and sides of the operative dressing
B. Supporting the head during mild range of motion exercise
C. Encouraging the client to ventilate her feelings about the surgery
D. Advising the client that she can resume her normal activities immediately

Answer: (A) Checking the back and sides of the operative dressing
Following surgery of the thyroid gland, bleeding is a potential complication. This
can best be assessed by checking the back and the sides of the operative
dressing as the blood may flow towards the side and back leaving the front dry
and clear of drainage.

33. On discharge, the nurse teaches the patient to observe for signs of surgically
induced hypothyroidism. The nurse would know that the patient understands the
teaching when she states she should notify the MD if she develops:

A. Intolerance to heat
B. Dry skin and fatigue
C. Progressive weight gain
D. Insomnia and excitability

Answer: (C) Progressive weight gain


Hypothyroidism, a decrease in thyroid hormone production, is characterized by
hypometabolism that manifests itself with weight gain.

34. What is the best reason for the nurse in instructing the client to rotate
injection sites for insulin?
A. Lipodystrophy can result and is extremely painful
B. Poor rotation technique can cause superficial hemorrhaging
C. Lipodystrophic areas can result, causing erratic insulin absorption rates from
these
D. Injection sites can never be reused

Answer: (C) Lipodystrophic areas can result, causing erratic insulin absorption
rates from these
Lipodystrophy is the development of fibrofatty masses at the injection site
caused by repeated use of an injection site. Injecting insulin into these scarred
areas can cause the insulin to be poorly absorbed and lead to erratic reactions.

35. Which of the following would be inappropriate to include in a diabetic


teaching plan?
A. Change position hourly to increase circulation
B. Inspect feet and legs daily for any changes
C. Keep legs elevated on 2 pillows while sleeping
D. Keep the insulin not in use in the refrigerator

Answer: (C) Keep legs elevated on 2 pillows while sleeping


The client with DM has decreased peripheral circulation caused by
microangiopathy. Keeping the legs elevated during sleep will further cause
circulatory impairment.
36. Included in the plan of care for the immediate post-gastroscopy period will
be:
A. Maintain NGT to intermittent suction
B. Assess gag reflex prior to administration of fluids
C. Assess for pain and medicate as ordered
D. Measure abdominal girth every 4 hours

Answer: (B) Assess gag reflex prior to administration of fluids


The client, after gastroscopy, has temporary impairment of the gag reflex due to
the anesthetic that has been sprayed into his throat prior to the procedure.
Giving fluids and food at this time can lead to aspiration.

36. Included in the plan of care for the immediate post-gastroscopy period will
be:
A. Maintain NGT to intermittent suction
B. Assess gag reflex prior to administration of fluids
C. Assess for pain and medicate as ordered
D. Measure abdominal girth every 4 hours

Answer: (B) Assess gag reflex prior to administration of fluids


The client, after gastroscopy, has temporary impairment of the gag reflex due to
the anesthetic that has been sprayed into his throat prior to the procedure.
Giving fluids and food at this time can lead to aspiration.

37. Which description of pain would be most characteristic of a duodenal ulcer?


A. Gnawing, dull, aching, hungerlike pain in the epigastric area that is relieved
by food intake
B. RUQ pain that increases after meal
C. Sharp pain in the epigastric area that radiates to the right shoulder
D. A sensation of painful pressure in the midsternal area

Answer: (A) Gnawing, dull, aching, hungerlike pain in the epigastric area that is
relieved by food intake
Duodenal ulcer is related to an increase in the secretion of HCl. This can be
buffered by food intake thus the relief of the pain that is brought about by food
intake.

38. The client underwent Billroth surgery for gastric ulcer. Post-operatively, the
drainage from his NGT is thick and the volume of secretions has dramatically
reduced in the last 2 hours and the client feels like vomiting. The most
appropriate nursing action is to:

A. Reposition the NGT by advancing it gently NSS


B. Notify the MD of your findings
C. Irrigate the NGT with 50 cc of sterile
D. Discontinue the low-intermittent suction

Answer: (B) Notify the MD of your findings


The client’s feeling of vomiting and the reduction in the volume of NGT drainage
that is thick are signs of possible abdominal distention caused by obstruction of
the NGT. This should be reported immediately to the MD to prevent tension and
rupture on the site of anastomosis caused by gastric distention.

39. After Billroth II Surgery, the client developed dumping syndrome. Which of
the following should
the nurse exclude in the plan of care?

A. Sit upright for at least 30 minutes after meals


B. Take only sips of H2O between bites of solid food
C. Eat small meals every 2-3 hours
D. Reduce the amount of simple carbohydrate in the diet

Answer: (A) Sit upright for at least 30 minutes after meals


The dumping syndrome occurs within 30 mins after a meal due to rapid gastric
emptying, causing distention of the duodenum or jejunum produced by a bolus
of food. To delay the emptying, the client has to lie down after meals. Sitting up
after meals will promote the dumping syndrome.

40. The laboratory of a male patient with Peptic ulcer revealed an elevated titer
of Helicobacter pylori.
Which of the following statements indicate an understanding of this data?

A. Treatment will include Ranitidine and Antibiotics


B. No treatment is necessary at this time
C. This result indicates gastric cancer caused by the organism
D. Surgical treatment is necessary

Answer: (A) Treatment will include Ranitidine and Antibiotics


One of the causes of peptic ulcer is H. Pylori infection. It releases toxin that
destroys the gastric and duodenal mucosa which decreases the gastric
epithelium’s resistance to acid digestion. Giving antibiotics will control the
infection and Ranitidine, which is a histamine-2 blocker, will reduce acid
secretion that can lead to ulcer.

41. What instructions should the client be given before undergoing a


paracentesis?
A. NPO 12 hours before procedure
B. Empty bladder before procedure
C. Strict bed rest following procedure
D. Empty bowel before procedure

Answer: (B) Empty bladder before procedure


Paracentesis involves the removal of ascitic fluid from the peritoneal cavity
through a puncture made below the umbilicus. The client needs to void before
the procedure to prevent accidental puncture of a distended bladder during the
procedure.
42. The husband of a client asks the nurse about the protein-restricted diet
ordered because of advanced liver disease. What statement by the nurse would
best explain the purpose of the diet?

A. “The liver cannot rid the body of ammonia that is made by the breakdown of
protein in the digestive system.”
B. “The liver heals better with a high carbohydrates diet rather than protein.”
C. “Most people have too much protein in their diets. The amount of this diet is
better for liver healing.”
D. “Because of portal hyperemesis, the blood flows around the liver and
ammonia made from protein collects in the brain causing hallucinations.”

Answer: (A) “The liver cannot rid the body of ammonia that is made by the
breakdown of protein in the digestive system.”
The largest source of ammonia is the enzymatic and bacterial digestion of dietary
and blood proteins in the GI tract. A protein-restricted diet will therefore
decrease ammonia production.

43. Which of the drug of choice for pain controls the patient with acute
pancreatitis?
A. Morphine
B. NSAIDS
C. Meperidine
D. Codeine

Answer: (C) Meperidine


Pain in acute pancreatitis is caused by irritation and edema of the inflamed
pancreas as well as spasm due to obstruction of the pancreatic ducts. Demerol is
the drug of choice because it is less likely to cause spasm of the Sphincter of
Oddi unlike Morphine which is spasmogenic.

44. Immediately after cholecystectomy, the nursing action that should assume
the highest priority is:
A. encouraging the client to take adequate deep breaths by mouth
B. encouraging the client to cough and deep breathe
C. changing the dressing at least BID
D. irrigate the T-tube frequently

Answer: (B) encouraging the client to cough and deep breathe


Cholecystectomy requires a subcostal incision. To minimize pain, clients have a
tendency to take shallow breaths which can lead to respiratory complications like
pneumonia and atelectasis. Deep breathing and coughing exercises can help
prevent such complications.

45. A Sengstaken-Blakemore tube is inserted in the effort to stop the bleeding


esophageal varices in a patient with complicated liver cirrhosis. Upon insertion of
the tube, the client complains of difficulty of breathing. The first action of the
nurse is to:

A. Deflate the esophageal balloon


B. Monitor VS
C. Encourage him to take deep breaths
D. Notify the MD

Answer: (A) Deflate the esophageal balloon


When a client with a Sengstaken-Blakemore tube develops difficulty of
breathing, it means the tube is displaced and the inflated balloon is in the
oropharynx causing airway obstruction

46. The client presents with severe rectal bleeding, 16 diarrheal stools a day,
severe abdominal pain, tenesmus and dehydration. Because of these symptoms
the nurse should be alert for other problems associated with what disease?

A. Chrons disease
B. Ulcerative colitis
C. Diverticulitis
D. Peritonitis

Answer: (B) Ulcerative colitis


Ulcerative colitis is a chronic inflammatory condition producing edema and
ulceration affecting the entire colon. Ulcerations lead to sloughing that causes
stools as many as 10-20 times a day that is filled with blood, pus and mucus.
The other symptoms mentioned accompany the problem.

47. A client is being evaluated for cancer of the colon. In preparing the client for
barium enema, the nurse should:

A. Give laxative the night before and a cleansing enema in the morning before
the test
B. Render an oil retention enema and give laxative the night before
C. Instruct the client to swallow 6 radiopaque tablets the evening before the
study
D. Place the client on CBR a day before the study

Answer: (A) Give laxative the night before and a cleansing enema in the morning
before the test
Barium enema is the radiologic visualization of the colon using a die. To obtain
accurate results in this procedure, the bowels must be emptied of fecal material
thus the need for laxative and enema.

48. The client has a good understanding of the means to reduce the chances of
colon cancer when
he states:

A. “I will exercise daily.”


B. “I will include more red meat in my diet.”
C. “I will have an annual chest x-ray.”
D. “I will include more fresh fruits and vegetables in my diet.”

Answer: (D) “I will include more fresh fruits and vegetables in my diet.”
Numerous aspects of diet and nutrition may contribute to the development of
cancer. A low-fiber diet, such as when fresh fruits and vegetables are minimal or
lacking in the diet, slows transport of materials through the gut which has been
linked to colorectal cancer.

49. Days after abdominal surgery, the client’s wound dehisces. The safest
nursing intervention when
this occurs is to

A. Cover the wound with sterile, moist saline dressing


B. Approximate the wound edges with tapes
C. Irrigate the wound with sterile saline
D. Hold the abdominal contents in place with a sterile gloved hand

Answer: (A) Cover the wound with sterile, moist saline dressing
Dehiscence is the partial or complete separation of the surgical wound edges.
When this occurs, the client is placed in low Fowler’s position and instructed to
lie quietly. The wound should be covered to protect it from exposure and the
dressing must be sterile to protect it from infection and moist to prevent the
dressing from sticking to the wound which can disturb the healing process.

50. An intravenous pyelogram reveals that Paulo, age 35, has a renal calculus.
He is believed to have a small stone that will pass spontaneously. To increase
the chance of the stone passing, the nurse would instruct the client to force
fluids and to

A. Strain all urine.


B. Ambulate.
C. Remain on bed rest.
D. Ask for medications to relax him.

Answer: (B) Ambulate.


Free unattached stones in the urinary tract can be passed out with the urine by
ambulation which can mobilize the stone and by increased fluid intake which will
flush out the stone during urination.

51. A female client is admitted with a diagnosis of acute renal failure. She is
awake, alert, oriented, and complaining of severe back pain, nausea and
vomiting and abdominal cramps. Her vital signs are blood pressure 100/70 mm
Hg, pulse 110, respirations 30, and oral temperature 100.4°F (38°C). Her
electrolytes are sodium 120 mEq/L, potassium 5.2 mEq/L; her urinary output for
the first 8 hours is 50 ml. The client is displaying signs of which electrolyte
imbalance?
A. Hyponatremia
B. Hyperkalemia
C. Hyperphosphatemia
D. Hypercalcemia

Answer: (A) Hyponatremia


The normal serum sodium level is 135 – 145 mEq/L. The client’s serum sodium
is below normal. Hyponatremia also manifests itself with abdominal cramps and
nausea and vomiting
52. Assessing the laboratory findings, which result would the nurse most likely
expect to find in a
client with chronic renal failure?

A. BUN 10 to 30 mg/dl, potassium 4.0 mEq/L, creatinine 0.5 to 1.5 mg/dl


B. Decreased serum calcium, blood pH 7.2, potassium 6.5 mEq/L
C. BUN 15 mg/dl, increased serum calcium, creatinine l.0 mg/dl
D. BUN 35 to 40 mg/dl, potassium 3.5 mEq/L, pH 7.35, decreased serum
calcium

Answer: (B) Decreased serum calcium, blood pH 7.2, potassium 6.5 mEq/L
Chronic renal failure is usually the end result of gradual tissue destruction and
loss of renal function. With the loss of renal function, the kidneys ability to
regulate fluid and electrolyte and acid base balance results. The serum Ca
decreases as the kidneys fail to excrete phosphate, potassium and hydrogen ions
are retained.

53. Treatment with hemodialysis is ordered for a client and an external shunt is
created. Which nursing action would be of highest priority with regard to the
external shunt?

A. Heparinize it daily.
B. Avoid taking blood pressure measurements or blood samples from the
affected arm.
C. Change the Silastic tube daily.
D. Instruct the client not to use the affected arm.

Answer: (B) Avoid taking blood pressure measurements or blood samples from
the affected arm.
In the client with an external shunt, don’t use the arm with the vascular access
site to take blood pressure readings, draw blood, insert IV lines, or give
injections because these procedures may rupture the shunt or occlude blood flow
causing damage and obstructions in the shunt.

54. Romeo Diaz, age 78, is admitted to the hospital with the diagnosis of benign
prostatic hyperplasia (BPH). He is scheduled for a transurethral resection of the
prostate (TURP). It would be inappropriate to include which of the following
points in the preoperative teaching?

A. TURP is the most common operation for BPH.


B. Explain the purpose and function of a two-way irrigation system.
C. Expect bloody urine, which will clear as healing takes place.
D. He will be pain free.

Answer: (D) He will be pain free.


Surgical interventions involve an experience of pain for the client which can
come in varying degrees. Telling the pain that he will be pain free is giving him
false reassurance.

55. Roxy is admitted to the hospital with a possible diagnosis of appendicitis. On


physical examination, the nurse should be looking for tenderness on palpation at
McBurney’s point, which is located in the
A. left lower quadrant
B. left upper quadrant
C. right lower quadrant
D. right upper quadrant

Answer: (C) right lower quadrant


To be exact, the appendix is anatomically located at the Mc Burney’s point at the
right iliac area of the right lower quadrant.

56. Mr. Valdez has undergone surgical repair of his inguinal hernia. Discharge
teaching should include
A. telling him to avoid heavy lifting for 4 to 6 weeks
B. instructing him to have a soft bland diet for two weeks
C. telling him to resume his previous daily activities without limitations
D. recommending him to drink eight glasses of water daily

Answer: (A) telling him to avoid heavy lifting for 4 to 6 weeks


The client should avoid lifting heavy objects and any strenuous activity for 4-6
weeks after surgery to prevent stress on the inguinal area. There is no special
diet required. The fluid intake of eight glasses a day is good advice but is not a
priority in this case.

57. A 30-year-old homemaker fell asleep while smoking a cigarette. She


sustained severe burns of the face,neck, anterior chest, and both arms and
hands. Using the rule of nines, which is the best estimate of total body-surface
area burned?

A. 18%
B. 22%
C. 31%
D. 40%

Answer: (C) 31%


Using the Rule of Nine in the estimation of total body surface burned, we allot
the following: 9% - head; 9% - each upper extremity; 18%- front chest and
abdomen; 18% - entire back; 18% - each lower extremity and 1% - perineum.

58. Nursing care planning is based on the knowledge that the first 24-48 hours
post-burn are characterized by:

A. An increase in the total volume of intracranial plasma


B. Excessive renal perfusion with diuresis
C. Fluid shift from interstitial space
D. Fluid shift from intravascular space to the interstitial space

Answer: (D) Fluid shift from intravascular space to the interstitial space
This period is the burn shock stage or the hypovolemic phase. Tissue injury
causes vasodilation that results in increase capillary permeability making fluids
shift from the intravascular to the interstitial space. This can lead to a decrease
in circulating blood volume or hypovolemia which decreases renal perfusion and
urine output.

59. If a client has severe bums on the upper torso, which item would be a
primary concern?
A. Debriding and covering the wounds
B. Administering antibiotics
C. Frequently observing for hoarseness, stridor, and dyspnea
D. Establishing a patent IV line for fluid replacement

Answer: (C) Frequently observing for hoarseness, stridor, and dyspnea


Burns located in the upper torso, especially resulting from thermal injury related
to fires can lead to inhalation burns. This causes swelling of the respiratory
mucosa and blistering which can lead to airway obstruction manifested by
hoarseness, noisy and difficult breathing. Maintaining a patent airway is a
primary concern.

60. Contractures are among the most serious long-term complications of severe
burns. If a burn is located on the upper torso, which nursing measure would be
least effective to help prevent contractures?
A. Changing the location of the bed or the TV set, or both, daily
B. Encouraging the client to chew gum and blow up balloons
C. Avoiding the use of a pillow for sleep, or placing the head in a position of
hyperextension
D. Helping the client to rest in the position of maximal comfort

Answer: (D) Helping the client to rest in the position of maximal comfort
Mobility and placing the burned areas in their functional position can help
prevent contracture deformities related to burns. Pain can immobilize a client as
he seeks the position where he finds less pain and provides maximal comfort.
But this approach can lead to contracture deformities and other complications.

61. An adult is receiving Total Parenteral Nutrition (TPN). Which of the following
assessment is essential?
A. evaluation of the peripheral IV site
B. confirmation that the tube is in the stomach
C. assess the bowel sound
D. fluid and electrolyte monitoring

Answer: (D) fluid and electrolyte monitoring


Total parenteral nutrition is a method of providing nutrients to the body by an IV
route. The admixture is made up of proteins, carbohydrates, fats, electrolytes,
vitamins, trace minerals and sterile water based on individual client needs. It is
intended to improve the clients nutritional status. Because of its composition, it
is important to monitor the clients fluid intake and output including electrolytes,
blood glucose and weight.

62. Which drug would be least effective in lowering a client's serum potassium
level?
A. Glucose and insulin
B. Polystyrene sulfonate (Kayexalate)
C. Calcium glucomite
D. Aluminum hydroxide

Answer: (D) Aluminum hydroxide


Aluminum hydroxide binds dietary phosphorus in the GI tract and helps treat
hyperphosphatemia. All the other medications mentioned help treat
hyperkalemia and its effects.

63. A nurse is directed to administer a hypotonic intravenous solution. Looking at


the following labeled solutions, she should choose
A. 0.45% NaCl
B. 0.9% NaCl
C. D5W
D. D5NSS

Answer: (A) 0.45% NaCl


Hypotonic solutions like 0.45% NaCl has a lower tonicity that the blood; 0.9%
NaCl and D5W are isotonic solutions with same tonicity as the blood; and D5NSS
is hypertonic with a higher tonicity thab the blood.

64. A patient is hemorrhaging from multiple trauma sites. The nurse expects that
compensatory mechanisms associated with hypovolemia would cause all of the
following symptoms EXCEPT
A. hypertension
B. oliguria
C. tachycardia
D. tachypnea

Answer: (A) hypertension


In hypovolemia, one of the compenasatory mechanisms is activation of the
sympathetic nervous system that increases the RR & PR and helps restore the BP
to maintain tissue perfusion but not cause a hypertension. The SNS stimulation
constricts renal arterioles that increases release of aldosterone, decreases
glomerular filtration and increases sodium & water reabsorption that leads to
oliguria.

65. Maria Sison, 40 years old, single, was admitted to the hospital with a
diagnosis of Breast Cancer. She was scheduled for radical mastectomy. Nursing
care during the preoperative period should consist of

A. assuring Maria that she will be cured of cancer


B. assessing Maria's expectations and doubts
C. maintaining a cheerful and optimistic environment
D. keeping Maria's visitors to a minimum so she can have time for herself

Answer: (B) assessing Maria's expectations and doubts


Assessing the client’s expectations and doubts will help lessen her fears and
anxieties. The nurse needs to encourage the client to verbalize and to listen and
correctly provide explanations when needed.

66. Maria refuses to acknowledge that her breast was removed. She believes
that her breast is intact under the dressing. The nurse should

A. call the MD to change the dressing so Kathy can see the incision
B. recognize that Kathy is experiencing denial, a normal stage of the grieving
process
C. reinforce Kathy’s belief for several days until her body can adjust to stress of
surgery.
D. remind Kathy that she needs to accept her diagnosis so that she can begin
rehabilitation exercises.

Answer: (B) recognize that Kathy is experiencing denial, a normal stage of the
grieving process
A person grieves to a loss of a significant object. The initial stage in the grieving
process is denial, then anger, followed by bargaining, depression and last
acceptance. The nurse should show acceptance of the patient’s feelings and
encourage verbalization.

67. A chemotherapeutic agent 5FU is ordered as an adjunct measure to surgery.


Which of the ff. statements about chemotherapy is true?
A. it is a local treatment affecting only tumor cells
B. it affects both normal and tumor cells
C. it has been proven as a complete cure for cancer
D. it is often used as a palliative measure.

Answer: (B) it affects both normal and tumor cells


Chemotherapeutic agents are given to destroy the actively proliferating cancer
cells. But these agents cannot differentiate the abnormal actively proliferating
cancer cells from those that are actively proliferating normal cells like the cells of
the bone marrow, thus the effect of bone marrow depression.

68. Which is an incorrect statement pertaining to the following procedures for


cancer diagnostics?
A. Biopsy is the removal of suspicious tissue and the only definitive method to
diagnose cancer
B. Ultrasonography detects tissue density changes difficult to observe by X-ray
via sound waves.
C. CT scanning uses magnetic fields and radio frequencies to provide cross-
sectional view of tumor
D. Endoscopy provides direct view of a body cavity to detect abnormality.

Answer: (C) CT scanning uses magnetic fields and radio frequencies to provide
cross-sectional view of tumor
CT scan uses narrow beam x-ray to provide cross-sectional view. MRI uses
magnetic fields and radio frequencies to detect tumors.
69. A post-operative complication of mastectomy is lymphedema. This can be
prevented by

A. ensuring patency of wound drainage tube


B. placing the arm on the affected side in a dependent position
C. restricting movement of the affected arm
D. frequently elevating the arm of the affected side above the level of the heart.

Answer: (D) frequently elevating the arm of the affected side above the level of
the heart.
Elevating the arm above the level of the heart promotes good venous return to
the heart and good lymphatic drainage thus preventing swelling.

70. Which statement by the client indicates to the nurse that the patient
understands precautions necessary during internal radiation therapy for cancer
of the cervix?

A. “I should get out of bed and walk around in my room.”


B. “My 7 year old twins should not come to visit me while I’m receiving
treatment.”
C. “I will try not to cough, because the force might make me expel the
application.”
D. “I know that my primary nurse has to wear one of those badges like the
people in the x-ray department, but they are not necessary for anyone else who
comes in here.”

Answer: (B) “My 7 year old twins should not come to visit me while I’m receiving
treatment.”
Children have cells that are normally actively dividing in the process of growth.
Radiation acts not only against the abnormally actively dividing cells of cancer
but also on the normally dividing cells thus affecting the growth and
development of the child and even causing cancer itself.

71. High uric acid levels may develop in clients who are receiving chemotherapy.
This is caused by:

A. The inability of the kidneys to excrete the drug metabolites


B. Rapid cell catabolism
C. Toxic effect of the antibiotic that are given concurrently
D. The altered blood ph from the acid medium of the drugs
Answer: (B) Rapid cell catabolism
One of the oncologic emergencies, the tumor lysis syndrome, is caused by the
rapid destruction of large number of tumor cells. . Intracellular contents are
released, including potassium and purines, into the bloodstream faster than the
body can eliminate them. The purines are converted in the liver to uric acid and
released into the blood causing hyperuricemia. They can precipitate in the
kidneys and block the tubules causing acute renal failure.

72. Which of the following interventions would be included in the care of plan in
a client with cervical
implant?

A. Frequent ambulation
B. Unlimited visitors
C. Low residue diet
D. Vaginal irrigation every shift

Answer: (C) Low residue diet


It is important for the nurse to remember that the implant be kept intact in the
cervix during therapy. Mobility and vaginal irrigations are not done. A low
residue diet will prevent bowel movement that could lead to dislodgement of the
implant. Patient is also strictly isolated to protect other people from the radiation
emissions

73. Which nursing measure would avoid constriction on the affected arm
immediately after mastectomy?

A. Avoid BP measurement and constricting clothing on the affected arm


B. Active range of motion exercises of the arms once a day.
C. Discourage feeding, washing or combing with the affected arm
D. Place the affected arm in a dependent position, below the level of the heart

Answer: (A) Avoid BP measurement and constricting clothing on the affected arm
A BP cuff constricts the blood vessels where it is applied. BP measurements
should be done on the unaffected arm to ensure adequate circulation and venous
and lymph drainage in the affected arm

74. A client suffering from acute renal failure has an unexpected increase in
urinary output to 150ml/hr. The nurse assesses that the client has entered the
second phase of acute renal failure. Nursing actions throughout this phase
include observation for signs and symptoms of

A. Hypervolemia, hypokalemia, and hypernatremia.


B. Hypervolemia, hyperkalemia, and hypernatremia.
C. Hypovolemia, wide fluctuations in serum sodium and potassium levels.
D. Hypovolemia, no fluctuation in serum sodium and potassium levels.

Answer: (C) Hypovolemia, wide fluctuations in serum sodium and potassium


levels.
The second phase of ARF is the diuretic phase or high output phase. The diuresis
can result in an output of up to 10L/day of dilute urine. Loss of fluids and
electrolytes occur.
75. An adult has just been brought in by ambulance after a motor vehicle
accident. When assessing the client, the nurse would expect which of the
following manifestations could have resulted from sympathetic nervous system
stimulation?

A. A rapid pulse and increased RR


B. Decreased physiologic functioning
C. Rigid posture and altered perceptual focus
D. Increased awareness and attention

Answer: (A) A rapid pulse and increased RR


The fight or flight reaction of the sympathetic nervous system occurs during
stress like in a motor vehicular accident. This is manifested by increased in
cardiovascular function and RR to provide the immediate needs of the body for
survival.

76. Ms. Sy undergoes surgery and the abdominal aortic aneurysm is resected
and replaced with a graft. When she arrives in the RR she is still in shock. The
nurse's priority should be

A. placing her in a trendeleburg position


B. putting several warm blankets on her
C. monitoring her hourly urine output
D. assessing her VS especially her RR

Answer: (D) assessing her VS especially her RR


Shock is characterized by reduced tissue and organ perfusion and eventual organ
dysfunction and failure. Checking on the VS especially the RR, which detects
need for oxygenation, is a priority to help detect its progress and provide for
prompt management before the occurrence of complications.
77. A major goal for the client during the first 48 hours after a severe bum is to
prevent hypovolemic shock. The best indicator of adequate fluid balance during
this period is

A. Elevated hematocrit levels.


B. Urine output of 30 to 50 ml/hr.
C. Change in level of consciousness.
D. Estimate of fluid loss through the burn eschar.

Answer: (B) Urine output of 30 to 50 ml/hr.


Hypovolemia is a decreased in circulatory volume. This causes a decrease in
tissue perfusion to the different organs of the body. Measuring the hourly urine
output is the most quantifiable way of measuring tissue perfusion to the organs.
Normal renal perfusion should produce 1ml/kg of BW/min. An output of 30-50
ml/hr is considered adequate and indicates good fluid balance.

78. A thoracentesis is performed on a chest-injured client, and no fluid or air is


found. Blood and fluids is administered intravenously (IV), but the client's vital
signs do not improve. A central venous pressure line is inserted, and the initial
reading is 20 cm H^O. The most likely cause of these findings is which of the
following?

A. Spontaneous pneumothorax
B. Ruptured diaphragm
C. Hemothorax
D. Pericardial tamponade

Answer: (D) Pericardial tamponade


Pericardial tamponade occurs when there is presence of fluid accumulation in the
pericardial space that compresses on the ventricles causing a decrease in
ventricular filling and stretching during diastole with a decrease in cardiac
output. . This leads to right atrial and venous congestion manifested by a CVP
reading above normal.

79. Intervention for a pt. who has swallowed a Muriatic Acid includes all of the
following except
A. administering an irritant that will stimulate vomiting
B. aspirating secretions from the pharynx if respirations are affected
C. neutralizing the chemical
D. washing the esophagus with large volumes of water via gastric lavage
Answer: (A) administering an irritant that will stimulate vomiting
Swallowing of corrosive substances causes severe irritation and tissue
destruction of the mucous membrane of the GI tract. Measures are taken to
immediately remove the toxin or reduce its absorption. For corrosive poison
ingestion, such as in muriatic acid where burn or perforation of the mucosa may
occur, gastric emptying procedure is immediately instituted, This includes gastric
lavage and the administration of activated charcoal to absorb the poison.
Administering an irritant with the concomitant vomiting to remove the swallowed
poison will further cause irritation and damage to the mucosal lining of the
digestive tract. Vomiting is only indicated when non-corrosive poison is
swallowed.

80. Which initial nursing assessment finding would best indicate that a client has
been successfully resuscitated after a cardio-respiratory arrest?
A. Skin warm and dry
B. Pupils equal and react to light
C. Palpable carotid pulse
D. Positive Babinski's reflex

Answer: (C) Palpable carotid pulse


Presence of a palpable carotid pulse indicates the return of cardiac function
which, together with the return of breathing, is the primary goal of CPR.
Pulsations in arteries indicates blood flowing in the blood vessels with each
cardiac contraction. Signs of effective tissue perfusion will be noted after.

81. Chemical burn of the eye are treated with


A. local anesthetics and antibacterial drops for 24 – 36 hrs.
B. hot compresses applied at 15-minute intervals
C. Flushing of the lids, conjunctiva and cornea with tap or preferably sterile
water
D. cleansing the conjunctiva with a small cotton-tipped applicator

Answer: (C) Flushing of the lids, conjunctiva and cornea with tap or preferably
sterile water
Prompt treatment of ocular chemical burns is important to prevent further
damage. Immediate tap-water eye irrigation should be started on site even
before transporting the patient to the nearest hospital facility. In the hospital,
copious irrigation with normal saline, instillation of local anesthetic and antibiotic
is done.

82. The Heimlich maneuver (abdominal thrust), for acute airway obstruction,
attempts to:
A. Force air out of the lungs
B. Increase systemic circulation
C. Induce emptying of the stomach
D. Put pressure on the apex of the heart

Answer: (A) Force air out of the lungs


The Heimlich maneuver is used to assist a person choking on a foreign object.
The pressure from the thrusts lifts the diaphragm, forces air out of the lungs and
creates an artificial cough that expels the aspirated material.

83. John, 16 years old, is brought to the ER after a vehicular accident. He is


pronounced dead on arrival. When his parents arrive at the hospital, the nurse
should:
A. ask them to stay in the waiting area until she can spend time alone with them
B. speak to both parents together and encourage them to support each other
and express their emotions freely
C. Speak to one parent at a time so that each can ventilate feelings of loss
without upsetting the other
D. ask the MD to medicate the parents so they can stay calm to deal with their
son’s death.

Answer: (B) speak to both parents together and encourage them to support each
other and express their emotions freely
Sudden death of a family member creates a state of shock on the family. They
go into a stage of denial and anger in their grieving. Assisting them with
information they need to know, answering their questions and listening to them
will provide the needed support for them to move on and be of support to one
another.

84. An emergency treatment for an acute asthmatic attack is Adrenaline 1:1000


given hypodermically. This is given to:

A. increase BP
B. decrease mucosal swelling
C. relax the bronchial smooth muscle
D. decrease bronchial secretions

Answer: (C) relax the bronchial smooth muscle


Acute asthmatic attack is characterized by severe bronchospasm which can be
relieved by the immediate administration of bronchodilators. Adrenaline or
Epinephrine is an adrenergic agent that causes bronchial dilation by relaxing the
bronchial smooth muscles.

85. A nurse is performing CPR on an adult patient. When performing chest


compressions, the nurse understands the correct hand placement is located over
the
A. upper half of the sternum
B. upper third of the sternum
C. lower half of the sternum
D. lower third of the sternum

Answer: (C) lower half of the sternum


The exact and safe location to do cardiac compression is the lower half of the
sternum. Doing it at the lower third of the sternum may cause gastric
compression which can lead to a possible aspiration.

86. The nurse is performing an eye examination on an elderly client. The client
states ‘My vision is blurred, and I don’t easily see clearly when I get into a dark
room.” The nurse best response is:

A. “You should be grateful you are not blind.”


B. “As one ages, visual changes are noted as part of degenerative changes. This
is normal.”
C. “You should rest your eyes frequently.”
D. “You maybe able to improve you vision if you move slowly.”

Answer: (B) “As one ages, visual changes are noted as part of degenerative
changes. This is normal.”
Aging causes less elasticity of the lens affecting accommodation leading to
blurred vision. The muscles of the iris increase in stiffness and the pupils dilate
slowly and less completely so that it takes the older person to adjust when going
to and from light and dark environment and needs brighter light for close vision.

87. Which of the following activities is not encouraged in a patient after an eye
surgery?
A. sneezing, coughing and blowing the nose
B. straining to have a bowel movement
C. wearing tight shirt collars
D. sexual intercourse

Answer: (D) sexual intercourse


To reduce increases in IOP, teach the client and family about activity restrictions.
Sexual intercourse can cause a sudden rise in IOP.

88. Which of the following indicates poor practice in communicating with a


hearing-impaired client?
A. Use appropriate hand motions
B. Keep hands and other objects away from your mouth when talking to the
client
C. Speak clearly in a loud voice or shout to be heard
D. Converse in a quiet room with minimal distractions

Answer: (C) Speak clearly in a loud voice or shout to be heard


Shouting raises the frequency of the sound and often makes understanding the
spoken words difficult. It is enough for the nurse to speak clearly and slowly.

89. A client is to undergo lumbar puncture. Which is least important information


about LP?

A. Specimens obtained should be labeled in their proper sequence.


B. It may be used to inject air, dye or drugs into the spinal canal.
C. Assess movements and sensation in the lower extremities after the
D. Force fluids before and after the procedure.

Answer: (D) Force fluids before and after the procedure.


LP involves the removal of some amount of spinal fluid. To facilitate CSF
production, the client is instructed to increase fluid intake to 3L, unless
contraindicated, for 24 to 48 hrs after the procedure.

90. A client diagnosed with cerebral thrombosis is scheduled for cerebral


angiography. Nursing care of the client includes the following EXCEPT

A. Inform the client that a warm, flushed feeling and a salty taste may be
B. Maintain pressure dressing over the site of puncture and check for
C. Check pulse, color and temperature of the extremity distal to the site of
D. Kept the extremity used as puncture site flexed to prevent bleeding.

Answer: (D) Kept the extremity used as puncture site flexed to prevent bleeding.
Angiography involves the threading of a catheter through an artery which can
cause trauma to the endothelial lining of the blood vessel. The platelets are
attracted to the area causing thrombi formation. This is further enhanced by the
slowing of blood flow caused by flexion of the affected extremity. The affected
extremity must be kept straight and immobilized during the duration of the
bedrest after the procedure. Ice bag can be applied intermittently to the
puncture site.

91. Which is considered as the earliest sign of increased ICP that the nurse
should closely observed for?

A. abnormal respiratory pattern


B. rising systolic and widening pulse pressure
C. contralateral hemiparesis and ipsilateral dilation of the pupils
D. progression from restlessness to confusion and disorientation to lethargy

Answer: (D) progression from restlessness to confusion and disorientation to


lethargy
The first major effect of increasing ICP is a decrease in cerebral perfusion
causing hypoxia that produces a progressive alteration in the LOC. This is initially
manifested by restlessness.

92. Which is irrelevant in the pharmacologic management of a client with CVA?

A. Osmotic diuretics and corticosteroids are given to decrease cerebral edema


B. Anticonvulsants are given to prevent seizures
C. Thrombolytics are most useful within three hours of an occlusive CVA
D. Aspirin is used in the acute management of a completed stroke.

Answer: (D) Aspirin is used in the acute management of a completed stroke.


The primary goal in the management of CVA is to improve cerebral tissue
perfusion. Aspirin is a platelet deaggregator used in the prevention of recurrent
or embolic stroke but is not used in the acute management of a completed
stroke as it may lead to bleeding.

93. What would be the MOST therapeutic nursing action when a client’s
expressive aphasia is severe?

A. Anticipate the client wishes so she will not need to talk


B. Communicate by means of questions that can be answered by the client
shaking the head
C. Keep us a steady flow rank to minimize silence
D. Encourage the client to speak at every possible opportunity.

Answer: (D) Encourage the client to speak at every possible opportunity.


Expressive or motor aphasia is a result of damage in the Broca’s area of the
frontal lobe. It is amotor speech problem in which the client generally
understands what is said but is unable to communicate verbally. The patient can
best he helped therefore by encouraging him to communicate and reinforce this
behavior positively.

94. A client with head injury is confused, drowsy and has unequal pupils. Which
of the following nursing diagnosis is most important at this time?

A. altered level of cognitive function


B. high risk for injury
C. altered cerebral tissue perfusion
D. sensory perceptual alteration

Answer: (C) altered cerebral tissue perfusion


The observations made by the nurse clearly indicate a problem of decrease
cerebral perfusion. Restoring cerebral perfusion is most important to maintain
cerebral functioning and prevent further brain damage.

95. Which nursing diagnosis is of the highest priority when caring for a client
with myasthenia gravis?
A. Pain
B. High risk for injury related to muscle weakness
C. Ineffective coping related to illness
D. Ineffective airway clearance related to muscle weakness

Answer: (D) Ineffective airway clearance related to muscle weakness


Myasthenia gravis causes a failure in the transmission of nerve impulses at the
neuromuscular junction which may be due to a weakening or decrease in
acetylcholine receptor sites. This leads to sporadic, progressive weakness or
abnormal fatigability of striated muscles that eventually causes loss of function.
The respiratory muscles can become weak with decreased tidal volume and vital
capacity making breathing and clearing the airway through coughing difficult.
The respiratory muscle weakness may be severe enough to require and
emergency airway and mechanical ventilation.

96. The client has clear drainage from the nose and ears after a head injury.
How can the nurse determine if the drainage is CSF?

A. Measure the ph of the fluid


B. Measure the specific gravity of the fluid
C. Test for glucose
D. Test for chlorides

Answer: (C) Test for glucose


The CSF contains a large amount of glucose which can be detected by using
glucostix. A positive result with the drainage indicate CSF leakage.

97. The nurse includes the important measures for stump care in the teaching
plan for a client with an amputation. Which measure would be excluded from the
teaching plan?
A. Wash, dry, and inspect the stump daily.
B. Treat superficial abrasions and blisters promptly.
C. Apply a "shrinker" bandage with tighter arms around the proximal end of the
affected limb.
D. Toughen the stump by pushing it against a progressively harder substance
(e.g., pillow on a foot-stool).

Answer: (C) Apply a "shrinker" bandage with tighter arms around the proximal
end of the affected limb.
The “shrinker” bandage is applied to prevent swelling of the stump. It should be
applied with the distal end with the tighter arms. Applying the tighter arms at
the proximal end will impair circulation and cause swelling by reducing venous
flow.

98. A 70-year-old female comes to the clinic for a routine checkup. She is 5 feet
4 inches tall and weighs 180 pounds. Her major complaint is pain in her joints.
She is retired and has had to give up her volunteer work because of her
discomfort. She was told her diagnosis was osteoarthritis about 5 years ago.
Which would be excluded from the clinical pathway for this client?

A. Decrease the calorie count of her daily diet.


B. Take warm baths when arising.
C. Slide items across the floor rather than lift them.
D. Place items so that it is necessary to bend or stretch to reach them.

Answer: (D) Place items so that it is necessary to bend or stretch to reach them.
Patients with osteoarthritis have decreased mobility caused by joint pain. Over-
reaching and stretching to get an object are to be avoided as this can cause
more pain and can even lead to falls. The nurse should see to it therefore that
objects are within easy reach of the patient.

99. A client is admitted from the emergency department with severe-pain and
edema in the right foot. His diagnosis is gouty arthritis. When developing a plan
of care, which action would have the highest priority?

A. Apply hot compresses to the affected joints.


B. Stress the importance of maintaining good posture to prevent deformities.
C. Administer salicylates to minimize the inflammatory reaction.
D. Ensure an intake of at least 3000 ml of fluid per day.

Answer: (D) Ensure an intake of at least 3000 ml of fluid per day.


Gouty arthritis is a metabolic disease marked by urate deposits that cause
painful arthritic joints. The patient should be urged to increase his fluid intake to
prevent the development of urinary uric acid stones.

100. A client had a laminectomy and spinal fusion yesterday. Which statement is
to be excluded from your plan of care?

A. Before log rolling, place a pillow under the client's head and a pillow between
the client's legs.
B. Before log rolling, remove the pillow from under the client's head and use no
pillows between the client's legs.
C. Keep the knees slightly flexed while the client is lying in a semi-Fowler's
position in bed.
D. Keep a pillow under the client's head as needed for comfort.

Answer: (B) Before log rolling, remove the pillow from under the client's head
and use no pillows between the client's legs.
Following a laminectomy and spinal fusion, it is important that the back of the
patient be maintained in straight alignment and to support the entire vertebral
column to promote complete healing.

101. The nurse is assisting in planning care for a client with a diagnosis of
immune deficiency. The nurse would incorporate which of the ff. as a priority in
the plan of care?

A. providing emotional support to decrease fear


B. protecting the client from infection
C. encouraging discussion about lifestyle changes
D. identifying factors that decreased the immune function

Answer: (B) protecting the client from infection


Immunodeficiency is an absent or depressed immune response that increases
susceptibility to infection. So it is the nurse’s primary responsibility to protect the
patient from infection.

102. Joy, an obese 32 year old, is admitted to the hospital after an automobile
accident. She has a fractured hip and is brought to the OR for surgery.

After surgery Joy is to receive a piggy-back of Clindamycin phosphate (Cleocin)


300 mg in 50 ml of D5W. The piggyback is to infuse in 20 minutes. The drop
factor of the IV set is 10 gtt/ml. The nurse should set the piggyback to flow at:

A. 25 gtt/min
B. 30 gtt/min
C. 35 gtt/min
D. 45 gtt/min

Answer: (A) 25 gtt/min


To get the correct flow rate: multiply the amount to be infused (50 ml) by the
drop factor (10) and divide the result by the amount of time in minutes (20)

103. The day after her surgery Joy asks the nurse how she might lose weight.
Before answering her question, the nurse should bear in mind that long-term
weight loss best occurs when:

A. Fats are controlled in the diet


B. Eating habits are altered
C. Carbohydrates are regulated
D. Exercise is part of the program

Answer: (B) Eating habits are altered


For weight reduction to occur and be maintained, a new dietary program, with a
balance of foods from the basic four food groups, must be established and
continued

104. The nurse teaches Joy, an obese client, the value of aerobic exercises in her
weight reduction program. The nurse would know that this teaching was
effective when Joy says that exercise will:

A. Increase her lean body mass


B. Lower her metabolic rate
C. Decrease her appetite
D. Raise her heart rate
Answer: (A) Increase her lean body mass
Increased exercise builds skeletal muscle mass and reduces excess fatty tissue.

105. The physician orders non-weight bearing with crutches for Joy, who had
surgery for a fractured hip. The most important activity to facilitate walking with
crutches before ambulation begun is:

A. Exercising the triceps, finger flexors, and elbow extensors


B. Sitting up at the edge of the bed to help strengthen back muscles
C. Doing isometric exercises on the unaffected leg
D. Using the trapeze frequently for pull-ups to strengthen the biceps muscles

Answer: (A) Exercising the triceps, finger flexors, and elbow extensors
These sets of muscles are used when walking with crutches and therefore need
strengthening prior to ambulation.

106. The nurse recognizes that a client understood the demonstration of crutch
walking when she places her weight on:

A. The palms and axillary regions


B. Both feet placed wide apart
C. The palms of her hands
D. Her axillary regions

Answer: (C) The palms of her hands


The palms should bear the client’s weight to avoid damage to the nerves in the
axilla (brachial plexus)

107. Joey is a 46 year-old radio technician who is admitted because of mild chest
pain. He is 5 feet, 8 inches tall and weighs 190 pounds. He is diagnosed with a
myocardial infarct. Morphine sulfate, Diazepam (Valium) and Lidocaine are
prescribed.

The physician orders 8 mg of Morphine Sulfate to be given IV. The vial on hand
is labeled 1 ml/ 10 mg. The nurse should administer:

A. 8 minims
B. 10 minims
C. 12 minims
D. 15 minims
Answer: (C) 12 minims
Using ratio and proportion 8 mg/10 mg = X minims/15 minims 10 X= 120 X =
12 minims The nurse will administer 12 minims intravenously equivalent to 8mg
Morphine Sulfate

108. Joey asks the nurse why he is receiving the injection of Morphine after he
was hospitalized for severe anginal pain. The nurse replies that it:

A. Will help prevent erratic heart beats


B. Relieves pain and decreases level of anxiety
C. Decreases anxiety
D. Dilates coronary blood vessels

Answer: (B) Relieves pain and decreases level of anxiety


Morphine is a specific central nervous system depressant used to relieve the pain
associated with myocardial infarction. It also decreases anxiety and
apprehension and prevents cardiogenic shock by decreasing myocardial oxygen
demand.

109. Oxygen 3L/min by nasal cannula is prescribed for Joey who is admitted to
the hospital for chest pain. The nurse institutes safety precautions in the room
because oxygen:

A. Converts to an alternate form of matter


B. Has unstable properties
C. Supports combustion
D. Is flammable

Answer: (C) Supports combustion


The nurse should know that Oxygen is necessary to produce fire, thus
precautionary measures are important regarding its use.

110. Myra is ordered laboratory tests after she is admitted to the hospital for
angina. The isoenzyme test that is the most reliable early indicator of myocardial
insult is:

A. SGPT
B. LDH
C. CK-MB
D. AST
Answer: (C) CK-MB
The cardiac marker, Creatinine phosphokinase (CPK) isoenzyme levels, especially
the MB sub-unit which is cardio-specific, begin to rise in 3-6 hours, peak in 12-
18 hours and are elevated 48 hours after the occurrence of the infarct. They are
therefore most reliable in assisting with early diagnosis. The cardiac markers
elevate as a result of myocardial tissue damage.

111. An early finding in the EKG of a client with an infarcted mycardium would
be:

A. Disappearance of Q waves
B. Elevated ST segments
C. Absence of P wave
D. Flattened T waves

Answer: (B) Elevated ST segments


This is a typical early finding after a myocardial infarct because of the altered
contractility of the heart. The other choices are not typical of MI.

112. Jose, who had a myocardial infarction 2 days earlier, has been complaining
to the nurse about issues related to his hospital stay. The best initial nursing
response would be to:

A. Allow him to release his feelings and then leave him alone to allow him to
regain his composure
B. Refocus the conversation on his fears, frustrations and anger about his
condition
C. Explain how his being upset dangerously disturbs his need for rest
D. Attempt to explain the purpose of different hospital routines

Answer: (B) Refocus the conversation on his fears, frustrations and anger about
his condition
This provides the opportunity for the client to verbalize feelings underlying
behavior and helpful in relieving anxiety. Anxiety can be a stressor which can
activate the sympathoadrenal response causing the release of catecholamines
that can increase cardiac contractility and workload that can further increase
myocardial oxygen demand.

113. Twenty four hours after admission for an Acute MI, Jose’s temperature is
noted at 39.3 C. The nurse monitors him for other adaptations related to the
pyrexia, including:
A. Shortness of breath
B. Chest pain
C. Elevated blood pressure
D. Increased pulse rate

Answer: (D) Increased pulse rate


Fever causes an increase in the body’s metabolism, which results in an increase
in oxygen consumption and demand. This need for oxygen increases the heart
rate, which is reflected in the increased pulse rate. Increased BP, chest pain and
shortness of breath are not typically noted in fever.

114. Jose, who is admitted to the hospital for chest pain, asks the nurse, “Is it
still possible for me to have another heart attack if I watch my diet religiously
and avoid stress?” The most appropriate initial response would be for the nurse
to:

A. Suggest he discuss his feelings of vulnerability with his physician.


B. Tell him that he certainly needs to be especially careful about his diet and
lifestyle.
C. Avoid giving him direct information and help him explore his feelings
D. Recognize that he is frightened and suggest he talk with the psychiatrist or
counselor.

Answer: (C) Avoid giving him direct information and help him explore his feelings
To help the patient verbalize and explore his feelings, the nurse must reflect and
analyze the feelings that are implied in the client’s question. The focus should be
on collecting data to minister to the client’s psychosocial needs.

115. Ana, 55 years old, is admitted to the hospital to rule out pernicious anemia.
A Schilling test is ordered for Ana. The nurse recognizes that the primary
purpose of the Schilling test is to determine the client’s ability to:

A. Store vitamin B12


B. Digest vitamin B12
C. Absorb vitamin B12
D. Produce vitamin B12

Answer: (C) Absorb vitamin B12


Pernicious anemia is caused by the inability to absorb vitamin B12 in the
stomach due to a lack of intrinsic factor in the gastric juices. In the Schilling test,
radioactive vitamin B12 is administered and its absorption and excretion can be
ascertained through the urine.

116. Ana is diagnosed to have Pernicious anemia. The physician orders 0.2 mg of
Cyanocobalamin (Vitamin B12) IM. Available is a vial of the drug labeled 1 ml=
100 mcg. The nurse should administer:

A. 0.5 ml
B. 1.0 ml
C. 1.5 ml
D. 2.0 ml

Answer: (D) 2.0 ml


First convert milligrams to micrograms and then use ratio and proportion (0.2
mg= 200 mcg) 200 mcg : 100 mcg= X ml : ml 100 X= 200 X = 2 ml. Inject 2
ml. to give 0.2 mg of Cyanocobalamin.

117. Health teachings to be given to a client with Pernicious Anemia regarding


her therapeutic regimen concerning Vit. B12 will include:

A. Oral tablets of Vitamin B12 will control her symptoms


B. IM injections are required for daily control
C. IM injections once a month will maintain control
D. Weekly Z-track injections provide needed control

Answer: (C) IM injections once a month will maintain control


Deep IM injections bypass B12 absorption defect in the stomach due to lack of
intrinsic factor, the transport carrier component of gastric juices. A monthly dose
is usually sufficient since it is stored in active body tissues such as the liver,
kidney, heart, muscles, blood and bone marrow

118. The nurse knows that a client with Pernicious Anemia understands the
teaching regarding the vitamin B12 injections when she states that she must
take it:

A. When she feels fatigued


B. During exacerbations of anemia
C. Until her symptoms subside
D. For the rest of her life

Answer: (D) For the rest of her life


Since the intrinsic factor does not return to gastric secretions even with therapy,
B12 injections will be required for the remainder of the client’s life.

119. Arthur Cruz, a 45 year old artist, has recently had an abdominoperineal
resection and colostomy. Mr. Cruz accuses the nurse of being uncomfortable
during a dressing change, because his “wound looks terrible.” The nurse
recognizes that the client is using the defense mechanism known as:

A. Reaction Formation
B. Sublimation
C. Intellectualization
D. Projection

Answer: (D) Projection


Projection is the attribution of unacceptable feelings and emotions to others
which may indicate the patients nonacceptance of his condition.

120. When preparing to teach a client with colostomy how to irrigate his
colostomy, the nurse should plan to perform the procedure:

A. When the client would have normally had a bowel movement


B. After the client accepts he had a bowel movement
C. Before breakfast and morning care
D. At least 2 hours before visitors arrive

Answer: (A) When the client would have normally had a bowel movement
Irrigation should be performed at the time the client normally defecated before
the colostomy to maintain continuity in lifestyle and usual bowel function/habit.

121. When observing an ostomate do a return demonstration of the colostomy


irrigation, the nurse notes that he needs more teaching if he:

A. Stops the flow of fluid when he feels uncomfortable


B. Lubricates the tip of the catheter before inserting it into the stoma
C. Hangs the bag on a clothes hook on the bathroom door during fluid insertion
D. Discontinues the insertion of fluid after only 500 ml of fluid has been instilled

Answer: (C) Hangs the bag on a clothes hook on the bathroom door during fluid
insertion
The irrigation bag should be hung 12-18 inches above the level of the stoma; a
clothes hook is too high which can create increase pressure and sudden intestinal
distention and cause abdominal discomfort to the patient.
122. When doing colostomy irrigation at home, a client with colostomy should be
instructed to report to his physician :

A. Abdominal cramps during fluid inflow


B. Difficulty in inserting the irrigating tube
C. Passage of flatus during expulsion of feces
D. Inability to complete the procedure in half an hour

Answer: (B) Difficulty in inserting the irrigating tube


Difficulty of inserting the irrigating tube indicates stenosis of the stoma and
should be reported to the physician. Abdominal cramps and passage of flatus can
be expected during colostomy irrigations. The procedure may take longer than
half an hour.

123. A client with colostomy refuses to allow his wife to see the incision or stoma
and ignores most of his dietary instructions. The nurse on assessing this data,
can assume that the client is experiencing:

A. A reaction formation to his recent altered body image.


B. A difficult time accepting reality and is in a state of denial.
C. Impotency due to the surgery and needs sexual counseling
D. Suicide thoughts and should be seen by psychiatrist

Answer: (B) A difficult time accepting reality and is in a state of denial.


As long as no one else confirms the presence of the stoma and the client does
not need to adhere to a prescribed regimen, the client’s denial is supported

124. The nurse would know that dietary teaching had been effective for a client
with colostomy when he states that he will eat:

A. Food low in fiber so that there is less stool


B. Everything he ate before the operation but will avoid those foods that cause
gas
C. Bland foods so that his intestines do not become irritated
D. Soft foods that are more easily digested and absorbed by the large intestines

Answer: (B) Everything he ate before the operation but will avoid those foods
that cause gas
There is no special diets for clients with colostomy. These clients can eat a
regular diet. Only gas-forming foods that cause distention and discomfort should
be avoided.

125. Eddie, 40 years old, is brought to the emergency room after the crash of his
private plane. He has suffered multiple crushing wounds of the chest, abdomen
and legs. It is feared his leg may have to be amputated.

When Eddie arrives in the emergency room, the assessment that assume the
greatest priority are:

A. Level of consciousness and pupil size


B. Abdominal contusions and other wounds
C. Pain, Respiratory rate and blood pressure
D. Quality of respirations and presence of pulsesQuality of respirations and
presence of pulses

Answer: (D) Quality of respirations and presence of pulsesQuality of respirations


and presence of pulses
Respiratory and cardiovascular functions are essential for oxygenation. These are
top priorities to trauma management. Basic life functions must be maintained or
reestablished

126. Eddie, a plane crash victim, undergoes endotracheal intubation and positive
pressure ventilation. The most immediate nursing intervention for him at this
time would be to:

A. Facilitate his verbal communication


B. Maintain sterility of the ventilation system
C. Assess his response to the equipment
D. Prepare him for emergency surgery

Answer: (C) Assess his response to the equipment


It is a primary nursing responsibility to evaluate effect of interventions done to
the client. Nothing is achieved if the equipment is working and the client is not
responding

127. A chest tube with water seal drainage is inserted to a client following a
multiple chest injury. A few hours later, the client’s chest tube seems to be
obstructed. The most appropriate nursing action would be to

A. Prepare for chest tube removal


B. Milk the tube toward the collection container as ordered
C. Arrange for a stat Chest x-ray film.
D. Clam the tube immediately

Answer: (B) Milk the tube toward the collection container as ordered
This assists in moving blood, fluid or air, which may be obstructing drainage,
toward the collection chamber

128. The observation that indicates a desired response to thoracostomy drainage


of a client with chest injury is:

A. Increased breath sounds


B. Constant bubbling in the drainage chamber
C. Crepitus detected on palpation of chest
D. Increased respiratory rate

Answer: (A) Increased breath sounds


The chest tube normalizes intrathoracic pressure and restores negative intra-
pleural pressure, drains fluid and air from the pleural space, and improves
pulmonary function

129. In the evaluation of a client’s response to fluid replacement therapy, the


observation that indicates adequate tissue perfusion to vital organs is:

A. Urinary output is 30 ml in an hour


B. Central venous pressure reading of 2 cm H2O
C. Pulse rates of 120 and 110 in a 15 minute period
D. Blood pressure readings of 50/30 and 70/40 within 30 minutes

Answer: (A) Urinary output is 30 ml in an hour


A rate of 30 ml/hr is considered adequate for perfusion of kidney, heart and
brain.

130. A client with multiple injury following a vehicular accident is transferred to


the critical care unit. He begins to complain of increased abdominal pain in the
left upper quadrant. A ruptured spleen is diagnosed and he is scheduled for
emergency splenectomy. In preparing the client for surgery, the nurse should
emphasize in his teaching plan the:

A. Complete safety of the procedure


B. Expectation of postoperative bleeding
C. Risk of the procedure with his other injuries
D. Presence of abdominal drains for several days after surgery

Answer: (D) Presence of abdominal drains for several days after surgery
Drains are usually inserted into the splenic bed to facilitate removal of fluid in
the area that could lead to abscess formation.

131. To promote continued improvement in the respiratory status of a client


following chest tube removal after a chest surgery for multiple rib fracture, the
nurse should:

A. Encourage bed rest with active and passive range of motion exercises
B. Encourage frequent coughing and deep breathing
C. Turn him from side to side at least every 2 hours
D. Continue observing for dyspnea and crepitus

Answer: (B) Encourage frequent coughing and deep breathing


This nursing action prevents atelectasis and collection of respiratory secretions
and promotes adequate ventilation and gas exchange.

132. A client undergoes below the knee amputation following a vehicular


accident. Three days postoperatively, the client is refusing to eat, talk or perform
any rehabilitative activities. The best initial nursing approach would be to:

A. Give him explanations of why there is a need to quickly increase his activity
B. Emphasize repeatedly that with as prosthesis, he will be able to return to his
normal lifestyle
C. Appear cheerful and non-critical regardless of his response to attempts at
intervention
D. Accept and acknowledge that his withdrawal is an initially normal and
necessary part of grieving

Answer: (D) Accept and acknowledge that his withdrawal is an initially normal
and necessary part of grieving
The withdrawal provides time for the client to assimilate what has occurred and
integrate the change in the body image. Acceptance of the client’s behavior is an
important factor in the nurse’s intervention.

133. The key factor in accurately assessing how body image changes will be
dealt with by the client is the:

A. Extent of body change present


B. Suddenness of the change
C. Obviousness of the change
D. Client’s perception of the change

Answer: (D) Client’s perception of the change


It is not reality, but the client’s feeling about the change that is the most
important determinant of the ability to cope. The client should be encouraged to
his feelings.

134. Larry is diagnosed as having myelocytic leukemia and is admitted to the


hospital for chemotherapy. Larry discusses his recent diagnosis of leukemia by
referring to statistical facts and figures. The nurse recognizes that Larry is using
the defense mechanism known as:

A. Reaction formation
B. Sublimation
C. Intellectualization
D. Projection

Answer: (C) Intellectualization


People use defense mechanisms to cope with stressful events. Intellectualization
is the use of reasoning and thought processes to avoid the emotional upsets.

135. The laboratory results of the client with leukemia indicate bone marrow
depression. The nurse should encourage the client to:

A. Increase his activity level and ambulate frequently


B. Sleep with the head of his bed slightly elevated
C. Drink citrus juices frequently for nourishment
D. Use a soft toothbrush and electric razor

Answer: (D) Use a soft toothbrush and electric razor


Suppression of red bone marrow increases bleeding susceptibility associated with
thrombocytopenia, decreased platelets. Anemia and leucopenia are the two other
problems noted with bone marrow depression.

136. Dennis receives a blood transfusion and develops flank pain, chills, fever
and hematuria. The nurse recognizes that Dennis is probably experiencing:

A. An anaphylactic transfusion reaction


B. An allergic transfusion reaction
C. A hemolytic transfusion reaction
D. A pyrogenic transfusion reaction

Answer: (C) A hemolytic transfusion reaction


This results from a recipient’s antibodies that are incompatible with transfused
RBC’s; also called type II hypersensitivity; these signs result from RBC
hemolysis, agglutination, and capillary plugging that can damage renal function,
thus the flank pain and hematuria and the other manifestations.

137. A client jokes about his leukemia even though he is becoming sicker and
weaker. The nurse’s most therapeutic response would be:

A. “Your laugher is a cover for your fear.”


B. “He who laughs on the outside, cries on the inside.”
C. “Why are you always laughing?”
D. “Does it help you to joke about your illness?”

Answer: (D) “Does it help you to joke about your illness?”


This non-judgmentally on the part of the nurse points out the client’s behavior.

138. In dealing with a dying client who is in the denial stage of grief, the best
nursing approach is to:

A. Agree with and encourage the client’s denial


B. Reassure the client that everything will be okay
C. Allow the denial but be available to discuss death
D. Leave the client alone to discuss the loss

Answer: (C) Allow the denial but be available to discuss death


This does not take away the client’s only way of coping, and it permits future
movement through the grieving process when the client is ready. Dying clients
move through the different stages of grieving and the nurse must be ready to
intervene in all these stages.

139. During and 8 hour shift, Mario drinks two 6 oz. cups of tea and vomits 125
ml of fluid. During this 8 hour period, his fluid balance would be:

A. +55 ml
B. +137 ml
C. +235 ml
D. +485 ml
Answer: (C) +235 ml
The client’s intake was 360 ml (6oz x 30 ml) and loss was 125 ml of fluid; loss is
subtracted from intake

140. Mr. Ong is admitted to the hospital with a diagnosis of Left-sided CHF. In
the assessment, the nurse should expect to find:

A. Crushing chest pain


B. Dyspnea on exertion
C. Extensive peripheral edema
D. Jugular vein distention

Answer: (B) Dyspnea on exertion


Pulmonary congestion and edema occur because of fluid extravasation from the
pulmonary capillary bed, resulting in difficult breathing. Left-sided heart failure
creates a backward effect on the pulmonary system that leads to pulmonary
congestion.

141. The physician orders on a client with CHF a cardiac glycoside, a vasodilator,
and furosemide (Lasix). The nurse understands Lasix exerts is effects in the:

A. Distal tubule
B. Collecting duct
C. Glomerulus of the nephron
D. Ascending limb of the loop of Henle

Answer: (D) Ascending limb of the loop of Henle


This is the site of action of Lasix being a potent loop diuretic.

142. Mr. Ong weighs 210 lbs on admission to the hospital. After 2 days of
diuretic therapy he weighs 205.5 lbs. The nurse could estimate that the amount
of fluid he has lost is:

A. 0.5 L
B. 1.0 L
C. 2.0 L
D. 3.5 L

Answer: (C) 2.0 L


One liter of fluid weighs approximately 2.2 lbs. Therefore a 4.5 lbs weight loss
equals approximately 2 Liters.
143. Mr. Ong, a client with CHF, has been receiving a cardiac glycoside, a
diuretic, and a vasodilator drug. His apical pulse rate is 44 and he is on bed rest.
The nurse concludes that his pulse rate is most likely the result of the:

A. Diuretic
B. Vasodilator
C. Bed-rest regimen
D. Cardiac glycoside

Answer: (D) Cardiac glycoside


A cardiac glycoside such as digitalis increases force of cardiac contraction,
decreases the conduction speed of impulses within the myocardium and slows
the heart rate.

144. The diet ordered for a client with CHF permits him to have a 190 g of
carbohydrates, 90 g of fat and 100 g of protein. The nurse understands that this
diet contains approximately:

A. 2200 calories
B. 2000 calories
C. 2800 calories
D. 1600 calories

Answer: (B) 2000 calories


There are 9 calories in each gram of fat and 4 calories in each gram of
carbohydrate and protein

145. After the acute phase of congestive heart failure, the nurse should expect
the dietary management of the client to include the restriction of:

A. Magnesium
B. Sodium
C. Potassium
D. Calcium

Answer: (B) Sodium


Restriction of sodium reduces the amount of water retention that reduces the
cardiac workload
146. Jude develops GI bleeding and is admitted to the hospital. An important
etiologic clue for the nurse to explore while taking his history would be:

A. The medications he has been taking


B. Any recent foreign travel
C. His usual dietary pattern
D. His working patterns

Answer: (A) The medications he has been taking


Some medications, such as aspirin and prednisone, irritate the stomach lining
and may cause bleeding with prolonged use

147. The meal pattern that would probably be most appropriate for a client
recovering from GI bleeding is:

A. Three large meals large enough to supply adequate energy.


B. Regular meals and snacks to limit gastric discomfort
C. Limited food and fluid intake when he has pain
D. A flexible plan according to his appetite

Answer: (B) Regular meals and snacks to limit gastric discomfort


Presence of food in the stomach at regular intervals interacts with HCl limiting
acid mucosal irritation. Mucosal irritation can lead to bleeding.

148. A client with a history of recurrent GI bleeding is admitted to the hospital


for a gastrectomy. Following surgery, the client has a nasogastric tube to low
continuous suction. He begins to hyperventilate. The nurse should be aware that
this pattern will alter his arterial blood gases by:

A. Increasing HCO3
B. Decreasing PCO2
C. Decreasing pH
D. Decreasing PO2

Answer: (B) Decreasing PCO2


Hyperventilation results in the increased elimination of carbon dioxide from the
blood that can lead to respiratory alkalosis.

149. Routine postoperative IV fluids are designed to supply hydration and


electrolyte and only limited energy. Because 1 L of a 5% dextrose solution
contains 50 g of sugar, 3 L per day would apply approximately:
A. 400 Kilocalories
B. 600 Kilocalories
C. 800 Kilocalories
D. 1000 Kilocalories

Answer: (B) 600 Kilocalories


Carbohydrates provide 4 kcal/ gram; therefore 3L x 50 g/L x 4 kcal/g = 600
kcal; only about a third of the basal energy need.

150. Thrombus formation is a danger for all postoperative clients. The nurse
should act independently to prevent this complication by:

A. Encouraging adequate fluids


B. Applying elastic stockings
C. Massaging gently the legs with lotion
D. Performing active-assistive leg exercises

Answer: (D) Performing active-assistive leg exercises


Inactivity causes venous stasis, hypercoagulability, and external pressure
against the veins, all of which lead to thrombus formation. Early ambulation or
exercise of the lower extremities reduces the occurrence of this phenomenon

151. An unconscious client is admitted to the ICU, IV fluids are started and a
Foley catheter is inserted. With an indwelling catheter, urinary infection is a
potential danger. The nurse can best plan to avoid this problem by:

A. Emptying the drainage bag frequently


B. Collecting a weekly urine specimen
C. Maintaining the ordered hydration
D. Assessing urine specific gravity

Answer: (C) Maintaining the ordered hydration


Promoting hydration, maintains urine production at a higher rate, which flushes
the bladder and prevents urinary stasis and possible infection

152. The nurse performs full range of motion on a bedridden client’s extremities.
When putting his ankle through range of motion, the nurse must perform:

A. Flexion, extension and left and right rotation


B. Abduction, flexion, adduction and extension
C. Pronation, supination, rotation, and extension
D. Dorsiflexion, plantar flexion, eversion and inversion

Answer: (D) Dorsiflexion, plantar flexion, eversion and inversion


These movements include all possible range of motion for the ankle joint

153. A client has been in a coma for 2 months. The nurse understands that to
prevent the effects of shearing force on the skin, the head of the bed should be
at an angle of:

A. 30 degrees
B. 45 degrees
C. 60 degrees
D. 90 degrees

Answer: (A) 30 degrees


Shearing force occurs when 2 surfaces move against each other; when the bed is
at an angle greater than 30 degrees, the torso tends to slide and causes this
phenomenon. Shearing forces are good contributory factors of pressure sores.

154. Rene, age 62, is scheduled for a TURP after being diagnosed with a Benign
Prostatic Hyperplasia (BPH). As part of the preoperative teaching, the nurse
should tell the client that after surgery:

A. Urinary control may be permanently lost to some degree


B. Urinary drainage will be dependent on a urethral catheter for 24 hours
C. Frequency and burning on urination will last while the cystotomy tube is in
place
D. His ability to perform sexually will be permanently impaired

Answer: (B) Urinary drainage will be dependent on a urethral catheter for 24


hours
An indwelling urethral catheter is used, because surgical trauma can cause
urinary retention leading to further complications such as bleeding.

155. The transurethral resection of the prostate is performed on a client with


BPH. Following surgery, nursing care should include:

A. Changing the abdominal dressing


B. Maintaining patency of the cystotomy tube
C. Maintaining patency of a three-way Foley catheter for cystoclysis
D. Observing for hemorrhage and wound infection

Answer: (C) Maintaining patency of a three-way Foley catheter for cystoclysis


Patency of the catheter promotes bladder decompression, which prevents
distention and bleeding. Continuous flow of fluid through the bladder limits clot
formation and promotes hemostasis

156. In the early postoperative period following a transurethral surgery, the


most common complication the nurse should observe for is:

A. Sepsis
B. Hemorrhage
C. Leakage around the catheter
D. Urinary retention with overflow

Answer: (B) Hemorrhage


After transurethral surgery, hemorrhage is common because of venous oozing
and bleeding from many small arteries in the prostatic bed.
157. Following prostate surgery, the retention catheter is secured to the client’s
leg causing slight traction of the inflatable balloon against the prostatic fossa.
This is done to:

A. Limit discomfort
B. Provide hemostasis
C. Reduce bladder spasms
D. Promote urinary drainage

Answer: (B) Provide hemostasis


The pressure of the balloon against the small blood vessels of the prostate
creates a tampon-like effect that causes them to constrict thereby preventing
bleeding.

158. Twenty-four hours after TURP surgery, the client tells the nurse he has
lower abdominal discomfort. The nurse notes that the catheter drainage has
stopped. The nurse’s initial action should be to:

A. Irrigate the catheter with saline


B. Milk the catheter tubing
C. Remove the catheter
D. Notify the physician

Answer: (B) Milk the catheter tubing


Milking the tubing will usually dislodge the plug and will not harm the client. A
physician’s order is not necessary for a nurse to check catheter patency.

159. The nurse would know that a post-TURP client understood his discharge
teaching when he says “I should:”

A. Get out of bed into a chair for several hours daily


B. Call the physician if my urinary stream decreases
C. Attempt to void every 3 hours when I’m awake
D. Avoid vigorous exercise for 6 months after surgery

Answer: (B) Call the physician if my urinary stream decreases


Urethral mucosa in the prostatic area is destroyed during surgery and strictures
my form with healing that causes partial or even complete ueinary obstruction.
160. Lucy is admitted to the surgical unit for a subtotal thyroidectomy. She is
diagnosed with Grave’s Disease. When assessing Lucy, the nurse would expect
to find:

A. Lethargy, weight gain, and forgetfulness


B. Weight loss, protruding eyeballs, and lethargy
C. Weight loss, exopthalmos and restlessness
D. Constipation, dry skin, and weight gain

Answer: (C) Weight loss, exopthalmos and restlessness


Classic signs associated with hyperthyroidism are weight loss and restlessness
because of increased basal metabolic rate. Exopthalmos is due to peribulbar
edema.

161. Lucy undergoes Subtotal Thyroidectomy for Grave’s Disease. In planning


for the client’s return from the OR, the nurse would consider that in a subtotal
thyroidectomy:
A. The entire thyroid gland is removed
B. A small part of the gland is left intact
C. One parathyroid gland is also removed
D. A portion of the thyroid and four parathyroids are removed

Answer: (B) A small part of the gland is left intact


Remaining thyroid tissue may provide enough hormone for normal function.
Total thyroidectomy is generally done in clients with Thyroid Ca.

162. Before a post- thyroidectomy client returns to her room from the OR, the
nurse plans to set up emergency equipment, which should include:

A. A crash cart with bed board


B. A tracheostomy set and oxygen
C. An airway and rebreathing mask
D. Two ampules of sodium bicarbonate

Answer: (B) A tracheostomy set and oxygen


Acute respiratory obstruction in the post-operative period can result from
edema, subcutaneous bleeding that presses on the trachea, nerve damage, or
tetany.

163. When a post-thyroidectomy client returns from surgery the nurse assesses
her for unilateral injury of the laryngeal nerve every 30 to 60 minutes by:

A. Observing for signs of tetany


B. Checking her throat for swelling
C. Asking her to state her name out loud
D. Palpating the side of her neck for blood seepage

Answer: (C) Asking her to state her name out loud


If the recurrent laryngeal nerve is damaged during surgery, the client will be
hoarse and have difficult speaking.

164. On a post-thyroidectomy client’s discharge, the nurse teaches her to


observe for signs of surgically induced hypothyroidism. The nurse would know
that the client understands the teaching when she states she should notify the
physician if she develops:

A. Intolerance to heat
B. Dry skin and fatigue
C. Progressive weight loss
D. Insomnia and excitability

Answer: (B) Dry skin and fatigue


Dry skin is most likely caused by decreased glandular function and fatigue
caused by decreased metabolic rate. Body functions and metabolism are
decreased in hypothyroidism.

165. A client’s exopthalmos continues inspite of thyroidectomy for Grave’s


Disease. The nurse teaches her how to reduce discomfort and prevent corneal
ulceration. The nurse recognizes that the client understands the teaching when
she says: “I should:

A. Elevate the head of my bed at night


B. Avoid moving my extra-ocular muscles
C. Avoid using a sleeping mask at night
D. Avoid excessive blinking

Answer: (C) Avoid using a sleeping mask at night


The mask may irritate or scratch the eye if the client turns and lies on it during
the night.

166. Clara is a 37-year old cook. She is admitted for treatment of partial and
full-thickness burns of her entire right lower extremity and the anterior portion of
her right upper extremity. Her respiratory status is compromised, and she is in
pain and anxious.

Performing an immediate appraisal, using the rule of nines, the nurse estimates
the percent of Clara’s body surface that is burned is:

A. 4.5%
B. 9%
C. 18 %
D. 22.5%

Answer: (D) 22.5%


The entire right lower extremity is 18% the anterior portion of the right upper
extremity is 4.5% giving a total of 22.5%

167. The nurse applies mafenide acetate (Sulfamylon cream) to Clara, who has
second and third degree burns on the right upper and lower extremities, as
ordered by the physician. This medication will:

A. Inhibit bacterial growth


B. Relieve pain from the burn
C. Prevent scar tissue formation
D. Provide chemical debridement

Answer: (A) Inhibit bacterial growth


Sulfamylon is effective against a wide variety of gram positive and gram
negative organisms including anaerobes
168. Forty-eight hours after a burn injury, the physician orders for the client 2
liters of IV fluid to be administered q12 h. The drop factor of the tubing is 10
gtt/ml. The nurse should set the flow to provide:

A. 18 gtt/min
B. 28 gtt/min
C. 32 gtt/min
D. 36 gtt/min

Answer: (B) 28 gtt/min


This is the correct flow rate; multiply the amount to be infused (2000 ml) by the
drop factor (10) and divide the result by the amount of time in minutes (12
hours x 60 minutes)

169. Clara, a burn client, receives a temporary heterograft (pig skin) on some of
her burns. These grafts will:
A. Debride necrotic epithelium
B. Be sutured in place for better adherence
C. Relieve pain and promote rapid epithelialization
D. Frequently be used concurrently with topical antimicrobials.

Answer: (C) Relieve pain and promote rapid epithelialization


The graft covers nerve endings, which reduces pain and provides a framework
for granulation that promotes effective healing.

170. A client with burns on the chest has periodic episodes of dyspnea. The
position that would provide for the greatest respiratory capacity would be the:

A. Semi-fowler’s position
B. Sims’ position
C. Orthopneic position
D. Supine position

Answer: (C) Orthopneic position


The orthopneic position lowers the diaphragm and provides for maximal thoracic
expansion

171. Jane, a 20- year old college student is admiited to the hospital with a
tentative diagnosis of myasthenia gravis. She is scheduled to have a series of
diagnostic studies for myasthenia gravis, including a Tensilon test. In preparing
her for this procedure, the nurse explains that her response to the medication
will confirm the diagnosis if Tensilon produces:

A. Brief exaggeration of symptoms


B. Prolonged symptomatic improvement
C. Rapid but brief symptomatic improvement
D. Symptomatic improvement of just the ptosis

Answer: (C) Rapid but brief symptomatic improvement


Tensilon acts systemically to increase muscle strength; with a peak effect in 30
seconds, It lasts several minutes.

172. The initial nursing goal for a client with myasthenia gravis during the
diagnostic phase of her hospitalization would be to:

A. Develop a teaching plan


B. Facilitate psychologic adjustment
C. Maintain the present muscle strength
D. Prepare for the appearance of myasthenic crisis

Answer: (C) Maintain the present muscle strength


Until diagnosis is confirmed, primary goal should be to maintain adequate
activity and prevent muscle atrophy

173. The most significant initial nursing observations that need to be made about
a client with myasthenia include:
A. Ability to chew and speak distinctly
B. Degree of anxiety about her diagnosis
C. Ability to smile an to close her eyelids
D. Respiratory exchange and ability to swallow

Answer: (D) Respiratory exchange and ability to swallow


Muscle weakness can lead to respiratory failure that will require emergency
intervention and inability to swallow may lead to aspiration

174. Helen is diagnosed with myasthenia gravis and pyridostigmine bromide


(Mestinon) therapy is started. The Mestinon dosage is frequently changed during
the first week. While the dosage is being adjusted, the nurse’s priority
intervention is to:

A. Administer the medication exactly on time


B. Administer the medication with food or mild
C. Evaluate the client’s muscle strength hourly after medication
D. Evaluate the client’s emotional side effects between doses

Answer: (C) Evaluate the client’s muscle strength hourly after medication
Peak response occurs 1 hour after administration and lasts up to 8 hours; the
response will influence dosage levels.

175. Helen, a client with myasthenia gravis, begins to experience increased


difficulty in swallowing. To prevent aspiration of food, the nursing action that
would be most effective would be to:

A. Change her diet order from soft foods to clear liquids


B. Place an emergency tracheostomy set in her room
C. Assess her respiratory status before and after meals
D. Coordinate her meal schedule with the peak effect of her medication,
Mestinon

Answer: (D) Coordinate her meal schedule with the peak effect of her
medication, Mestinon
Dysphagia should be minimized during peak effect of Mestinon, thereby
decreasing the probability of aspiration. Mestinon can increase her muscle
strength including her ability to swallow.
Vision changes, such as diplopia, nystagmus, and blurred vision are symptoms of
multiple sclerosis. Deep tendon reflexes may be increased or hyperactive -- not absent.

Normal pressure in the anterior chamber of the eye remains relatively constant at 20 to
25 mm Hg.

subdural hematoma - A condition involving the collection of blood between the dura
mater and the brain.

Lichtheim's sign is the inability to speark associated with subcortical aphasia.

An injury from C5 to C6 results in quadriplegia, with diaphragmatic and gross arm


movemens.

transsphenoidal hypophysectomy - Microsurgery in which an incision is made at the


junction of the gums and upper lip. A surgical microscope is advanced and a special
surgical instrument is used to excise all or part of the pituitary gland.

polyneuritis - Degeneration of peripheral nerves primarily supplying the distal muscles


of the extremities. It results in muscle weakness, with sensory loss and atrophy, and
decreased or absent deep tendon reflexes.

Decerebrate posturing, characterized by abnormal extension in the response to painful


stimuli, indicates damage to the midbrain.

With damage to the diencephalon or cortex, abnormal flexion (decorticate posturing)


occurs when a painful stimulus is applied.

Damage to the medulla results in flacidity.

Receptive aphasia = damage to temporal lobe.

Damage to the parietal lobe affects the client's ability to identify special relationship
with the environment.

Dames to the occipital lobe affects visual association.

PERRLA stands for "Pupils Equal, Round, Reactive to Light and Accommodation"

audiometry - Evaluation of hearing using an audiometer. Various audiometric tests


identify the lowest intensity of sound at which a client can perceive an auditory
stimulus, hear different frequencies, and differentiate speech sounds. Pure tone
audiometry evaluates the ability to hear frequencies, usually ranging from 125 to 8,000
Hz, and can determine whether a hearing loss results from a problem in the middle ear,
inner ear, or auditory nerve.

Myasthenia gravis is characterized by a weakness of muscles, especially in the face and


throat, caused by a lower motor neuron lesion at the myoneural junction.

The nurse must dissolve crystallized mannitol before administering it. This is best doen
by warming it in hot water and shaking the container vigorously, then allowing the
solution to return to room temperature before administering it.

To prevent adverse reaction, which are commmon, I.V. diazepam should be


administered no faster than 5 mg/minute in an adult and should be given over at least
3 minutes in children.

Presbycusis is progression hearing loss associated with aging.

A decreased acetylcholine level has been implicated as a cause of cognitive changes in


healthy geriatric clients and in the severity of dementia.

A normal and stable ICP value is less than 15 mm Hg.

Chewing is a function of cranial nerve V (Trigeminal).

Swallowing is a motor function of cranial nerves IX and X.

epilepsy - A group of neurologic disorders marked by uncontrolled electrical discharge


from the cerebral cortex and typically manifested by seizures with clouding of
consciousness. Epilepsy is most commonly of unknown cause (idiopathic) but is
sometimes associated with head trauma, intracranial infection, brain tumor, vascular
disturbances, intoxication, or chemical imbalance.

cerebral contusion - A bruising of the brain tissue as a result of a severe blow to the
head. A contusion disrupts normal nerve function in the bruised area and may cause
loss of consciousness, hemorrhage, edema, and even death.

The catheter for measuring ICP is inserted through a burr hole into a lateral ventricle of
the cerebrum, thereby creating a risk of infection.

Succinylcholine (Anectine) a depolarizing blocking agent, is the drug of choice when


short-term muscle relaxation is desired -- for example during ECT or intubation.

Cones provide daylight color vision, and their stimulation is interpreted as color. Rods
are sensitive to low levels of illumination but cants discriminate color.
The sweat chloride test is used to confirm cystic fibrosis. Edrophonium (Tensilon) test
confirms the diagnosis of myasthenia gravis.

A helicopod gait is an abnormal gait in which the client's feet make a half circle with
each step. In a steppage gait, the feet and toes rish high off the floor and then heel
comes down heavily with each step.

When administering pilocarpine, the nurse should apply pressure on the inner canthus
to prevent systemic absorption of the drug.

Nonfluent aphasia is characterized by telegraphic speech, failure to use conjunctions


and pronouns and impared repetition and ability to read aloud.

In global aphasia, spontaneous speech is absent or limited to a few stereotyped words;


comprehension is limited to the client's name or a few words.

In fluent aphasia, auditory comprehension is disturbed; speech lacks meaningful


content, is unrelated to questions and includes paraphasias. The client with fluent
aphasia seems unaware that speech doesnt make sense and that reading and writing
are impared.

In anomic aphasia, the client can't name objects, has trouble finding words and may be
unable to read or write.

A basilar skull fracture commonly causes only periorbital ecchymosis (racoon's eyes)
and postmastoid ecchymosis (Battle's sign).

Atropine sulfate is a cholinergic blocker.

Lower brain stem dysfunction alters bulbar functions such as breathing, talking,
swallowing and coughing.
SITUATION : Arthur, A registered nurse, witnessed an old woman hit by a motorcycle
while crossing a train railway. The old woman fell at the railway. Arthur rushed at the
scene.

1. As a registered nurse, Arthur knew that the first thing that he will do at the scene is 

A. Stay with the person, Encourage her to remain still and Immobilize the leg while
While waiting for the ambulance.
B. Leave the person for a few moments to call for help.
C. Reduce the fracture manually.
D. Move the person to a safer place.

2. Arthur suspects a hip fracture when he noticed that the old woman’s leg is 

A. Lengthened, Abducted and Internally Rotated.


B. Shortened, Abducted and Externally Rotated.
C. Shortened, Adducted and Internally Rotated.
D. Shortened, Adducted and Externally Rotated.

3. The old woman complains of pain. John noticed that the knee is reddened, warm to
touch and swollen. John interprets that this signs and symptoms are likely related to 

A. Infection
B. Thrombophlebitis
C. Inflammation
D. Degenerative disease

4. The old woman told John that she has osteoporosis; Arthur knew that all of the
following factors would contribute to osteoporosis except 

A. Hypothyroidism
B. End stage renal disease
C. Cushing’s Disease
D. Taking Furosemide and Phenytoin.

5. Martha, The old woman was now Immobilized and brought to the emergency room.
The X-ray shows a fractured femur and pelvis. The ER Nurse would carefully monitor
Martha for which of the following sign and symptoms? 

A. Tachycardia and Hypotension


B. Fever and Bradycardia
C. Bradycardia and Hypertension
D. Fever and Hypertension

SITUATION: Mr. D. Rojas, An obese 35 year old MS Professor of OLFU Lagro is admitted
due to pain in his weight bearing joint. The diagnosis was Osteoarthritis.

6. As a nurse, you instructed Mr. Rojas how to use a cane. Mr. Rojas has a weakness on
his right leg due to self immobilization and guarding. You plan to teach Mr. Rojas to
hold the cane 

A. On his left hand, because his right side is weak.


B. On his left hand, because of reciprocal motion.
C. On his right hand, to support the right leg.
D. On his right hand, because only his right leg is weak.

7. You also told Mr. Rojas to hold the cane 

A. 1 Inches in front of the foot.


B. 3 Inches at the lateral side of the foot.
c. 6 Inches at the lateral side of the foot.
D. 12 Inches at the lateral side of the foot.

8. Mr. Rojas was discharged and 6 months later, he came back to the emergency room
of the hospital because he suffered a mild stroke. The right side of the brain was
affected. At the rehabilitative phase of your nursing care, you observe Mr. Rojas use a
cane and you intervene if you see him 

A. Moves the cane when the right leg is moved.


B. Leans on the cane when the right leg swings through.
C. keeps the cane 6 Inches out to the side of the right foot.
D. Holds the cane on the right side.

SITUATION: Alfred, a 40 year old construction worker developed cough, night sweats
and fever. He was brought to the nursing unit for diagnostic studies. He told the nurse
he did not receive a BCG vaccine during childhood

9. The nurse performs a Mantoux Test. The nurse knows that Mantoux Test is also
known as 

A. PPD
B. PDP
C. PDD
D. DPP

10. The nurse would inject the solution in what route? 


A. IM
B. IV
C. ID
D. SC

11. The nurse notes that a positive result for Alfred is 

A. 5 mm wheal
B. 5 mm Induration
C. 10 mm Wheal
D. 10 mm Induration

12. The nurse told Alfred to come back after 

A. a week
B. 48 hours
C. 1 day
D. 4 days

13. Mang Alfred returns after the Mantoux Test. The test result read POSITIVE. What
should be the nurse’s next action? 

A. Call the Physician


B. Notify the radiology dept. for CXR evaluation
C. Isolate the patient
D. Order for a sputum exam

14. Why is Mantoux test not routinely done in the Philippines? 

A. It requires a highly skilled nurse to perform a Mantoux test


B. The sputum culture is the gold standard of PTB Diagnosis and it will definitively
determine the extent of the cavitary lesions
C. Chest X Ray Can diagnose the specific microorganism responsible for the lesions
D. Almost all Filipinos will test positive for Mantoux Test

15. Mang Alfred is now a new TB patient with an active disease. What is his category
according to the DOH? 

A. I
B. II
C. III
D. IV
16. How long is the duration of the maintenance phase of his treatment? 

A. 2 months
B. 3 months
C. 4 months
D. 5 months

17. Which of the following drugs is UNLIKELY given to Mang Alfred during the
maintenance phase? 

A. Rifampicin
B. Isoniazid
C. Ethambutol
D. Pyridoxine

18. According to the DOH, the most hazardous period for development of clinical
disease is during the first 

A. 6-12 months after


B. 3-6 months after
C. 1-2 months after
D. 2-4 weeks after

19. This is the name of the program of the DOH to control TB in the country 

A. DOTS
B. National Tuberculosis Control Program
C. Short Coursed Chemotherapy
D. Expanded Program for Immunization

20. Susceptibility for the disease [ TB ] is increased markedly in those with the
following condition except 

A. 23 Year old athlete with diabetes insipidus


B. 23 Year old athlete taking long term Decadron therapy and anabolic steroids
C. 23 Year old athlete taking illegal drugs and abusing substances
D. Undernourished and Underweight individual who undergone gastrectomy

21. Direct sputum examination and Chest X ray of TB symptomatic is in what level of
prevention? 

A. Primary
B. Secondary
C. Tertiary
D. Quarterly

SITUATION: Michiel, A male patient diagnosed with colon cancer was newly put in
colostomy.

22. Michiel shows the BEST adaptation with the new colostomy if he shows which of the
following? 

A. Look at the ostomy site


B. Participate with the nurse in his daily ostomy care
C. Ask for leaflets and contact numbers of ostomy support groups
D. Talk about his ostomy openly to the nurse and friends

23. The nurse plans to teach Michiel about colostomy irrigation. As the nurse prepares
the materials needed, which of the following item indicates that the nurse needs further
instruction? 

A. Plain NSS / Normal Saline


B. K-Y Jelly
C. Tap water
D. Irrigation sleeve

24. The nurse should insert the colostomy tube for irrigation at approximately 

A. 1-2 inches
B. 3-4 inches
C. 6-8 inches
D. 12-18 inches

25. The maximum height of irrigation solution for colostomy is 

A. 5 inches
B. 12 inches
C. 18 inches
D. 24 inches

26. Which of the following behavior of the client indicates the best initial step in
learning to care for his colostomy? 

A. Ask to defer colostomy care to another individual


B. Promises he will begin to listen the next day
C. Agrees to look at the colostomy
D. States that colostomy care is the function of the nurse while he is in the hospital
27. While irrigating the client’s colostomy, Michiel suddenly complains of severe
cramping. Initially, the nurse would 

A. Stop the irrigation by clamping the tube


B. Slow down the irrigation
C. Tell the client that cramping will subside and is normal
D. Notify the physician

28. The next day, the nurse will assess Michiel’s stoma. The nurse noticed that a
prolapsed stoma is evident if she sees which of the following? 

A. A sunken and hidden stoma


B. A dusky and bluish stoma
C. A narrow and flattened stoma
D. Protruding stoma with swollen appearance

29. Michiel asked the nurse, what foods will help lessen the odor of his colostomy. The
nurse best response would be 

A. Eat eggs
B. Eat cucumbers
C. Eat beet greens and parsley
D. Eat broccoli and spinach

30. The nurse will start to teach Michiel about the techniques for colostomy irrigation.
Which of the following should be included in the nurse’s teaching plan? 

A. Use 500 ml to 1,000 ml NSS


B. Suspend the irrigant 45 cm above the stoma
C. Insert the cone 4 cm in the stoma
D. If cramping occurs, slow the irrigation

31. The nurse knew that the normal color of Michiel’s stoma should be 

A. Brick Red
B. Gray
C. Blue
D. Pale Pink

SITUATION: James, A 27 basketball player sustained inhalation burn that required him
to have tracheostomy due to massive upper airway edema.

32. Wilma, His sister and a nurse is suctioning the tracheostomy tube of James. Which
of the following, if made by Wilma indicates that she is committing an error? 
A. Hyperventilating James with 100% oxygen before and after suctioning
B. Instilling 3 to 5 ml normal saline to loosen up secretion
C. Applying suction during catheter withdrawal
D. Suction the client every hour

33. What size of suction catheter would Wilma use for James, who is 6 feet 5 inches in
height and weighing approximately 145 lbs? 

A. Fr. 5
B. Fr. 10
C. Fr. 12
D. Fr. 18

34. Wilma is using a portable suction unit at home, What is the amount of suction
required by James using this unit? 

A. 2-5 mmHg
B. 5-10 mmHg
C. 10-15 mmHg
D. 20-25 mmHg

35. If a Wall unit is used, What should be the suctioning pressure required by James? 

A. 50-95 mmHg
B. 95-110 mmHg
C. 100-120 mmHg
D. 155-175 mmHg

36. Wilma was shocked to see that the Tracheostomy was dislodged. Both the inner
and outer cannulas was removed and left hanging on James’ neck. What are the 2
equipment’s at james’ bedside that could help Wilma deal with this situation? 

A. New set of tracheostomy tubes and Oxygen tank


B. Theophylline and Epinephrine
C. Obturator and Kelly clamp
D. Sterile saline dressing

37. Which of the following method if used by Wilma will best assure that the
tracheostomy ties are not too tightly placed? 

A. Wilma places 2 fingers between the tie and neck


B. The tracheotomy can be pulled slightly away from the neck
C. James’ neck veins are not engorged
D. Wilma measures the tie from the nose to the tip of the earlobe and to the xiphoid
process.

38. Wilma knew that James have an adequate respiratory condition if she notices that 

A. James’ respiratory rate is 18


B. James’ Oxygen saturation is 91%
C. There are frank blood suction from the tube
D. There are moderate amount of tracheobronchial secretions

39. Wilma knew that the maximum time when suctioning James is 

A. 10 seconds
B. 20 seconds
C. 30 seconds
D. 45 seconds

SITUATION : Juan Miguel Lopez Zobel Ayala de Batumbakal was diagnosed with Acute
Close Angle Glaucoma. He is being seen by Nurse Jet.

40. What specific manifestation would nurse Jet see in Acute close angle glaucoma that
she would not see in an open angle glaucoma? 

A. Loss of peripheral vision


B. Irreversible vision loss
C. There is an increase in IOP
D. Pain

41. Nurse jet knew that Acute close angle glaucoma is caused by 

A. Sudden blockage of the anterior angle by the base of the iris


B. Obstruction in trabecular meshwork
C. Gradual increase of IOP
D. An abrupt rise in IOP from 8 to 15 mmHg

42. Nurse jet performed a TONOMETRY test to Mr. Batumbakal. What does this test
measures 

A. It measures the peripheral vision remaining on the client


B. Measures the Intra Ocular Pressure
C. Measures the Client’s Visual Acuity
D. Determines the Tone of the eye in response to the sudden increase in IOP.

43. The Nurse notices that Mr. Batumbakal cannot anymore determine RED from BLUE.
The nurse knew that which part of the eye is affected by this change? 

A. IRIS
B. PUPIL
c. RODS [RETINA]
D. CONES [RETINA]

44. Nurse Jet knows that Aqueous Humor is produce where? 

A. In the sub arachnoid space of the meninges


B. In the Lateral ventricles
C. In the Choroids
D. In the Ciliary Body

45. Nurse Jet knows that the normal IOP is 

A. 8-21 mmHg
B. 2-7 mmHg
c. 31-35 mmHg
D. 15-30 mmHg

46. Nurse Jet wants to measure Mr. Batumbakal’s CN II Function. What test would
Nurse Jet implement to measure CN II’s Acuity? 

A. Slit lamp
B. Snellen’s Chart
C. Wood’s light
D. Gonioscopy

47. The Doctor orders pilocarpine. Nurse jet knows that the action of this drug is to 

A. Contract the Ciliary muscle


B. Relax the Ciliary muscle
C. Dilate the pupils
D. Decrease production of Aqueous Humor

48. The doctor orders timolol [timoptic]. Nurse jet knows that the action of this drug is 

A. Reduce production of CSF


B. Reduce production of Aquesous Humor
C. Constrict the pupil
D. Relaxes the Ciliary muscle

49. When caring for Mr. Batumbakal, Jet teaches the client to avoid 
A. Watching large screen TVs
B. Bending at the waist
C. Reading books
D. Going out in the sun

50. Mr. Batumbakal has undergone eye angiography using an Intravenous dye and
fluoroscopy. What activity is contraindicated immediately after procedure? 

A. Reading newsprint
B. Lying down
C. Watching TV
D. Listening to the music

51. If Mr. Batumbakal is receiving pilocarpine, what drug should always be available in
any case systemic toxicity occurs? 

A. Atropine Sulfate
B. Pindolol [Visken]
C. Naloxone Hydrochloride [Narcan]
D. Mesoridazine Besylate [Serentil]

SITUATION : Wide knowledge about the human ear, it’s parts and it’s functions will help
a nurse assess and analyze changes in the adult client’s health.

52. Nurse Anna is doing a caloric testing to his patient, Aida, a 55 year old university
professor who recently went into coma after being mauled by her disgruntled 3rd year
nursing students whom she gave a failing mark. After instilling a warm water in the ear,
Anna noticed a rotary nystagmus towards the irrigated ear. What does this means? 

A. Indicates a CN VIII Dysfunction


B. Abnormal
C. Normal
D. Inconclusive

53. Ear drops are prescribed to an infant, The most appropriate method to administer
the ear drops is 

A. Pull the pinna up and back and direct the solution towards the eardrum
B. Pull the pinna down and back and direct the solution onto the wall of the canal
C. Pull the pinna down and back and direct the solution towards the eardrum
D. Pull the pinna up and back and direct the solution onto the wall of the canal
54. Nurse Jenny is developing a plan of care for a patient with Menieres disease. What
is the priority nursing intervention in the plan of care for this particular patient? 

A. Air, Breathing, Circulation


B. Love and Belongingness
C. Food, Diet and Nutrition
D. Safety

55. After mastoidectomy, Nurse John should be aware that the cranial nerve that is
usually damage after this procedure is 

A. CN I
B. CN II
C. CN VII
D. CN VI

56. The physician orders the following for the client with Menieres disease. Which of the
following should the nurse question? 

A. Dipenhydramine [Benadryl]
B. Atropine sulfate
C. Out of bed activities and ambulation
D. Diazepam [Valium]

57. Nurse Anna is giving dietary instruction to a client with Menieres disease. Which
statement if made by the client indicates that the teaching has been successful? 

A. I will try to eat foods that are low in sodium and limit my fluid intake
B. I must drink atleast 3,000 ml of fluids per day
C. I will try to follow a 50% carbohydrate, 30% fat and 20% protein diet
D. I will not eat turnips, red meat and raddish

58. Peachy was rushed by his father, Steven into the hospital admission. Peachy is
complaining of something buzzing into her ears. Nurse Joemar assessed peachy and
found out It was an insect. What should be the first thing that Nurse Joemar should try
to remove the insect out from peachy’s ear? 

A. Use a flashlight to coax the insect out of peachy’s ear


B. Instill an antibiotic ear drops
C. Irrigate the ear
D. Pick out the insect using a sterile clean forceps

59. Following an ear surgery, which statement if heard by Nurse Oca from the patient
indicates a correct understanding of the post operative instructions? 
A. Activities are resumed within 5 days
B. I will make sure that I will clean my hair and face to prevent infection
C. I will use straw for drinking
D. I should avoid air travel for a while

60. Nurse Oca will do a caloric testing to a client who sustained a blunt injury in the
head. He instilled a cold water in the client’s right ear and he noticed that nystagmus
occurred towards the left ear. What does this finding indicates? 

A. Indicating a Cranial Nerve VIII Dysfunction


B. The test should be repeated again because the result is vague
C. This is Grossly abnormal and should be reported to the neurosurgeon
D. This indicates an intact and working vestibular branch of CN VIII

61. A client with Cataract is about to undergo surgery. Nurse Oca is preparing plan of
care. Which of the following nursing diagnosis is most appropriate to address the long
term need of this type of patient? 

A. Anxiety R/T to the operation and its outcome


B. Sensory perceptual alteration R/T Lens extraction and replacement
C. Knowledge deficit R/T the pre operative and post operative self care
D. Body Image disturbance R/T the eye packing after surgery

62. Nurse Joseph is performing a WEBERS TEST. He placed the tuning fork in the
patients forehead after tapping it onto his knee. The client states that the fork is louder
in the LEFT EAR. Which of the following is a correct conclusion for nurse Josph to
make? 

A. He might have a sensory hearing loss in the left ear


B. Conductive hearing loss is possible in the right ear
C. He might have a sensory hearing loss in the right hear, and/or a conductive hearing
loss in the left ear.
D. He might have a conductive hearing loss in the right ear, and/or a sensory hearing
loss in the left ear.

63. Aling myrna has Menieres disease. What typical dietary prescription would nurse
Oca expect the doctor to prescribe? 

A. A low sodium , high fluid intake


B. A high calorie, high protein dietary intake
C. low fat, low sodium and high calorie intake
D. low sodium and restricted fluid intake
SITUATION : [ From DEC 1991 NLE ] A 45 year old male construction worker was
admitted to a tertiary hospital for incessant vomiting. Assessment disclosed: weak rapid
pulse, acute weight loss of .5kg, furrows in his tongue, slow flattening of the skin was
noted when the nurse released her pinch.

Temperature: 35.8 C , BUN Creatinine ratio : 10 : 1, He also complains for postural


hypotension. There was no infection.

64. Which of the following is the appropriate nursing diagnosis? 

A. Fluid volume deficit R/T furrow tongue


B. Fluid volume deficit R/T uncontrolled vomiting
C. Dehydration R/T subnormal body temperature
D. Dehydration R/T incessant vomiting

65. Approximately how much fluid is lost in acute weight loss of .5kg? 

A. 50 ml
B. 750 ml
C. 500 ml
D. 75 ml

66. Postural Hypotension is 

A. A drop in systolic pressure less than 10 mmHg when patient changes position from
lying to sitting.
B. A drop in systolic pressure greater than 10 mmHg when patient changes position
from lying to sitting
C. A drop in diastolic pressure less than 10 mmHg when patient changes position from
lying to sitting
D. A drop in diastolic pressure greater than 10 mmHg when patient changes position
from lying to sitting

67. Which of the following measures will not help correct the patient’s condition 

A. Offer large amount of oral fluid intake to replace fluid lost


B. Give enteral or parenteral fluid
C. Frequent oral care
D. Give small volumes of fluid at frequent interval

68. After nursing intervention, you will expect the patient to have 

1. Maintain body temperature at 36.5 C


2. Exhibit return of BP and Pulse to normal
3. Manifest normal skin turgor of skin and tongue
4. Drinks fluids as prescribed

A. 1,3
B. 2,4
C. 1,3,4
D. 2,3,4

SITUATION: A 65 year old woman was admitted for Parkinson’s Disease. The charge
nurse is going to make an initial assessment.

69. Which of the following is a characteristic of a patient with advanced Parkinson’s


disease? 

A. Disturbed vision
B. Forgetfulness
C. Mask like facial expression
D. Muscle atrophy

70. The onset of Parkinson’s disease is between 50-60 years old. This disorder is
caused by 

A. Injurious chemical substances


B. Hereditary factors
C. Death of brain cells due to old age
D. Impairment of dopamine producing cells in the brain

71. The patient was prescribed with levodopa. What is the action of this drug? 

A. Increase dopamine availability


B. Activates dopaminergic receptors in the basal ganglia
C. Decrease acetylcholine availability
D. Release dopamine and other catecholamine from neurological storage sites

72. You are discussing with the dietician what food to avoid with patients taking
levodopa? 

A. Vitamin C rich food


B. Vitamin E rich food
C. Thiamine rich food
D. Vitamin B6 rich food

73. One day, the patient complained of difficulty in walking. Your response would be 
A. You will need a cane for support
B. Walk erect with eyes on horizon
C. I’ll get you a wheelchair
D. Don’t force yourself to walk

SITUATION: Mr. Dela Isla, a client with early Dementia exhibits thought process
disturbances.

74. The nurse will assess a loss of ability in which of the following areas? 

A. Balance
B. Judgment
C. Speech
D. Endurance

75. Mr. Dela Isla said he cannot comprehend what the nurse was saying. He suffers
from: 

A. Insomnia
B. Aphraxia
C. Agnosia
D. Aphasia

76. The nurse is aware that in communicating with an elderly client, the nurse will 

A. Lean and shout at the ear of the client


B. Open mouth wide while talking to the client
C. Use a low-pitched voice
D. Use a medium-pitched voice

77. As the nurse talks to the daughter of Mr. Dela Isla, which of the following statement
of the daughter will require the nurse to give further teaching? 

A. I know the hallucinations are parts of the disease


B. I told her she is wrong and I explained to her what is right
C. I help her do some tasks he cannot do for himself
D. Ill turn off the TV when we go to another room

78. Which of the following is most important discharge teaching for Mr. Dela Isla 

A. Emergency Numbers
B. Drug Compliance
C. Relaxation technique
D. Dietary prescription

SITUATION : Knowledge of the drug PROPANTHELINE BROMIDE [Probanthine] Is


necessary in treatment of various disorders.

79. What is the action of this drug? 

A. Increases glandular secretion for clients affected with cystic fibrosis


B. Dissolve blockage of the urinary tract due to obstruction of cystine stones
C. Reduces secretion of the glandular organ of the body
D. Stimulate peristalsis for treatment of constipation and obstruction

80. What should the nurse caution the client when using this medication 

A. Avoid hazardous activities like driving, operating machineries etc.


B. Take the drug on empty stomach
C. Take with a full glass of water in treatment of Ulcerative colitis
D. I must take double dose if I missed the previous dose

81. Which of the following drugs are not compatible when taking Probanthine? 

A. Caffeine
B. NSAID
C. Acetaminophen
D. Alcohol

82. What should the nurse tell clients when taking Probanthine? 

A. Avoid hot weathers to prevent heat strokes


B. Never swim on a chlorinated pool
C. Make sure you limit your fluid intake to 1L a day
D. Avoid cold weathers to prevent hypothermia

83. Which of the following disease would Probanthine exert the much needed action for
control or treatment of the disorder? 

A. Urinary retention
B. Peptic Ulcer Disease
C. Ulcerative Colitis
D. Glaucoma

SITUATION : Mr. Franco, 70 years old, suddenly could not lift his spoons nor speak at
breakfast. He was rushed to the hospital unconscious. His diagnosis was CVA.
84. Which of the following is the most important assessment during the acute stage of
an unconscious patient like Mr. Franco? 

A. Level of awareness and response to pain


B. Papillary reflexes and response to sensory stimuli
C. Coherence and sense of hearing
D. Patency of airway and adequacy of respiration

85. Considering Mr. Franco’s conditions, which of the following is most important to
include in preparing Franco’s bedside equipment? 

A. Hand bell and extra bed linen


B. Sandbag and trochanter rolls
C. Footboard and splint
D. Suction machine and gloves

86. What is the rationale for giving Mr. Franco frequent mouth care? 

A. He will be thirsty considering that he is doesn’t drink enough fluids


B. To remove dried blood when tongue is bitten during a seizure
C. The tactile stimulation during mouth care will hasten return to consciousness
D. Mouth breathing is used by comatose patient and it’ll cause oral mucosa dying and
cracking.

87. One of the complications of prolonged bed rest is decubitus ulcer. Which of the
following can best prevent its occurrence? 

A. Massage reddened areas with lotion or oils


B. Turn frequently every 2 hours
C. Use special water mattress
D. Keep skin clean and dry

88. If Mr. Franco’s Right side is weak, What should be the most accurate analysis by
the nurse? 

A. Expressive aphasia is prominent on clients with right sided weakness


B. The affected lobe in the patient is the Right lobe
C. The client will have problems in judging distance and proprioception
D. Clients orientation to time and space will be much affected

SITUATION : a 20 year old college student was rushed to the ER of PGH after he fainted
during their ROTC drill. Complained of severe right iliac pain. Upon palpation of his
abdomen, Ernie jerks even on slight pressure. Blood test was ordered. Diagnosis is
acute appendicitis.
89. Which result of the lab test will be significant to the diagnosis? 

A. RBC : 4.5 TO 5 Million / cu. mm.


B. Hgb : 13 to 14 gm/dl.
C. Platelets : 250,000 to 500,000 cu.mm.
D. WBC : 12,000 to 13,000/cu.mm

90. Stat appendectomy was indicated. Pre op care would include all of the following
except? 

A. Consent signed by the father


B. Enema STAT
C. Skin prep of the area including the pubis
D. Remove the jewelries

91. Pre-anesthetic med of Demerol and atrophine sulfate were ordered to : 

A. Allay anxiety and apprehension


B. Reduce pain
C. Prevent vomiting
D. Relax abdominal muscle

92. Common anesthesia for appendectomy is 

A. Spinal
B. General
C. Caudal
D. Hypnosis

93. Post op care for appendectomy include the following except 

A. Early ambulation
B. Diet as tolerated after fully conscious
C. Nasogastric tube connect to suction
D. Deep breathing and leg exercise

94. Peritonitis may occur in ruptured appendix and may cause serious problems which
are 

1. Hypovolemia, electrolyte imbalance


2. Elevated temperature, weakness and diaphoresis
3. Nausea and vomiting, rigidity of the abdominal wall
4. Pallor and eventually shock
A. 1 and 2
B. 2 and 3
C. 1,2,3
D. All of the above

95. If after surgery the patient’s abdomen becomes distended and no bowel sounds
appreciated, what would be the most suspected complication? 

A. Intussusception
B. Paralytic Ileus
C. Hemorrhage
D. Ruptured colon

96. NGT was connected to suction. In caring for the patient with NGT, the nurse must 

A. Irrigate the tube with saline as ordered


B. Use sterile technique in irrigating the tube
C. advance the tube every hour to avoid kinks
D. Offer some ice chips to wet lips

97. When do you think the NGT tube be removed? 

A. When patient requests for it


B. Abdomen is soft and patient asks for water
C. Abdomen is soft and flatus has been expelled
D. B and C only

Situation: Amanda is suffering from chronic arteriosclerosis Brain syndrome she fell
while getting out of the bed one morning and was brought to the hospital, and she was
diagnosed to have cerebrovascular thrombosis thus transferred to a nursing home.

98. What do you call a STROKE that manifests a bizarre behavior? 

A. Inorganic Stroke
B. Inorganic Psychoses
C. Organic Stroke
D. Organic Psychoses

99. The main difference between chronic and organic brain syndrome is that the
former 

A. Occurs suddenly and reversible


B. Is progressive and reversible
C. tends to be progressive and irreversible
D. Occurs suddenly and irreversible

100. Which behavior results from organic psychoses? 

A. Memory deficit
B. Disorientation
C. Impaired Judgement
D. Inappropriate affect
Situation 1: A nurse who is assigned in a medical ward took time to be prepared with
her task and give quality nursing care.

1. If a client with increased pressure (ICP) demonstrates decorticate


posturing, the nurse will observe:

a. Flexion of both upper and lower extremities 


b. Extension of elbows and knees, plantar flexion of feet, and flexion of the wnsts
c. Flexion of elbows, extension of the knees, and plantar flexion of the feet 
d. Extension of upper extremities, flexion of lower extremities

2.The physician orders propranolol (Inderal) for a client's angina. The effect of
this drug is to:

a. Act as a vasoconstrictor 
b. Act as a vasodilator 
c. Block beta stimulation in the heart 
d. Increase the heart rate 

3. A client with alcoholic cirrhosis with ascites and portal hypertension is to


receive neomydn. The desired effect of this drug is to;

a. Sterilize the bowel


b. Reduce abdominal distention
c. Decrease the serum ammonia 
d. Prevent infection

4. A retention catheter for a male client is correctly taped if it is:

a. On the lower abdomen 


b. On the umbilicus 
c. Under the thigh 
d. On the inner thigh

5. When assessing a client for Cournadin therapy, the condition that will
eyclude this client from Coumadin therapy is: 

a. Diabetes 
b. Arthritis 
c. Pregnancy 
d. Peptic ulcer disease

6. Preparing for an intravenous pyelosram (IVP), the nurse instructs a 25-


year-old male client to restrict her: 

a. Fluid intake 
b. Physical activity 
c. Use of stimulants such as tobacco 
d. Use of any medications

7. Immediately following a thoracentesis, which clinical manifestations


indicate that a complication has occurred and the physician should be
notified? 

a. Serosanguimeous drainage from the puncture site


b increased temperature and blood pressure
c. increased pulse and pallor
d. Hypotension and hypothermia

8. The nurse is collecting a urine specimen from a client who has been
catheterized. When the urine begins to flow through ths catheter, the next
action is to:

a. Inflate the catheter balloon with sterile water


b. Place the catheter tip into the specimen container
c. Connect the catheter into the drainage tubing
c. Place the catheter tip into the urine collection receptacle

9. During a retention catheter insertion or bladder irrigation, the nurse must


use:

a. Sterils equipment and wear sterile gloves 


b. Clean equipment and maintain surgical asepsis 
c. Sterile equipment and maintain medical asepsis 
d. Clean equipment and technique

10. If a client continues to hypoventilate, the nurse will continually assess for
a complication of this condition;

a. Respiratory acidosis 
b. Respiratory alkalosis 
c. Metabolic acidosis 
d. Metabolic alkalosis

Situation 2: Diabetes Meilitus is a common disease among Filipinos. Caring for these
patients require meticulous assessment and follow-up.
11. The nurse will know a diabetic client understands exercise and its relation
to glucose when he says that he eats bread and milk before, or juice or fruit
during exercise activity because

a. Exercise enhances the passage of glucose Into muscle celts


b. Exercise stimulates pancreatic insulin production
c. A diabetic's muscle require more glucose during exercise
d. The pancreas utilizes more glucose during exercise

12. The ADA exchange diet is compiled of lists of foods. The statement that
indicates the diabetic has an understanding of the purpose of these food lists
is:

a. Exchanges are allowed within groups


b. Exchanges are allowed between groups
c. Only meat and fat exchanges can be interchanged
d. Vegetables and fruit exchanges can be Interchanged

13. The non-insulin-dependent diabetic who is obese is best controlled by


weight
loss because obesity

a. Reduces the number of insulin receptors 


b. Cause pancreatic islet cell exhaustion 
c. Reduces insulin binding at receptor cites 
d. Reduces pancreatic insulin production

14. A person with a diagnosis of adult diabetes (NIDDM) should understand


the symptoms of a hyperglycemic reaction. The nurse wiiS know tills client
understands if she says these symptoms are:

a. Thirst, poiyuria and decreased appetite


b. Flushed cheeks, acetone breath, and increased thirst
c. Nausea, vomiting and diarrhea
d. Weight gain, normal breath, and thirst

15. The diabetic client the nurse is counseling is a young man who
occasionally goes drinking with his buddies. The nurse will know the client
understands the diet when he says that when he consumes alcohol, he
includes il as part of:

a. Protein
b. Simple carbohydrates
c. Complex carbohydrates
d. Fats

16. The nurse is teaching a Type 1 diabetic client about her diet, which is
based on the exchange system. The nurse wiil know the dient has learned
correctiy when she says that she can have as much as she wants of:

a. Lettuce
b. Tomato
c. Grapefruit juice
d. Skim milk

17. The nurse should evpiain to a dient with diabetes meliitus that self-
monitoring of blood glucose is preferred to urine glucose testing because it is:

a. More accurate .
b. Easier to perform
c. Done by the cient
d. Not influenced by drugs

18.A client is diagnosed as having non-insulin-dependent diabetes mellitus


how to
provide self-care to prevent infections of the feet. The nurse recognizes that
the
teaching was effective when the client says, I should:

a. "Massage my feet and feet with oil or lotion."


b. "Apply heat intermittently to my feet and legs."
c. "Eat foods high in kilocalories of protein and carbohydrates."
d. "Control my diabetes through diet, exercise, and medication."

19. A client is admitted to the hospital with diabetic ketoadosis. The nurse
understands that the elevated ketone level present with this disorder is
caused by the incomplete oxidation of: 

a. Fats
b. Protein
c. Potassium
d. Carbohydrates

20. A client with insulin-dependent diabetes is pjaced on an insulin pump. The


most appropriate short-term goal in teaching this client to control the
diabetes: " The client will: 

a. Adhere to the medical regimen."


b. Remain normogtycemic for 3 weeks."
c. Demonstrate the correct use of the insulin pump."
d. List three self-care activities necessary to control the diabetes."

Situation 3: In the CCU, the nurse has a patient who needs to be,watched out.

21. To determine the status of a clients carotid pulse, the nurse should
palpate:

a. In the lateral neck region 


b. Immediately below the mandible 
c. At the anterior necK, lateral to the trachea 
d. At the base of the neck", along the clavicle

22. To help reduce a client's risk factors for a heart disease, the nurse, in
discussing dietary guidelines, should teach the client to:

a. Avoid eating between meals 


b. Decrease the amount of uhsaturated fat 
c. Decrease the amount of fat-binding fiber 
d. Increase the ratio of complex carbohydrates

23. The nurse would expect a client diagnosed as having hypertension to


report
experiencing the most common symptom associated with this disorder, which
is:

a. Fatigue 
b. Headache 
c. Nosebleeds
d. Flushed face

24. A client with a history of hypertension develops pedal edema and


demonstrates dyspnea on exertion. The nurse recognizes that the client's
dyspnea on exertion is probably;

a. Caused by cor pulmonale


b. A result of left ventricular failure
c. A result of right ventricular failure
d. Associated with wheezing and coughing

25. A client who has been admitted to the cardiac care unit with myocardial
infarction complains of chest pain. The nursing intervention that would be
most effective in relieving the client's pain would be to administer the
ordered:

a. Morphine sulfate 2 mg IV
b. Oxygen per nasal cannula
c. Nitroglycerine sublingually
d. Lidocaine hydrochloride 50 mg IV bolus

26. The nurse admitting a client with a myocardial Infarction to ICU


understands that the pain the client is experiencing is a result of:

a. Compression of the heart muscle 


b. Release of myocardia! isoenzymes 
c. Inadequate perfusion of the myocardium 
d. Rapid vasodilation of the coronary arteries

27. A male client who is hospitalized following a myocardial infarction asks the
nurse why he is receiving morphine. The nurse replies that morphine;

a. Dilates coronary blood vessels


b. Relieve pain and prevents shock
c. Helps prevent fibrillation of the heart
d. Decreases anxiety and restlessness

28. Several days following surgery a client develops pyrexia. The nurse should
monitor the client for other adaptations related to the pyrexia including:

a. Dyspnea
b. Chest pain
c. Increased pulse rate 
d. Elevated blood pressure

29. The nurse recognizes that a pacemaker is indicated when a client is


experiencing; 

a. Angina
b. Chest pain
c. Heart block 
d. Tachycardia

30. When assessing a client with a diagnosis of left ventricular failure


(congestive heart failure), the nurse should expect to find:

a. Crushing chest pain


b. Dyspnea on exertion
c. Jugular vein distention
d. Extensive peripheral edema

Situation 4: In the recall of the fluids and electrolytes, the nurse should be able to
understand the calculations and other conditions related to loss or retention.

31. After a Whippie procedure for cancer of the pancreas, a client is to receive
the following intravenous (IV) fluids over 24 hours; 1000 ml D5W; 0.5 liter
normal saline; 1500 ml D5NS. In addition, an antibiotic piggyback in 50 ml
D5W is ordered every 8 hours. The nurse calculates that the clients IV fluid
intake Tor 24 hours will be:

a. 3150ml
b. 3200 ml
c. 3650 ml 
d. 3750ml

32. The dietary practice that will help a client reduce the dietary intake of
sodium is

a. Increasing the use of dairy products 


b. Using an artificial sweetener in coffee 
c. Avoiding the use of carbonated beverages
d. Using catsup for cooking and flavoring foods

33. When evaluating a client's response to fluid replacement therapy, the


observation that indicates adequate tissue perfusion to vital organ is;

a. Urinary output of 30 ml in an hour


b. Central venous pressure reading of 2 cm H20
c. Pulse rates of 120 and 110 in a 15- minute period
d. Blood pressure readings of 50/30 and 70/40 mm Hg within 30 minutes

34. When monitoring for hypernatremia, the nurse should assess the client
for:

a. Dry skin
b. Confusion 
c. Tachycardia 
d. Pale coloring

35. Serum albumin Is to be administered intravenously to client with ascites,


The expected outcome of this treatment will be a decrease in:
a. Urinary output
b. Abdominal girth
c. Serum ammonia level
d. Hepatic encephalopathy

36. A client with a history of cardiac dysrhythmias is admitted to the hospital


with the diagnosis of dehydration. The nurse should anticipate that the
physician will order; 

a. A glass of water every hour until hydrated 


b. Small frequent intake of juices, broth, or milk
c. Short-term NG replacement of fluids and nutrients 
d. A rapid IV infusion of an electrolyte and glucose solution

37.The nurse, in assessing the adequacy of a client's fluid replacement during


the first 2 to 3 days following full-thickness burns to the trunk and right thigh,
would be aware that the most significant data would be obtained from
recording

a. Weights every day .


b. Urinary output every hour 
c. Blood pressure every 15 minutes
d. Extent of peripheral edema every 4 hours

38. A client with ascites has a paracentesis, and 1500 ml of fluid is removed.
Immediately following the procedure it is most important for the nurse to
observe for:

a. A rapid, thready pulse 


b. Decreased peristalsis .
c. Respiratory congestion
d. An increased in temperature

39. The nurse is aware that the shift of body fluids associated with the
intravenous administration of albumin occurs by the process of:

a. Filtration
b. Diffusion
c. Osmosis
d. Active Transport

40. A client's IV fluid orders for 24 hour's are 1500 ml D5W followed by 1250
ml of NS. The IV tubing has a drop factor of 15 gtt/ml. To administer the
required fluids the nurse should set the drip rate at; 

a. 13 gtt/min
b. 16 gtt/min
c. 29 gtt/min
d. 32 gtt/min

Situation 5: Protection of self and patient can be done by supporting the body's
immunity.

41. Halfway through the administration of a unit of blood, a client complains of


lumbar pain. The nurse should:

a. Obtain vita! signs


b. Stop the transfusion
c. Assess the pain further 
d. Increase the flow of normal saline

42.A client comes to the clinic complaining of weight loss, fatigue, and a low-
grade fever. Physical examination reveals a slight enlargement of the cervical
lymph nodes. To assess possible causes for the fever, it would be most
appropriate for the nurse to initially ask:

a. "Have you bee sexually active lately?"


b, "Do you have a sore throat at the present time?"
c. "Have you been exposed recently to anyone with an infection?"
d. "When did you first notice that your temperature had gone up?"

43. The nursing staff has a team conference on AIDS and discusses the routes
of transmission of the human immunodeficiency virus (HSV). The discussion
reveals that an individual has no risk of exposure to HIV when that individual;

a. Has intercourse with just the spouse


b. Makes a donation of a pint of whole blood 
c. Limits sexual contact to those without HIV antibodies
d. Uses a'condom each time there is a sexual intercourse

44. The knows that a positive diagnosis for HIV infection is made based on;

a. A history of high-risk sexual behaviors


b. Positive ELISA and Western blot tests
c. Evidence of extreme weight loss and high fever
d. Identification of an associated opportunistic infection
45. When taking the blood pressure of a client who has AIDS the nurse must;

a. Wear dean gloves


b. Use barrier techniques 
c. Wear a mask and gown 
d. Wash the hands thoroughly

46. The nurse should plan to teach the client with pancytopenia caused by a
chemotherapy to; 

a. Begin a program of aggressive, strict mouth care


b. Avoid traumatic injuries and exposure to any infection 
c. increase oral fluid intake to a minimum of 3000 ml daily 
d. Report any unusual muscle cramps or tingling sensations in the extremities

47. An elderly client develops severe bone barrow depression from


chemotheraphy for cancer of the prostate. The nurse should;

a. Monitor for signs of alopecia


b. Increase dally intake of fluids 
c. Monitor Intake and output of fluids 
d. Use a soft toothbrush for oral hygiene

48. A tuberculin skin test with purified protein derivative (PP!) tuberculin is
performed as part of a routine physical examination. The nurse should instruct
the client to make an appointment so the test can be read in:

a. 3 days
b. 5 days
c. 7 days
d. 10 days

49.A client is admitted with cellulites of the left teg a temperature of 103°F.
The physician orders IV antibiotics. Before instituting this therapy, the nurse
should;

a. Determine whether the client has allergies


b. Apply a warm, moist dressing over the area
c. Measure the amount of swelling in the client's leg
d. Obtain the results of the culture and sensitivity tests

50. Following multiple bee stings, a client has an anaphylactic reaction. The
nurse is aware that the symptoms the client is experiencing are caused by;
a. Respiratory depression and cardiac standstill 
b. bronchial constriction and decreased peripheral resistance 
c. Decreased cardiac out and dilation of major biood vessels 
d. Constriction of capillaries and decreased peripheral circulation

Situation 6: Following these diagnostic tests, Mr. Mangoni's physical discussed possible
therapies with him. It was decided that a partial gastrectomy, vagotomy, and
gastrojejunostomy would be performed.

51. Mr. Mangoni asks why the vagotomy is being done. You explain that a
vagotomy is done in conjunction with a subtotal gastrectomy because the
vagus nerve:

a. Stimulates increased gastric motility.


b. Decreases gastric motiiity, thereby preventing the movement of HCl out of the
stomach.
c. Stimulates both increased gastric secretion and gastric motiiity. 
d. Stimulates decreased gastric secretion, thereby increasing nausea and vomiting.

52. Which of the following nursing interventions would be included. in the


preoperative period for Mr. Mangoni?

a. Insertion of a nasogastric tube on the morning of surgery.


b. Administration of Vallum 4 mg with 4 oz water 1 hour before surgery.
c. Detailed description of the possible complications that could happen postoperatively
d. Instructions to avoid taking pain medication too frequently in the first 2
postoperative days to avoid drug dependency.

53. Which of the following complications, would you primarily anticipate in Mr.
Mangoni's postoperative period?

a. Thrombophlebitis from decreased mobility. 


b. Abdominal distention due to air swallowing
c. Atelectasis due to shallow breathing
d. Urinary retention due to prolonged use of antichoiinergic medications.

54. The nurse would recognize drainage from the nasogastric tube after
surgery as abnormal If:

a. It after 6 hours
b. It continued for a period greater than 12 hours.
c. ft turned greenish yeiiow in less than 24 hours.
d. It was dark red in the immediate postoperative period.
55. Which of the following statements would the nurse include in teaching
regarding nasogastric tubes?

a. Nasogastric tubes should be irrigated with sterile water. 


b. Client should be in sitting position with head slightly flexed for tube Insertion
c. When resistance is met while irrigating a nasogastric tube, pressure should be
increased to complete that irrigation, and the physician should be notified at the
completion. d. Ice chips- can be taken as often as desired to promote comfort in the
throat. 

56. The nurse must observe for which of the following imbalances to occur
with prolonged nasogastric suctioning? 

a. Hypernatremia 
b. Hyperkalemia
c. Metabolic alkalosis
d. Hypoproteinemia

57. Of the following mouth care measures by the nurse, which one should be
used with caution when a client has a nasogastric tube?

a. Regularly brushing teeth and tongue with soft brush.


b. Sucking on ice chips to relieve dryness.
c. Occasionally rinsing mouth with a nonastringent substance and massaging gums. 
d. Application of lemon juice and glycerine swabs to the lips.

58. The nurse tells Mr. Mangoni that the nasogastric tube will be removed:

a. Standardly on the fourth postoperative day. 


b. When bowel sounds are established and the client has passed flatus or Stool
c. Thirty-six hours after the cessation of bloody drainage. 
d. After 2 days of alternate clamping and unclamping of the tube.

59. Following surgery the nurse must observe for signs of pernicious anemia,
which may be a problem after gastrectomy because:

a. The extrinsic factor is produced In the stomach.


b. The extrinsic factor is absorbed in the antral portion of the stomach.
c. The intrinsic factor Is produced in the stomach.
d. Decreased hydrochloric acid production Inhibits vitamin B12 reabsorption.

60.The nurse will usually ambulate the post gastrectomy patient beginning;
a. The day after surgery 
b. Three to four days after surgery
c. After 4 days bedrest 
d. immediately upon awakening .

Situation 7: Donald Lee, a 70-year-old retired businessman, went .to his


ophthalmologist wilt's complaints of decreasing peripheral vision. Tonometry revealed
increased intraocular pressures. Mr. Lee was admitted to the hospital with a diagnosis
of open-angle glaucoma.

61. The signs and symptoms of open-angle glaucoma are related to:

a. An imbalance between the rats of secretion of intraocular fluids and the rate of
absorption of aqueous humor. 
b. A degenerative disease characterized by narrowing of the arterioles of the retina and
areas of ischemia.
c. An infectious process that causes clouding and scarring of the cornea. 
d. A dysfunction of aging in which the retina of the eye buckles from inadequate fluid
pressures. .

62. Assessment of the intraocular pressure as measured by tonometry would


be normal if the value is in the range; 

a. 5-10 mm Hg 
b. 12-22 mm Hg 
c. 10-20 cm H20 
d. 20-30 mm Hg 

63. While taking Mr. Lee's history, the nurse would be alerted to a sudden
increase in intraocular pressure if he complained of;

a. Generalized decrease in peripheral vision over the past year. 


b. Difficulty with close vision. 
c. increasing discomfort in the left eye with radiation to his forehead and left
temple. 
d. Halos around lights.

64. Client teaching about glaucoma should include a comparison of the two
types. Open-angle, or chronic, glaucoma differs from close-angle, or acute,
glaucoma in, that

a. Open-angle glaucoma occurs less frequently than closed-angle glaucoma. 


b. Open-angle glaucoma's symptomatology Includes pain, severe headache, nausea,
and vomiting; whereas closed-angle glaucoma has a slow, silent, and generally painless
onset. 
c. The obstruction to aqueous flow In open-angle glaucoma generally occurs
somewhere in Schlemm's canal or aqueous veins. It does not narrow or close the angle
of the anterior chamber, as in closed-angle glaucoma. 
d. Open-angle glaucoma rarely occurs in families; however, there is a heredity
predisposition for closed-angle glaucoma.

65. Piiocarpine is the drug of choice in the treatment of open-angle glaucoma.


The expected outcome following administration would be:

a. Blocked action of cholinesterase at the cholinergic nerve endings, and therefore


increased pupil size. 
b. Constricted pupil and therefore widened outflow channels and increased flow of
aqueous fluid.
c. Impaired vision from decreased aqueous humor production. 
d. Constriction of aqueous veins and therefore decreased venous pooling in the eye.

66. Bedrest is ordered for Mr, Lee because activity tends to increase
intraocular pressure. Which of the following activities of daily living should he
be instructed to avoid?

a. Watching television
b. Brushing teeth and hair 
c. Seif-feeding 
d. Passive range-of-motion exercises

67. To correctly instill pilocarpine in Mr. Lee's eyes, the nurse should gently
pull down the lower lid of the eye and instill the drop:

a. Dirediy on the central surface of the cornea 


b. On the inner canthus of the eye 
c. into the conjunctive sac 
d. Directly on the dilated pupil

68. Which of the following aspects of open angle glaucoma and its medical
treatment is the most frequent cause of client noncompliance?

a. Loss of mobility due to severe-driving restrictions


b. The painful insidious progression of this type of glaucoma. 
c. Decreased light and near-vision accommodation due to miotic effects of pilocarpine. 
d. The frequent nausea and vomiting accompanying use of miotic drugs.

Situation 8: Gladys Meeker is a 30-year-oid advertising executive with a history of


ulcerative colitis since age 22. Her chief complaint is severe abdominal cramping and
18- 20 stools per day for four days.

69. Blood and fluid loss from frequent diarrhea may cause hypovolemia. You
can quickly assess volume depletion In Miss Meeker by;

a. Measuring the quantity and speciflc gravity of her urine output 


b. Taking her blood pressure first supine, then sitting, noting any changes. 
c. Comparing the client's present weight with her weight on her last admission. 
d. Administering the oral water test.

70. The nurse would recognize other signs of hypovolemia, which include:

a. Dry mucous membranes and soft eyeballs.


b. Decreased hematocrit and hemoglobin
c. Decreased pulse rate and widened pulse pressure. 
d, Dyspnea and crackles.

71. With severe diarrhea, electrolytes as well as fluid are lost. The nurse
would conclude that the client is experiencing hypokalemia if which of the
following were observed?

a. Spasms, diarrhea, irregular pulse.


b. Kussmaul breathing, thirst, furrowed tongue.
c. Apathy, weakness, GI disturbance

72. Three days after admission Ms. Meeker continued to have frequent stools.
Her oral intake of both fluids and solids was poor. Her physician ordered
parenteral hyperalimentation. While administering the ordered solution, It is
important to remember that hyperalimentation solutions are: 

a. Hypotonic solutions used primarily for hydration when hemoconcentration is


present. 
b. Hypertonic solutions used primarily to increase osmotic pressure of blood plasma. 
c. Alkalizing solutions used to treat metabolic acidosis, thus reducing cellular swelling.
d. Hyperosmoiar solutions used primarily to reverse negative nitrogen balance.

73.Maintaining the infusion rate of hyperalimentation solutions is a nursing


responsibility. What side effects from too rapid an infusion rate would the
nurse expect Ms. Meeker to demonstrate?

a. Cellular dehydration and potassium depletion


b. Circulatory overload and hypoglycemia.
c. Hypoglycemia and hypovolemia.
d. Potassium excess and congestive heart failure.
74.Which of the following statements is correct regarding nursing care of Ms.
Meeker while she is receiving hyperlimentation?

a. The client's urine should be tested for glucoseacetone every 8-12 hours. 
b. The hyperlimentation subclavian line may be utilized for CVP readings and/or blood
withdrawal. 
c. Occlusive dressings at the catheter insertion site are changed every 48 hours using
the clean technique. 
d. Records of intake and output and daily weights should be kept. .

Situation 9: After 10 days of therapy, Ms. Meeker's physician decided to perform an


iieostomy. For 3 days prior to surgery she was given neomycin. On the morning of
surgery she catheterized and nasogastric tube was inserted.

75. Neomycin was administered by the nurse prior to surgery:

a. To decrease the incidence of postoperative atelectasis due to decreased depth of


respirations. 
b. To increase the effectiveness of the body's immunologic response following surgical
trauma. 
c. To reduce the incidence of wound infections by decreasing the number of intestinal
organisms. 
d. To prevent postoperative bladder atony due to catheterization.

76. Following iieostomy, the nurse would expect the drainage appliance to be
applied to the stoma;

a. 24 hours later, when edema has subsided.


b. In the operating room.
c. After the ileostomy begins to function.
d. When the client is able to begin self-care procedures.

77.Which of the goals would be described to Ms. Meeker as the highest


postoperative nursing priority?

a. Relief of pain to promote rest and relaxation.


b. Assisting the client with self-care activities.
c. Maintenance of fluid, electrolyte, and nutritional balances.
d. Skin care and control of odors.

78. During the early postoperative period, the nurse initiates ileostomy
teaching with Ms. Meeker. The primary objective of this procedure is;
a. To facilitate maintenance of intake and output records
b. To control unpleasant odors. 
c. To prevent excoriation of the skin around the stoma. 
d. To reduce [he risk of postoperative wound infection.

79. After discharge, Ms. Meeker calls you at the hospital to report the sudden
onset of abdominal cramps, vomiting, and watery discharge from her
iieostomy. What would you advise?

a. Call the physician if symptoms persist for 24 hours.


b, Take 30 cc of m.o.m. (milk of magnesia).
c. NPO until vomiting stops.
d. Call the physician immediately.

Situation 10: Joseph Clifford, age 38, has extensive bums over much of his trunk and
arms. He complains of intense pain during wound cleansing, dressing change,
debridement, and physical therapy.

80. This pain most likely is related to:

a. Thermal stimulation
b. Menta! stimulation 
c. Mechanical stimulation 
d. Chemical stimulation

81. Mr. Clifford dreads physical therapy and resists activity; he has difficulty
sleeping due to pain and fatigue after the treatments. He lacks appetite for
food or fluid. Based on this information, his priority nursing diagnosis would
be:

a. Activity Intolerance related to pain secondary to bums. 


b. Altered Nutrition; Less Than Body Requirements related to pain secondary to bums.
c. Sleep Pattern Disturbance reiated to pain secondary to bums. 
d. Pain related to bums.

82. Mr. Clifford continues to experience significant pain after his expensive
bum wounds have healed - 6 months after his injury. He also expresses
concern over possible loss of job and disfigurement. At this; stage, the nurse
can most effectively intervene for his pain by:

a. Referring him for his counseling and occupational therapy. 


b. Staying with him as much as possible and building trust
c. Providing cutaneous stimulation and pharmacoiogic therapy. 
d. Providing distraction and guided imagery.
83. Eventually, Mr- Clifford's chronic pain and anxiety about his appearance
did contribute to his losing his job and disrupting his plans for marriage. 

He finally heeded the nurse's recommendation and sought treatment at a pain


center, after which his pain subsided and he permitted his former fiancee to
participate in his rehabilitation process, including looking for a new job. 

Evaluation criteria for Mr. Clifford's successful rehabilitation should include


which of the following:

a. The patient has no aftermath phase of his pain experience. 


b. The patient experiences decreased frequency of acute pain episodes. 
c. The patient continues normal growth and development with his support systems
intact. 
d. The patient develops increased tolerance for severe pain in the future.

84. Which of the following statements regarding pain is incorrect? 

a. intractable pain may not be relieved by treatment. 


b. Pain is an objective sign of a more serious problem. 
c. Psychologic factors can contribute to a patient's pain perception. 
d. Pain sensation is affected by a patient's anticipation of pain.

85. Billy Bragg, aged 5, received a small paper cut on his finger. His mother
left him wash it and apply a smail amount of bacitracin and a Band-aid. She
then let him watch TV and eat an apple Her intervention for pain are examples
of:

a. Providing pharmacologic therapy


b. Providing control and distraction
c. Altering Billy's environment
d. Providing cutaneous stimulation

Situation 11: Mrs. Smith, age 64, has been diagnosed with COPD. Although she was
hospitalized several times in the last year for acute respiratory failure, she is presently
in stable condition.

86. The primary focus of care in the long-term nursing care for Mrs. Smith
would be to:

a. Decrease activity to conserve functional Sung tissue.


b. Increase the frequency of postural drainage to every 2 hours he awake.
c. Increase the RV.
d. improve and maintain pulmonary ventilation and gas exchange.

87. Mrs. Smith's condition has changed over a period of days,, and her arterial
blood studies now indicate she is again in acute respiratory failure. The
primary nursing intervention most commonly required .in the care of patient
with COPD who are in acute respiratory failure is to:

a. Establish initial stage of activity. 


b. Discourage patient from sitting in Fowler's position in order to reduce work of heart.
c. Remove bronchia! secretions, and manage oxygen therapy. 
d. Plan with family for home care.

88. Mrs. Smith has been treated aggressively for acute respiratory failure and
has improved over the past four weeks. She experienced anxiety about being
prepared for discharge. The nurse who cares for her should help her develop
ways to cope with her chronic obstructive lung disease by:

a. Encouraging the family to take increased responsibility for the patients care. 
b. Discouraging the patient from performing activities of daily living if they make her
tired. 
c. Teaching the patient relaxation techniques and breathing refraining exercises. 
d. Protecting the patient from knowing the prognosis of her disease.

Situation 12: Mrs. Lippett, age 66, is experiencing sensory and perceptual problems
that affect her right visual field (right homonymous hemianopia).

89. When placing a meal tray in front of Mrs. Lippett, the nurse should;

a. Place all the food on the right side of the tray.


b. Before leaving the room, remind the patient to look over all the tray.
c. Place food and utensils within the patient's left visual field.
d. Stay with the patient & periodically draw her attention of the food on the right side
of the tray to prevent unilateral neglect

90. The nurse should include which of the following in preprocedure teaching
for a patient scheduled for carotid angiography?

a. "You will be put to sleep before the needle Is inserted." 


b. "The test will take several hours." 
c. "You may fee! a burning sensation when the dye is injected." 
d. "There will be no complications."

91.What deficits would the nurse expect in a right-handed person


experiencing a stroke affecting the left side of the cortex?
a. Expressive aphasia and paralysis on the right side of the body. 
b. Expressive aphasia and paralysis on the left side of the body. . 
c. Dysarthria and paralysis on the right side of the body. 
d. Mixed aphasia and paralysis on the right side of the body.

92. What would be the most appropriate intervention for a patient with
aphasia who state, "I want a ..." and then stops?

a. Wait for the patient to complete the sentence.


b. Immediately begin showing the patient various objects In the environment.
c. Leave the room and come back later.
d. Begin naming various objects that the patient could be referring to.

93. Which of the following statements would be most appropriate when


assisting a patient who has the nursing diagnosis ofAltered Thought Process
with Persona! Hygiene Needs?

a. "What would you like to do first, brush your teeth?" 


b. "Where is y our toothbrush?"
c. "When would you like to have your bath?"
d. "Would you like to brush your teeth, or do you want me to do it for you? it's good to
do things for yourself."

94. Which of the following positions would be most appropriate for a patient
with right-sided paralysis following a stroke?

a. On the side with support to the back, with pillows to keep the body in alignment,
hips slightly flexed, and hands tightly holding a rolled washcloth.
b. On the side with support to the back, pillows to keep the body in alignment, hips
slightly flexed, and a washcloth placed so that fingers are slightly curled.
c. On the back with two large pillows under the head, pillow under" the knees, and a
footboard. 
d. On the back with no pillows used, with trochanter rolls and a footboard.

95.To prevent infection in a patient with a subdura! intracranial pressure


monitoring system in place, the nurse should;

a. Use aseptic technique for the insertion site.


b. Use clean technique for cleansing connections and aseptic technique for the insertion
site.
c. Use sterile technique when cleansing the insertion site 
d. Close any leaks in the tubing with tape.
Situation 13: Mrs. Taylor, age 74, suffers from degenerative joint disease due to
osteoarthritis and is admitted for a total joint replacement of the right hip.

96. During the preoperative period, the nurse should focus assessment
primarily on:

a. Local and systemic infections


b. Self-care ability
c. Response to pain medications
d. Range of motion in the affected joint

97. Following arthroplasty, the nurse should maintain correct position of Mrs,
Taylor's operative leg by:

a. Placing an abductor wedge or pillows between the legs. 


b. Placing sandbags or pillows to Keep leg abducted. 
c. Elevating the affected leg on two pillows or supports. 
d. Positioning her supine and on the operative side.

98. When discussing physical activities with Mrs. Tayior, the nurse should
instruct her to;

a. Avoid weight bearing until the hip is completely heated. 


b. Intermittently cross and uncross legs several times daily. 
c. Maintain hip flexion at 90 degrees when sitting. 
d. Limit hip flexion to only 45 to 50 degrees.

99. Before discharge, the nurse reviews the signs and symptoms of joint
dislocation with Mrs. Tayior. The nurse would determine that Mrs. Taylor
understands the instructions by her identification of which of the following
symptoms? 

a. Positive Homan's sign and Inability to bear weight. 


b. Painiess, sudden deformity of the affected hip joint. 
c. Severe hip pain with shortening of the extremity. 
d. Severe pain and swelling of the affected hip joint.

100. As part of treatment of gouty arthritis for Mrs. Martin, age 66, the
physician orders antiuric acid medication to be given in large doses until the
maximum safe dosage can be determined. The nurse would determine the
maximum dosage and the need for dosage reduction by asking Mrs. Martin to
report which of the following symptoms?

a. Bleeding gums and bruising


b. Nausea, vomiting, and diarrhea
c. Gastric irritation and heartburn
d. Blurred vision and nausea
Situation 1 A client is brought into the emergency department with brain stem
contusion

1. Two days after the admission, the client has a large amount of urine and a
serum sodium level of 155 mEq/dl. Which, of the following conditions may be
developing? 

a. Myxedema coma
b. Diabetic insipidus
c. Type 1 diabetes mellitus
d. Syndrome of inappropriate ant-diuretic Hormone secretion 

2. After a thorough assessment and laboratory works shall shows serum


ketones and serum glucose level above 300mg/dl, what condition would be
diagnosed to patient? 

a. Diabetes insipidus
b. Diabetes ketoacidosis 
c. Hypoglycemia
d. Somogyi phenomena

3. Which of the following combinations of adverse effects must be carefully


monitored when administering I.V. insulin to a client diagnosed with diabetic
ketoacidosis?

a. Hypokalemia and hypoglycemia 


b. Hypocalcemia and Hyperkalemia 
c. Hyperkalemia and hyperglycemia 
d. Hypernatremia and hypercaleemia

4. Which of the following method of insulin administration would be used in


the initial treatment of hyperglycemia in a client with diabetic ketoacidosis.

a. Subcutaneous
b. Intramuscular
c. I.V bolus only
d. I.V. bolus followed by continuous infusion

5. Hyperosmolar hyperglycemic nonketotic syndrome (HHNS) can be


differentiated from diabetic ketoacidosis by which of the following conditions?

a. Hyperglycemia
b. Serum osmolarity
c. Absence of ketosis 
d. Hypokalemia

Situation 2: Mr. Reynaldo Layag executive officer, was brought to the hospital because
of chest pain-Diagnosis of angina was established.

6. Mr. Layag state that his anginal pain increases after activity. The nurse
should realize that the angina pectoris is a sign of: 

a. Mitral insufficiency 
b. Myocardial infraction 
c. Myocardial ischemia 
d. Coronary thrombosis

7. Nitroglycerine S.L. is prescribed for Mr. Layag'a anginal pain. When teaching
how to use nitroglycerine, the nurse tells him to place 1 tablet under the
tongue when pain occurs and to repeat the dose in 5 minutes if pain persist.
The nurse should tell Mr. Layag to:

a. Place two tablets under the tongue when the intense pain occurs 
b. Swallow 1 tablet and place 1 tablet under the tongue when pain is intense 
c. Place 1 tablet under the tongue 3 minutes before activity and repeat the dose in 5
minutes if pain occurs 
d. Place 1 tablet under the tongue when pain occurs and use an additional tablet after
the attack to prevent reoccurrence 

8. The nurse realizes that the pain associated with coronary occlusion is
caused primarily by:

a. Arterial Spasm 
b. Ischemia of the heart muscle 
c. Blocking of the coronary veins
d. Irritation of the nerve endings in the cardiac plexus

9. When cardiovascular disease is concern, reduction of the saturated fat in


the diet may be desired and substance made of polyunsaturated fat When
teaching about this diet the nurse should instruct Mr. Layag to avoid :

a. Fish
b. Corn Oil
c. Whole milk
d. soft margarine

10. When teaching Mr. Layag, who has been placed on a high-unsaturated
fatty acid diet, the nurse should stress the importance of increasing the intake
of:

a. Enriched whole milk


b. Red meats, such as beef
c. Vegetables and Whole Grains
d. Liver and other glandular organ meals

Situation 3: A group of nursing students were discussing the normal growth and
development concepts when assigned to observe the school children.

11. During the oedipal stage of growth and development, the child: 

a. Loves and hates ( ambivalence) both parents 


b. Loves the parent of the same sex and the parent of the opposite sex 
c. Loves the parent of the opposite sex and hates the parent of the same sex 
d. Love the parent of the same sex and hates the parent of the opposite sex.

12. The stage of growth and development basically concerned with the role
identification is the:

a. Oral Stage 
b. Genital-Stage
c. Oedipal Stage
d. Latency stage 

13. Play for the preschool-age child is necessary for the emotional
development of:

a. Projection
b. Introjection 
c. Competition
d. Independence

14. Resolution of the oedipal complex takes place when the child overcomes
the castration complex and:

a. Rejects the parent of the same sex


b. Introjects behavior of both parents
c. Identities with me parent of the same sex
d. Identifies with the parent of the opposite sex

15. Any surgery should be delayed, if possible, because of me effects on


personality development during the 
a. Oral Stage. 
b. Anal Stage 
c. Oedipal Stage
d. Latency Stage 

Situation 4 - Transurethral resection prostatectomy, (TURP) is performed to Mr. Recto,


60 years old, due to prostate enlargement. Post operatively he has continuous irrigation
(Cystoclysis).

16. Which of these statements explain the reason for continuous bladder
irrigation?

a. To remove clot from the bladder 


b. To maintain the patency of the catheter 
c. To maintain the patency of me bladder 
d. To dilute urine

17. Nursing assessment is vital to prevent and detect indications of


postoperative complications. The following are the possible complications
after prostatectomy except:

a. Residual urine 
b. Urethral structure 
c. Erectile dysfunction 
d. The drainage has stopped

18. When should the nurse increase, the flow rate of cystoclysis of Mr.Recto? 

a. The drainage appear cloudy 


b. The drainage is continuous but slow
c. The drainage is bright red 
d. The drainage has stopped 

19. After the removal of the three way catheter, the nurse should inform Mr.
Recto that he can normally experience:

a. Dribbling incontinence 
b. Polyuria 
c. Dysuria
d. The drainage has stopped

20. Which of the following measures should you encourage Mr. Recto to do, in
order to regain urinary control?
a. Wear scrotal support 
b. Take warm bath 2 times daily 
c. Ambulate frequently
d. Alternately tense and relax the perineal muscles 

Situation 5 - Nurses are generalist, in order to cope up with the works demand you
must have broad knowledge on anything. Nurse Joan was assigned in the medical ward.
During the endorsement she found out that she was assigned to several patients of
different case

21. When developing a teaching session on glaucoma for the community,


which of the following statements would the nurse stress

a. Glaucoma is easily corrected with eye glasses 


b. White and Asian individuals are the highest risk of glaucoma 
c. Yearly screening for people ages 20 to 40 years is recommended 
d. Glaucoma can be painless and visions may be lost before the person is aware of the
problem.

22. Which of the conditions is an early symptoms cgmmonly seen in


Myasthenia Gravis?

a. Dysphagia 
b. Fatigue improving at the end of the day 
c. Ptosis 
d. Respiratory Distress

23. Which of the following statements best describes the Parkinson's Disease?

a. Loss of myelin sheath surrounding peripheral nerves 


b. Degeneration of the substantia nigra; depleting dopamine
c. Bleeding into the brain stem, resulting in meter dysfunction 
d. An autoimmune disorder that destroys acetlycholine receptors

24. Which of the following pathophysiological processes are involved in


multiple sclerosis (MS)? 

a. Destruction of the brain stem and basal ganglia in the brain 


b. Degeneration of the nucleus pulposus, causing pressure on the spinal cord
c. Chronic inflammation of rhizomes just outside the central nervous system
d. Development of demyelinization of the myelin sheath, interfering with the nerve
transmission

25. When teaching the client, with Meniere's disease, which of the following
instructions would a nurse give about vertigo.

a. Report dizziness at once


b. Drive in daylight hours only
c. Get up slowly, turning the entire body
d. Change your position using the logroll technique

Situatitm 6 - Mr. Punsalan is 36 years old, was admitted to the hospital with complaints
of a burning sensation in the epigastric area after eating and inability to sleep at night.
He was placed on bed rest and schedule for diagnostic studies. A diagnosis of Peptic
Ulcer was made.

26. Mr. Punsalan with gastric pain is advised to take any one of the following
antacids, except:

a. Aluminum hydroxide 
b. Calcium bicarbonate 
c. Magnesium carbonate
d. Sodium bicarbonate

27. An occult blood examination was ordered. The specific specimen needed
from Mr. Punsalan is;

a. Stool
b. Blood 
c. Sputum 
d. Gastric juice

28. Preparation of Mr. Punsalan for occult blood examination is :

a. Fluid intake is limited only 1 liter/day 


b. NPO for 12 hours prior to obtaining of specimen 
c. Fluid intake is increased 
d. Meatless diet for 48 hours prior to obtaining of specimen 

29. X -ray examination for Mr. Punsalan to detect tumors or ulcerations of the
stomach and duodenum is:

a. Gastroscopy 
b. GIT series 
c. Cold G.I. series
d. Ba enema

30. Diet that prevents gastric irritation in case of Mr. Punsalan is:
a. Bland Diet 
b. Liquid Diet
c. Full Diet
d. High Protein low fat diet

Situation 7 - Mr. Reyes suffered head injuries in a motor vehicle accident

31. When caring for Mr. Reyes, the nurse should assess for

a. Decreased carotid pulses 


b. Bleeding from oral cavity 
c. Altered level of consciousness
d. Absence of deep tendon-reflexes

32. Mr. Reyes is extremely confused. The nurse provide new information
slowly and in small amounts because;

a. Confusion or delirium can be a defense against further stress


b. Destruction of brain cells has occurred, interrupting mental activity
c. Teaching based on information progressing from the simple to the complex
d. A minimum of information should be given, since he is unaware of surroundings

33. Mr. Reyea complains of hearing ringing noises. The nurse recognizes that
this assessment suggests injury of the

a. Frontal lobe 
b. Occipital lobe
c. Six cranial nerve (abducent)
d. Eight Cranial Nerve (Vestibulocochlear)

34. Mr. Reyes has a possible skull fracture. The nurse should:

a. Observe him for signs of Brain injury 


b. Check for hemorrhaging from the oral cavity 
c. Elevate the foot of the bed if he develops symptoms of shock 
d. Observe for symptoms of decreased intracranial pressure and temperature

35. Mr. Reyea has expressive aphasia. As a part of a long range planning. The
nurse should ;

a. Provide positive feedback when he uses the word correctly 


b. Wait for him to verbally state needs regardless of how long it may take c. Suggest
that he get help at home because the disability is permanent 
d. Help the family to accept the fact that Mr, Reyes cannot participate in verbal
communication

Situation 8 - Patricia Zeno is a client with history myasthenia gravis.

36. Clients with myastherda gravis, Guillain - Barre Syndrome or amyothrophic


sclerosis experience: 

a. Progressive deterioration until death 


b. Increased risk of respiratory complications 
c. Deficiencies of essential neurotransmitter 
d. Involuntary twitching of small muscle groups

37. Myasthenia gravis most frequently affect:

a. Males ages 15 to 3 5 years 


b. Children ages 5 to 15 years
c. Female ages 10 to 30 years old
d. Both sexes ages 20 to 40 years

38. Mrs. Zeno asks the nurse why the disease has occurred. The nurse bases
the reply on the knowledge that there is:

a. A genetic defect in the production of acetylcholine


b. A reduced amount of neurotransmitter acetylcholine
c. A decreased number of functioning acetyl-choline receptor sites
d. An inhibition of the enzyme Ache leaving the end plates folded.

39. To provide safe care for Mrs. Zeno, it is important for the nurse to check
the bedside for the presence of:

a. A tracheostomy set 


b. An intravenous set-up 
c. A hypothermia blanket 
d. A syringe and edrophonium HCl(Tensilon)

40. Mrs. Zeno continues to become a weaker despite .treatment with


neostigmine. Edrophonium HCL is ordered:

a. For its synergestic effect 


b. To rule out cholinergic crisis 
c. To confirm the diagnosis of myasthenia 
d. Because of the client's resistance to Neostigmine

Situation 9 - Hariet, a 38 year-old school teacher with rheumatoid arthritis, is admitted


to the hospital with severe and swelling of the joints of both hands.

41. A regimen of rest, exercises and physical therapy is ordered for Hariet This
regimen will;

a. Prevent arthritic pain


b. Halt me inflammatory process
c. Help prevent the drippling effects of the disease
d. Provide for the return of joint motion after prolonged loss

42. Hariet ask the nurse why the physician is going to inject hydrocortisone
into her affected joint. The nurse explains that the most important reason for
doing this is to:

a. Relieve pain 
b. Reduce inflammation 
c. Provide Psychotherapy 
d. Prevent ankylosis of the joint

43. When planning nursing care for Hariet, the nurse should take into
consideration the fact that:

a. Inflammation of the synovial membrane will rarely occur 


b. Bony ankylosis of the joint is irreversible and causes immobility 
c. Complete immobility is desired during the acute phase of inflammation 
d. If the redness and swelling of a joint occur, they signify irreversible damage

44. The diet the nurse would expect the physician to order for Hariet would be:

a. Salt free and low fiber 


b. High calorie with low cholesterol 
c. High protein with minimal calcium 
d. Regular diet with vitamins and minerals 

45. The medication the nurse would expect to prescribed to relieve Hariet's
pain;

a. Xanax 0.5 mg, TID 


b. Aspirin, 0.6 g_q4 
c. Codeine , 30 mg, q4 
d. Meperidine 30 mg q4 pm

Situation 10 - Lizbeth 20 year-old college student is brought to the hospital by her


mother who states that for the past week her behavior has become very strange. She
has become more and more withdrawn - Diagnosis: Schizophrenia Catatosis. ' •

46. During the physical assessment Lizbeth's arms remains outstretched after
her pulse and blood pressure were taken and the nurse has to reposition it for
her. Lizbeth is showing; 

a. Distractability 
b. Muscle rigidity
c. Waxy flexibility 
d. Echopraxia 

47. Lizbeth keeps her eyes closed and does not answer the questions asked by
the nurse or physician. The nurse know that;

a. The patient can cannot hear nor understand what is being asked
b. The patient is aware of what is happening around her even though she does not
respond
c. The patient is in regressed state and should be treated like a frightened child
d. The patient is aware of what is going on around her and could respond if she wants
to.

48. While Lizbeth remains in an unreasonable state, does not eat or drink, the
nurse first priority id to assess her:

a. Fluid intake and output 


b. Skin turgor 
c. Bowel elimination
d. Vital signs such as T.P.R. and blood pressure

49. One evening, Lizbeth suddenly begins running up and down the hall. She
strips her clothing and strikes out widely at anyone she sees. All of the
following interventions would be appropriate except: 

a. Restrain me patient and call for help 


b. Call for the assistance of at least three staff members 
c. Clear the area of other patients
d. Obtain me order and prepare chlorpromazine (thorazine)

50. When Lizbeth become agitated, the therapeutic approach of the nurse is
one that is:

a. Authoritarian and directive 


b. Related casual and friendly 
c. Permissive and comforting
d. Calm and firm but not threatening 

Situation 11- Michelle, 36 weeks gestation visits the hospital because the suspects that
her bag of water was ruptured. -

51. While the nurse is assessing Michelle, she states that her bag of water
ruptured few minutes ago. Which of the following should the nurse do first? 

a. Check the status of the fetal heart rate 


b. Turn the client to her right side 
c. Test the leaking fluid with nitrazine paper 
d. Perform a sterile vaginal examination 

52. To confirm Michelle's statement, the nurse uses nitrazine paper; if the
membrane has ruptured the paper which of the following color?

a. Yellow 
b. Green 
c. Blue 
d. Blue

53. After being confirmed that membranes has been ruptured and there was
no evidence of labor, which of me following would the nurse expect the
physician to order? 

a. Frequent assessment of cervical dilation 


b. Intravenous oxytocin adminitration 
c. Vaginal culture for Neisseria Gonorrhoeae 
d. Sonogram for amniotic fluid volume index

54. Few hours after, the nurse noted that her cervix is 2 cm dilated and 50%
effaced. Which of the following would the priority assessment for this client? 

a. Red blood cell count 


b. Degree of Discomfort 
c. Urinary Output 
d. Temperature 

55. Michelle is to be discharged home on bed rest with follow -tip by the
community health nurse. After instruction about care while at home, which of
the following client's statements indicates effective teaching? 

a. "It is permissible to douche if the fluid irritates my vaginal area."


b. " I can take either a tub Bath or a shower when I feel it"
c. "I shouldn't limit my fluid intake to less than 1 quart daily."
d. " I should contact the doctor if my temperature is 100.4 F or higher."

Situation 12 - Jerome, a 37 years old man, was admitted to the hospital with periodic
episode of manic behavior alternating with me depression. Diagnosis: Bipolar I
disorder.

56. Which of the following statements is true and manic reaction? It is;

a. An expression of destructive impulse


b. A means of coping with frustrations and disappointments
c. A Means of Ignoring reality
d. An attempt toward off feeling of underlying depression.

57. Nursing care plan for a hyperactive patient like Jerome, should give
priority to:

a. Discourage him from manipulating the staff


b. Prevent him from assaulting other patients 
c. Protect him against suicidal attempts
d. Provide adequate food and fluid intake

58. During a nurse patient interaction, Jerome jumps rapidly from one topic to
another. This is known as: 

a. Flight of Ideas
b. Idea of Reference
c. Clang association
d. Neologism

59. A priority nursing diagnosis would be

a. Ineffective individual coping


b. Altered family process
c. Potential for violence, self directed
d. Sensory perceptual disturbance

60. Initially one of the following activities would be appropriate for Jerome;

a. Playing basketball
b. Playing chess 
c. Gardening 
d. Writing
Situation 13 - .Mr,. Baldo , 36 years old patient complaints of fatigue, weight loss, and
low-grade fever. He also has pa in his fingers, elbows, and ankles.

61. Which of the following conditions is suspected?

a. Anemia
b. Leukemia
c. Rheumatic arthritis 
d. Systematic Lupus Erythematosus (SLE)

62. Systematic lupus erymematosus (SLE) primarily attacks which of the


following tissues?

a. Connective 
b. B. Heart 
c. Lung
d. Nerve

63. Which of the following elements shows that the client does not understand
the cause of exacerbation of system lupus erythematosus (SLE)?

a. " I need to stay away from sunlight"


b. "I don't have to worry if I get a strep. throat
c. I need to work on managing stress in life." 
d. "I don't have to worry about changing my diet."

64. Which of the following symptoms is a classic sign of systemic lupus


erythematosus (SLE)? 

a. Vomiting
b. Weight loss
c. Difficulty urinating
d. Superficial lesions over the cheek and nose

65. Mr Balao asks the nurse as to the source of this disease. The nurse is
aware that this is a disease of:

a. Joints
b. Bones
c. connective tissue
d. purine metabolism

Situation 14 - Mr Gil age 86 years, has been diagnosed with Alzheimer's disease.
66. Which characteristics could the nurse expect when observing Mr. Gil?

a. Transient ischemic attacks


b. Remissions & exacerbations
c. Rapid deterioration of mental functioning because of arteriosclerosis
d. Slowly progressive deficits in intellect, which may be noted for a long time

67. Mr. Gil frequently switches from being pleasant and happy to being hostile
and sad without apparent external cause. How can the nurse best care for Mr.
Gil?

a. Try to point out reality to him 


b. Avoid Mr. Gilwhen he is angry and sad
c. Encourage him to talk about his feelings
d. Attempt to give nursing care when he is in a pleasant mood

68. What type of environment should be provided by the health care team for
Mr. Gil?

a. Familiar
b. Variable
c. Challenging
d. Non-stimulating

69. Mr. Gil will need assistance in maintaining contact with society for as long
as possible. Which therapy might help him achieve this goal?

a. Psychodrama
b. Recreation therapy
c. Occupational therapy
d. Remotivation therapy

70. What is the nurse's primary objective for Mr. Gil when he is experiencing
dementia and delirium?

a. Diminished psychologic faculties


b. Interaction with the environment
c. Participation with the environment
d. Face to face contact with the other clients

Situation I5: Baby Philip, a full term male child, is delivered by his mother who is RH
negative.

71. At the time of delivery, baby Philip's blood is typed to determine the ABO
group and the presence of the RH factor. The nurse is aware that:

a. The RH factor is not genetically determined


b. Not all infants of RH-positive fathers are RH positive
c. The RH factor of the fetus is determined by the father 
d. During gestation, the RH factor of the fetus may change

72. Baby Philip is RH positive and his mother is RH negative. Baby Philip is to
receive an exchange transfusion. The nurse know that he will receive RH-
negative blood because:

a. It is me same as die mother's blood


b. It is neutral and will not react with his blood
c. It eliminates the possibility of a transfusion reaction occurring
d. His RBC's will not be destroyed by the maternal anti-RH antibodies

73. Hyperbilirubinemia is anticipated to baby Philip because of RH


incompatibility. Hyperbilirubinemia occurs with incompatibility between
mother and fetus because

a. The mother's blood does not contain the RH factor, so she produces anti-RH
antibodies that cross the placental barrier and cause hemolysis of red blood cells in
infants
b. The mother's blood contains the RH factor and the infant's does not, and antibodies
are formed in the fetus that destroy red blood cells. 
c. The mother has the history of previous yellow jaundice caused by a blood
transfusion, which was passed the fetus through the placenta. 
d. The infant develops a congenital defect shortly after birth that causes the destruction
of red blood cells.

74. If RhoGAm is given to Baby Philip's mother after delivering Baby Philip,
the condition that must be present rbr the globulin to be effective is that:

a. Philip's mother is Rh positive 


b. Baby Philip is Rh negative
c. Philip's mother has no titer in her blood
d. Philip's mother has some titer in her blood

75. When the nurse brings Philip to his mother, she comments about the milia
on the baby's face. The nurse should:

a. Tell her that all babies have them and they clear up in 2 to 3 days
b. Explain that these are birthmarks that will disappear within a few months
c. Instruct her about proper handwashing since the milia can be infectious
d. Instruct her to avoid squeezing them or attempting to wash them off

Situation 16: Ronald 23 years old was voluntarily admitted to the in-patient unit with a
diagnosis of paranoid schizophrenia.

76. As the nurse approaches Ronald he says, "If come any closer. I'll die." This
is an example of:

a. Hallucination
b. Delusion
c. Illusion
d. Idea of reference

77. The best response for the nurse to make to this behavior is:

a. "How can I hurt you?"


b. "I'm the nurse."
c. "Tell me more about this."
d. "That's a silly thing to say."

78. When communicating with the paranoid client, the main principle is to:

a. Use logic and be persistent


b. Provide an anxiety-free environment
c. Express doubt and do not argue
d. Encourage ventilation of anger

79. Ronald is pacing the hall and is agitated. The nurse hears him saying,
"Those doctors are faying to commit me to the state hospital. The nurse's
continued assessment should include:

a. Clarifying information with the doctor


b. Observing Ronald for rising anxiety
c. Reviewing history of involuntary commitment
d. Checking dosage of prescribed medication

80. An appropriate activity for the nurse is to recommend for a client who is
extremely agitated is:

a. Competitive sports
b. Bingo
c. Trivial Pursuit
d. Daily walks
Situation 17: Mrs. Lim has had confirmation of her pregnancy. She presents the
emergency room with abdominal pain not yet. diagnosed.

81. The nurse would suspect an ectopic pregnancy if Mrs Lim complained of:

a. An adherent painful ovarian mass


b. Lower abdommal cramping for a long period of time
c. Leukonhea and dysuria a few days after the first missed period
d. Sharp lower right or left abdominal pain radiating to the shoulder

82. The most common type of ectopic pregnancy is tubal. Within a few weeks
after conception the tube may rupture suddenly, causing:

a. Painless vaginal bleeding


b. Intermittent abdominal contractions
c. Continues dull, upper-quadrant abdominal pain
d. Sudden knife-like, lower-quadrant abdominal pain

83. Mrs. Lim has been complaining of vaginal bleeding and one sided lower
quadrant pain. The nurse suspects mat she has:

a. Abruptio placenta
b. An incomplete abortion
c. An ectopic pregnancy
d. A rupture of graafian follicle

84. A few hours after being admitted with a diagnosis of inevitable abortion, a
client begins to experience bearing down sensations and suddenly expels the
products of conception in bed. To give safe nursing care, the nurse should first

a. Check the fundus for firmness


b. Give her the sedation
c. Immediately notify the physician
d. take her immediately to the delivery home

85. After a spontaneous abortion the nurse should observe the client for:

a. Hemorrhage and infection


b. Dehydration and hemorrhage
c. Subiiivolution and dehydration
d. Signs of pregnancy-induced hypertension

Situation 18: Arnold, age 67, has had successfully treated depressive disease for more
than 10 years. Lately he has been developing a plan of action. Arnold is admitted to
hospital for reassessment.

86. Which assessment would best aid the nurse in evaluating Arnold's
potential for suicide?

a. Ask him about plans for the future 


b. Ask other clients about suicide while in a group 
c. Ask the family if he had ever attempted suicide
d. Ask him if suicide was ever or is now being considered

87. Which factor is most important in evaluating Arnold's risk for suicide? 

a. Presence of multiple personal problems


b. Length of time the depression has existed
c. Impending of the loss of a loved one 
c. development plans for discharge from hospital or program

88. Arnold confides to the nurse that he has been thinking of suicide. Which of
the following motivations should the nurse recognize in Arnold? 

a. Wishes to frighten the nurse 


b. Wants attention from the staff 
c. Feels safe and can share his feelings with the nurse
d. Shows fearful of his own impulses and is seeking protection from them

89. Arnold is placed on suicide precautions. Which would be the most


therapeutic way to provide his safety measures?

a. Not allow him to leave his room


b. Remove all sharp and cutting objects
c. Give him the opportunity to ventilate feelings
d. Assign staff member to be with him at all times

90. The psychiatrist prescribes Electro convulsive therapy for Arnold. The
nurse when discussing ECT with Arnold, should tell him which of the following
information?

a. Sleep will be induced and treatment will not cause pain


b. There will be a memory loss aa a result of the treatment
c. It is better not to talk about it, but he can asks any question

Situation 19: Josh is a 2-year old child who was bom with a unilateral cleft lip and
palate. He is being readmitted for a palate repair.
91. When a toddler is hospitalized, age appropriate toys would include:

a. Wind-up toys, music boxes, and electric trains


b. Toys requiring pushing, pulling and to big to be swallowed
c. Marble tracks and small blocks encouraging fine-motor coordination
d. Colorful mobiles, wind-up toys, and marble tracks

92. Which of the following would be the most important factor in preparing
Josh for his hospitalization?

a. Gratification of Josh wishes


b. Josh's previous hospitalization
c. Never leaving Josh with strangers
d. Assurance of affection and security

93. Prior to a repair of a unilateral cleft lip and palate, feeding will probably
be:

a. Limited to IV fluids
b. Wish a soft, large altered nipple
c. Accomplished per gastrostomy tube 
d. Facilitated by the use of spoon or medicine dropper

94. Which of the following nursing actions would have been included for Josh
following his cleft lip repair?

a. Using a spoon to administer oral feedings


b. Cleansing the suture line to prevent infection
c. Allowing Josh to suck on a pacifier to prevent crying 
d. Positioning Josh on the abdomen to avoid aspiration

95. Why will Josh be unable to use toothbrush postoperatively?

a. The suture line might be injured


b. Josh would probably have no teeth
c. The toothbrush might be frightening to Josh
d. Josh would not be accustomed to a brush at home

Situation 20: Vincent, age 26, who is caught in me raging conflict between his mother
and his wife, complains of pains in his right leg that has progressed to the point of
paralysis. After orthopedic consultation has shown no pathology, he is referred for a
psychiatric consultation and is found to have a conversion disorder.

96. The nurse understands which of the following concepts about Vincent's
conversion disorder? 

a. It is an unconscious method for him to cope with the present situation 


b. It is usually necessary for him to cope with the present situation 
c. It is reversible and will subside if he is helped to focus on other things 
d. It will probably be solved when he learns to deal with ongoing family conflicts

97. Vincent's conflict may be caused by which of the following stimuli? 

a. Hostile feelings towards his home


b. Ambivalent feelings toward his wife
c. Needs to be a dependent child and an independent adult
d. Inadequate feelings in regard to assuming the role of husband

98. Which behavior is Vincent most likely to manifest?

a. Demonstrate a spread of paralysis to other body parts 


b. Require continuous psychiatric treatment to maintain individual functioning 
c. Recover the use of the affected leg but under stress, again develop similar
symptoms 
d. Follow a rather unpredictable emotional course I the future, depending on exposure
to stress

99. How would the nurse expect Vincent to behave? 

a. Appear gently depressed 


b. Exhibit free floating anxiety 
c. Appear calm and composed 
d. Demonstrate anxiety when discussing symptoms

100. Which intervention would be most therapeutic for the nurse to make? 

a. Encourage him to try to walk


b. Tell him there is nothing wrong 
c. Avoid focusing on his physical symptoms 
d. Help him follow through with the physical therapy plan
1. A client is admitted with Wernicke's encephaiopathy. The nurse anticipates
that the first physician's order will include:

a. Ordering an MRI
b. Administering a steroid medication, such as Decadron
c. Giving thiamine 100 mg IM STAT
d. Ordering an EEG

2. Which of the following statements, if made by a four year old child whose
brother just died of cancer, would be age-appropriate?

a. "I know i will never see my mother again."


b. "I'm glad my mother isn't crying anymore."
c. "I can't wait to go get pizza with my brother."
d. "i know where my brother is buried."

3. A patient who has AIzheimer's disease is told by the nurse to brush his
teeth. He shouts angrily, "Tomato soup!" Which of the following actions by the
nurse would be correct?

a. Focusing on the emotional reaction


b. Clarifying the meaning of his statement
c. Giving him step-by-step directions
d. Doing the procedure for him

4. A nurse should teach a patient who is taking chlorpromazine (Thorazine) to


avoid:

a. Exposure to the sun


b. Swimming in a chlorinated pool
c. Drinking fluids high in sodium
d. Eating foods such as chocolate and aged cheese

5. in caring for a psychotic patient who is experiencing hallucinations, which


of the following interventions is considered critical? 

a. Setting fewer limits in order to allow for more expressions of feeling


b. Maintaining constant observation.
c. Providing more frequent opportunities for interaction with others.
d. Constantly negating the patient's hallucinatory Ideations.

6. A 22-year-old client is being admitted with a diagnosis of brief psychotic


disorder. Two weeks ago, his girlfriend broke off their engagement and
cancelled the wedding. Given the Diagnosis and Statistical Manual of Mental
Disorders, edition, text' revised (DSM-IV-TR) criteria for this disorder the
nurse expects to find which of the following data during the interview with the
client?

a. Current treatment for pneumonia


b. Regular use of alcohol and marijuana
c. Evidence of delusions and hallucinations
d. A history of chronic depression

7. A set of monozygotic twins who are 23 years old have begun attending
groups at mental health center. One twin is diagnosed with schizophrenia. Her
twin has no diagnoses but has been experiencing significant anxiety since
becoming engaged. In counseling the engaged twin, it would be crucial to
include which of the following tacts?

a. Her future children will be at risk for developing schizophrenia


b. She may have a predisposition for schizophrenia
c. One of her parents may develop schizophrenia later in life
d. It is unlikely that she wil! develop schizophrenia, at her age

8. A client tells the nurse that her co-workers are sabotaging the computer.
When the nurse asks questions, the client becomes argumentative. This
behavior shows personality traits associated with which of the following
personality disorders?

a. Antisocial
b. Histrionic
c. Paranoid
d. Schizotypal

9. Which of the following types of behavior is expected from a client diagnosed


with paranoid personality disorder?

a. Eccentric
b. Exploitative
c. Hypersensitive
d. Seductive

10. A nurse is reviewing the serum laboratory test results for a client with
sickle cell anemia. The nurse finding that which of the following values is
elevated?

a. Hemoglobin F
b. Hemoglobin S
c. Hemoglobin C
d. Hemoglobin a

11. A parent with a daughter with bulimia nervosa asks a nurse, "How can my
child have an eating disorder when she isn't underweight?" Which of the
following responses is best?

a. "A person with bulimia nervosa can maintain a normal weight."


b. It's hard to face this type of problem in a person you love."
c. "At first there is no weight loss; it comes later In the disease."
d. "This is a serious problem even though there is no weight loss."

12. A nurse is assessing an adolescent girl recently diagnosed with an eating


disorder and symptoms of bulimia nervosa. Which of the following findings is
expected based on laboratory test results?

a. Hypocalcemia
b. Hypoglycemia
c. Hypokalemia
d. Hypophosphatemia

13. Which of the following complications of bulimia nervosa Is life


threatening?

a. Amenorrhea
b. Bradycardia
c. Electrolyte Imbalance
d. Yellow skin

14. A nurse is talking to a client with bulimia nervosa about the complications
of Laxative abuse. Which of the foilowing complications should be included?

a. Loss of taste
b. Swollen glands
c. Dental problems
d. Malabsorption of nutrients

15. A nurse is assessing a client to determine the distress experienced after


binge eating. Which of the following symptoms are typical after bingeing?

a. Ageusia
b. Headache
c. Pain
d. Sore throat
16. Which of the following difficulties are frequently found in families with a
member who has bulimia nervosa?

a. Mental Illness
b. Multiple losses
c. Chronic anxiety
d. Substance abuse

17. A client with anorexia nervosa tells a nurse, "My parents never hug me or
say I've done anything right." Which of the following Interventions is the best
to use with this family?

a. Teach the family principles of assertive behavior.


b. Discuss the difficulties the family has in social situations.
c. Help the family convey a positive attitude toward the client.
d. Explore the family's ability to express affection appropriately.

18. A client with anorexia nervosa tells a nurse she always feels fat. Which of
the following interventions is the best for this client?

a. Talk about how important the client is.


b. Encourage her to look at herself in a mirror.
c. Address the dynamics of the disorder.
d. Talk about how she's different from her peers.

Ms. J.K. is a 24-year old woman admitted to the neurosurgery floor 2 days following a
hypophysectomy for a pituitary tumor. She is alert, oriented, and eager to return to her
job as an executive to the hospital director. She is alert, oriented and eager to return to
her job as an executive assistant to the hospital director. She calls the nurse to her
room to express her concern about the frequency of urination she is experiencing, as
well as the feeling of weakness that began this morning.

19. The most likely cause of her chief complaint this morning is 

a. A decrease in postoperative stress causing poiyuria


b. The onset of diabetes mellitus, an unusual complication
c. An expected result of the removal of the pituitary gland
d. A frequent complication of the hypophysectomy

20. Following hypophysectomy, patients require extensive teaching regarding


this major alteration in their lifestyle 

a. Abnormal distribution of body hair


b. Lifetime dependency on hormone replacement
c. The need to drink many fluids to replace those lost
d. The need to undergo repeat surgical procedures

21. The Glasgow coma scale is used to .evaluate the level of consciousness in
the neurological and neurological patients. The three assessment factors
included in this scale are:

a. pupil size, response to pain, motor responses


b. Pupil size, verbal response, motor response
c. Eye opening, verbal response, motor response
d. Eye opening, response to pain, motor response

J.E, is an 18-year old freshman admitted to the ICU following a motor vehicle accident
in which he sustained multiple trauma including a ruptured spleen, myocardial
contusion, fractured pelvis, and fractured right femur. He had a mild contusion, but is
alert and oriented. His vital signs BP 120/80, pulse 84, respirations 12, and
temperature 99 F orally.

22. The nurse will monitor J.E. for the following signs and symptoms:

a. Change in the levei of consciousness, tachypnea, tachycardia, petechiae


b. Onset of chest pain, tachycardia, diaphoresis, nausea and vomiting
c. Loss of consciousness, bradycardia, petechiae, and severe leg pain
d. Change in leve! of consciousness, bradycardia, chest pain and oliguria

23. Appropriate nursing interventions for J.E. would be

a. Skin care and position q2h and prn; maintain alignment of extremities; respiratory
exercises
b. Skin care/bathe daily; passive leg exercises daily; respiratory therapy for
intermittent positive pressure breathing therapy
c. Skin care and position q2h; teach use of overhead trapeze; respiratory exercises,
and intermittent positive pressure breathing q2h
d. Skin care q2h; teach use of overhead trapeze; respiratory exercises; use pressure
relief devices

Ms. J., a 34-year old white female, is admitted via the emergency room complaining of
abdominal pain, fatigue, anorexia, muscle cramping, and nausea. She is a diabetic who
been managed at 30 U NPH insulin every AM and a 1200-calorie ADA diet. Her glucose
in ER 700 mg/dL. Regular insulin 30 U was given and a repeat glucose were drawn.
Results were not avaiIable upon transfer to the unit.

24. Given the above Information, which nursing activities should be highest
priority?

a. Monitoring vita i signs


b. Obtaining blood glucose results
c. Assessing neurological status
d. Assessing pedai pulses and feet

25. The nurse received the lab results from the biood sample drawn in ER. Her
glucose is now-100. However, her WBC count is 25,000 mm3. What conclusion
can the nurse draw basing on this information?

a. Lab results are within normal limits, no action Is necessary


b. Her diabetes is out of control
c. insulin administration increase WBC count
d. Infection has increased her insulin needs

26. Later that evening, Ms. J's abdominal pain increased in intensity. A
diagnosis of appendicitis is made and Ms. J is scheduled for surgery in the
morning. The physician has written the following orders:

-NPO after midnight


-At 6 AM start-ari iV of D5W to be'infused at 250 ml/hr
-15 U NPH insulin at 6AM
-Draw FBS prior to initiating iV fluids

The statement that best describe the rationale for these orders Is:

a. To provide calories to offset the patient being NPO


b. To prevent a hypoglycemic reaction
c. To prevent a fluid volume deficit
d. To assist with the body's response to stress

27. When ambulating a client following surgical removal of a protruded


intervertebral lurnbar disc, the nurse would do which of the following?

a. Maintain proper body alignment


b. Administer anaigesia after walking
c. Provide a cane for support
d. Immobilize the head and neck

28. Which of the following point scores on the post anesthesia chart, indicates
that the client has fulfilled minimal criteria for discharge from the PACU?

a. One point In each of the five areas .for a total score of 5.


b. One point in at least three areas" respiratory, circulatory, and consciousness - for a
total of 3
c. A total score for the five areas of 7 or.above.
d. Two points each in each of the five areas for a total score of 10.

29. Which of the following statements would be the nurse's response to a


famiiy member asking questions about a client's transient ischemic attack
(TIA)?

a. "I think you should ask the doctor. Would you like me to cail him for you?"
b. " The blood supply to the brain has decreased causing permanent brain damage."
c. "It Is a temporary interruption in the blood flow to the brain."
d. "TIA means a transient ischemic attack."

30. While receiving radiation therapy for the treatment of breast cancer, a
client complains of dysphagia and skin texture changes, at the radiation site.
Which of the following instructions would be most appropriate to suggest to
minimize the risk of complications, and promote healing?

a. Wash the radiation site vigorously with soap and water to remove dead cells.
b. Eat a diet high in protein and calories to optimize tissue repair.
c. Apply coo! compresses to the radiation site to reduce edema,
d. Drink warm fluids throughout the day to relieve discomfort in swallowing.

31. A client using an over-the counter nasal decongestant spray reports


unrelieved and worsening nasal congestion. The nurse should instruct the
client to do which of the following?

a. Switch to a stronger dosage of the medication.


b. Discontinue the medication for a few weeks
c. Use the spray more frequently
d. Combine the spray with an oral decongestant.

32. Following a thyroidectomy, the client experiences.hemorrhage. The nurse


would prepare for which of the following emergency Interventions?

a. intravenous administration of calcium


b. insertion of an oral airway
c. Creation of a tracheostomy
d. Intravenous administration of thyroid hormone

33. After a client signs the form, giving informed consent for surgery and the
physician !eaves the room, the client asks the nurse, "When will this hotel
bring me some food?" After confirming that the client is confused, which of
the following would be the nurse's priority action?

a. Reporting that the consent has been obtained from a confused client.
b. Teaching preoperative moving, coughing, and deep-breathing,exercises.
c. Inserting a bladder catheter to urine output.
d. Administering preoperative medication immediately ,

34. At 16 weeks gestation, no fetal heart rate was detected during assessment
of a pregnant patient. An ultrasound confirmed a hydatidiform molar
pregnancy. Which of the following actions should the nurse tell the patient to
expect during her one-year follow-up?

a. Multiple serum chorionic gonadotropin levels will be drawn


b. An Intrauterine device will be used to decrease vaginal bleeding
c. Pregnancy will be restricted for another year
d. Oral contraceptives will not be prescribed because they will increase the risk' of
cancer

35. Thirty minutes after the nurse removes a nasogastric tube that has been
In piace for seven days, the patient experiences epistaxis (nosebleed). Which
of the following nursing actions is most appropriate to control the bleeding?

a. Apply pressure by pinching the anterior portion of the for five to ten minutes
b. Place the patient in a sitting position with the neck hyperextended
c. Pack the nostrils with gauze and keep the gauze in piace for four to five days
d. Apply ice compresses to the patient's forehead and back of the neck

36. The staff nurse calls a physician regarding an order to administer digoxin
(Lanoxin) to a patient with a pulse of 55 and a serum potassium levei of 2.9
mEq/L The physician says to give the medication, as ordered .The staff nurse's
best response would be

a. "I'll give the medication but you wiil still be responsbIe if anything happens to the
patient."
b. "I will not give this medication."
c. '"I think we should discuss this with the nursing supervisor."
d. "I'm sorry, but if you want the medication given, you will have to give it yourself."

37. During the night, shift report, the charge nurse learns that an elderly
patient has become very confused and is shouting obscenities and undressing
himseif. Which of the following actions is the most appropriate Initial nursing
response?

a. Restrain the patient with a Posey jacket


b. Medicate the patient with haloperidol (Haldol) as ordered.
c. Notify the physician
d. Complete a nursing assessment of the patient

38. When a woman is 10weeks pregnant which of the following hematology


test results would need further Investigation?

a. Hemoglobin level of 9 mg/dL 


b. white blood cell count of 15,000/cu mm 
c. platelet count of 200,000/cu mm
d. red blood cell count of 4,200,000/ cu mm

39. Which of the foitowing techniques would a nurse use when interviewing a
94-year-old patient?

a. Using a low-pitched voice


b. Enunciating each word .slowly
c. Varying voice intonations
d. Reinforcing the words with pictures .

40. A patient who is receiving total parenteral nutrition has an elevated blood
glucose eve! and is to be administered intravenous insulin. Which of the
following types of insulin should a nurse has available?

a. Isophane insulin (NPH)


b. Regular insulin (Humulin R)
c. Insulin zinc suspension (Lente)
d. Semi-Lente Insulin (Semiterd)

41. A nurse is taking history from a patient who has just been admitted to the
hospital withl an acute myocardia! infarction. Which of the following questions
would be most important for the nurse to ask?

a. "At what time did the pain start?"


b. "When did you eat your last meal?"
c. "Have you experienced a pounding headache?"
d. "Did you feel fluttering in your chest"

42. An infant who weighs 11 lbs. is to receive 750 mg of an antibiotic in a 24-


hour period. The liquid antibiotic comes in a concentration of 125 mg/5ml. If
the antibiotic were to be given three times each day. how many ml would the
nurse administer with each dose?

a. 2
b. 5
c. 6.25
d. 10

43. Spasm of the neck muscles developed in a patient who is taking


phenothiazine (Nemazine). Which of the following medications should the
nurse administer?

a. Vistaril) 
b. Acetaminophen (Tyienol) 
c. Acetylsalicylic acid (Aspirin) 
d. Benztropine mesyiate (Cogentin) 

Mr. Anthony Malailinelii is a 54-year old truck driver. He is admitted for possible gastric
ulcer, He is a heavy smoker.

44. When discussing his smoking habits with Mr. Martinelli. the nurse should
advise him to:

a. Smoke low-tar, filter cigarettes


b. Smoke cigars instead
c. Smoke only right after meals
d. Chew gum Instead 

45. As the nurse preparing Ivlr. Martinelii for gastric analysis. You should
know which of the following Is not.correct concerning this test

a. The patient Is fasting 12 hours prior to test


b. Gastric contents are aspirated via a tube 
c. Smoking for 8 hours prior to test is not allowed 
d. Various position changes are necessary during the test 

46. Mr. Martinelli had an Hgb of 9.8. You would not find which of the following
assessments in a patient with severe anemia? 

a. Pallor
b. Cold sensitivity 
c. Fatigue
d. Dyspnea only on exertion

47. When you report on duty, your team leader tells you that Mr. MartineHi
accidentally received 1000 ml of fluids in 2 hours and that you are to be alert
for signs of circulatory overload. Which of the following signs would not be
likely to occur? 
a. moist gurgling respirations 
b. Weak, slow pulse 
c. Distended neck veins
d. Dyspnea and coughing

48. A new staff nurse is on an orientation tour with the head nurse. A client
approaches her and says, "I don't belong here. Please try to get me out." The
staff nurse's best response would be: 

a. "What would you do if you were out of the hospital?" 


b. "I am a. new staff member, and I'm on a tour. I'll come back and talk with you
later."
c. "I think you should talk to the head nurse about that.' 
d. "I can't do anything about that."

49. A 50 year-old male client has a history of many hospitalizations for


schizophrenic disorder. He has been on long-term phenothiazines (Thorazine),
400 mg/day. The nurse assessing this client observes that he demonstrates a
shuffling gait, drooling and exhibits generaj dystonic symptoms.. From these
symptoms and his history, the nurse concludes that the client has developed:

a. Tardive dyskinesia 
b. Parkinsonism 
c. Dystonia 
d. Akathisia

50. A client with antisocial personality disorder tells a nurse "Life has been full
of problems since childhood." Which of the following situations or conditions
would the nurse explore in the assessment?

a. Birth defects 
b. Distracted easily
c. Hypoactive behavior 
d. Substance abuse

51. A client with antisocial personality disorder is trying to manipulate the


healthcare team. Which of the following strategies is important for the staff to
use?

a. Focus on how to teach the client more effective behaviors for meeting basic needs.
b. Help the client verbalize underlying feelings of hopelessness and learn coping skills.
c. Remain calm and don't emotionally respond to the client's manipulative actions.
d. Help the client eliminate the intense desire to have everything in life turn out
perfectly. 

52. A client with antisocial personality disorder is beginning to practice several


socially acceptable behaviors in the group setting. Which of the following
outcomes will result from this change?

a. Fewer panic attacks


b. Acceptance of reality 
c. Improved self-esteem
d. decreased physical symptoms

53. Which of the following discharge instructions would be most accurate to


provide to a female client who has suffered a spinal cord injury at the C4
level?

a. After a spinal cord injury, women usually remain fertile; therefore, you may consider
contraception if you don't want to become pregnant.
b. After a spinal cord injury, women usually are unable to conceive a child. 
c. Sexual intercourse shouldn't be different for you. 
d. After a spinal cord injury, menstruation usually stops.

54.A client with chronic obstructive pulmonary disease (COPD) tells the nurse,
"I no longer have enough energy to make love to my husband." Which of the
following nursing interventions would be most appropriate?

a. Refer the couple to a sex therapist.


b. Advise the woman to seek a gynecologic consult
c. Suggest methods and measures that facilitate sexual activity. 
d. Tell the client, "if you talk this over with your husband, he will understand.

55. A cllent tells the nurse she is having her menstrual period every 2 weeks
and it lasts for 1 week. Which of the following conditions is best defined by
this menstrual pattern?

a. Amenorrhea
b. Dyspareunia
d. menororrhagia 
d. metrorrhagia

56. A nurse has just been toSd by a. physician that an order has been written
to administer an iron injection to an adult client. The nurse plans to administer
the medication In which of the following locations?

a. In the gluteal muscle using Z-track technique


b. In the deltoid muscle using an air lock
c. In the subcutaneous fesue of the abdomen
d. in the anterior lateral thigh using a 5/8 inch needle '

57. A 59-year-old patient with a diagnosis of delirium is admitted to the


hospital. To evaluate the cause of a patient's delirium, blood is sent to the
laboratory for analysis. The results are as follows: M,a+ 1.56, Cr 100. K4' 4.0,
C0221, BUN 86, glucose 100. Based on these laboratory result, the nurse
should record which of the following nursing diagnoses on the patient's care.
plan? 

a. Alteration in patterns of urinary elimination.


b. Fluid volume deficit
c. Nutritional deficit: less than body requirements 
d. Self-care deficit: feeding

58. The nurse knows that gender Is part of one's identity. Which of the
following events signifies when gender is first ascribed? 

a. A baby is born 
b. A child attends school 
c. A child receives sex-specific toys 
d. A child receives sex-specific clothing

You might also like